171
2014 BAR EXAMINATIONS REMEDIAL LAW NOTES Part III By: ELEUTERIO L. BATHAN Presiding Judge, RTC, Br. 92, (NCJR) Quezon City Acting Presiding Judge, RTC, Br. 5, Lemery, Batangas Judge Designate, RTC, Br. 39, Calapan City, Or. Mindoro Judge Designate, RTC, Br. 40, Calapan City, Or. Mindoro Former Executive/Presiding Judge, MTCC, Br. 2, Batangas City Former Acting Judge: MTCC, Br. 1, Batangas City; MTC, Lian, Batangas; Sablayan, Occidental Mindoro; Former Judge Designate, MTC, Romblon, Romblon Law Professor, Remedial Law Review and Political Law University of Batangas (U.B.) (Batangas & Lipa City Campuses) MCLE Lecturer Author, Handbook on Local Governance (Rex Book Store, Inc.) Awardee, Judicial Excellence (2014), Rotary Club of Sta. Mesa District 3780, Quezon City Outstanding Graduate (2014), SSC-R, College of Law Awardee, Quezon City RTCJA 2012 Outstanding Judge Former Member, Sangguniang Bayan, San Jose, Batangas Former Law Practitioner A.B.; Ll.B., San Sebastian College-R (Manila) ---0---

ATT_1441627983757_Remedial Law part three.pdf

Embed Size (px)

Citation preview

2014 BAR EXAMINATIONS

REMEDIAL LAW NOTES Part III

By:

ELEUTERIO L. BATHAN Presiding Judge, RTC, Br. 92, (NCJR) Quezon City

Acting Presiding Judge, RTC, Br. 5, Lemery, Batangas

Judge Designate, RTC, Br. 39, Calapan City, Or. Mindoro

Judge Designate, RTC, Br. 40, Calapan City, Or. Mindoro

Former Executive/Presiding Judge, MTCC, Br. 2, Batangas City

Former Acting Judge: MTCC, Br. 1, Batangas City; MTC, Lian,

Batangas; Sablayan, Occidental Mindoro; Former Judge Designate, MTC, Romblon, Romblon

Law Professor, Remedial Law Review and Political Law

University of Batangas (U.B.) (Batangas & Lipa City Campuses)

MCLE Lecturer

Author, Handbook on Local Governance (Rex Book Store, Inc.) Awardee, Judicial Excellence (2014), Rotary Club of Sta. Mesa

District 3780, Quezon City

Outstanding Graduate (2014), SSC-R, College of Law

Awardee, Quezon City RTCJA 2012 Outstanding Judge

Former Member, Sangguniang Bayan, San Jose, Batangas

Former Law Practitioner A.B.; Ll.B., San Sebastian College-R (Manila)

---0---

REMEDY OF PLAINTIFF IF DEFENDANT’S

MOTION TO DISMISS COMPLAINT

IS GRANTED

If defendant’s motion to dismiss is granted, the remedy of the

plaintiff is to appeal the order of dismissal by way of filing a notice of

appeal, because an order of dismissal is a final order (See Rules 40 and

41, 1997 Revised Rules of Civil Procedure).

An order of dismissal, whether correct or not, is a final order. It is

not interlocutory because the proceedings were terminated; it leaves

nothing more to be done by the lower court. Therefore, the remedy of the

plaintiff is to appeal the order (Dael vs. Spouses Beltran, 4).

The plaintiff may resort first to the filing of a motion for

reconsideration before filing a notice of appeal. In situation like this, filing

a motion for reconsideration is optional. It is discretionary on the part of

the aggrieved party or plaintiff.

If he chooses to file answer, he may avail of an extension of time

to file one (See above-discussions; See also Virata vs. Sandiganbayan).

---0---

REMEDY OF DEFENDANT IF HIS

MOTION TO DISMISS COMPLAINT IS DENIED

If the defendant’s motion to dismiss is denied, the following are

the remedies available to the defendant:

1. upon denial of the motion for reconsideration to the order

of denial, he may file a petition for certiorari under Rule

65; or to

2. file an answer

Though filing a petition for certiorari under Rule 65 is the most

dangerous option, but still, defendant can avail of this remedy. The danger

or the risk is that instead of focusing on the alternative mode which is the

filing of an answer, defendant, upon plaintiff’s motion might be declared

by the court in default for failure to file an answer. So, if he chooses to

resort to the filing of certiorari under Rule 65, and in order to avoid the

possibility of declaring him in default later on, he shall see to it that his

petition should be accompanied with an application for ancillary relief of

temporary restraining order and/or preliminary injunction, but also take

note of the limitations provided under A.M. No. 07-7-12-SC dated

December 4, 2007 [effective December 27, 2007] amending Rule 65 of the

1997 Revised Rules of Civil Procedure).

As a general rule, an order denying a motion to dismiss is merely

interlocutory and cannot be subject of appeal until final judgment or order

is rendered (Newsweek, Inc vs. IAC,).

It is interlocutory in the sense that it does not finally dispose of the

case, and, in effect, directs the case to proceed until final adjudication by

the court (Marmo et al., vs. Anacay,).

The ordinary procedure to be followed in such a case is to file an

answer, go to trial and if the decision is adverse, reiterate the issue on

appeal from the final judgment, but this may be subject to certain

exceptions (Newsweek, Inc vs. IAC,).

EXCEPTIONS:

Only when the court issues an order outside or in excess of

jurisdiction or with grave abuse of discretion, and the remedy of appeal

would not afford adequate and expeditious relief, will certiorari be

considered an appropriate remedy to assail an interlocutory order (Heirs of

Hinog vs. Melicor,).

If the court, in denying the motion to dismiss, acts without or in

excess of jurisdiction or with grave abuse of discretion, then certiorari or

prohibition lies. The reason is that it would be unfair to require the

defendant to undergo the ordeal and expense of a trial if the court has no

jurisdiction over the subject matter, or is not the court of proper venue, or

if the denial of the motion to dismiss is made with grave abuse of

discretion or a whimsical and capricious exercise of judgment. In such

cases, the ordinary remedy of appeal cannot be plain and adequate. The

following are a few examples of the exceptions to the general rule

(Newsweek, Inc vs. IAC,).

In De Jesus vs. Garcia, , upon the denial of a motion to dismiss

based on lack of jurisdiction over the subject matter, the High Court

granted the petition for certiorari and prohibition against the City Court of

Manila and directed the respondent court to dismiss the case.

However, in Bank of America vs. CA, , it was held that the remedy

of the defendant is to file an answer to the complaint, proceed to trial and

await judgment before making an appeal instead of petition of certiorari

under Rule 65.

The High Court in Bank of America:

“It is a well-settled rule that the order denying the

motion to dismiss cannot be the subject of petition for

certiorari. Petitioners should have filed an answer to the

complaint, proceed to trial and await judgment before

making an appeal. As repeatedly held by this Court:

“An order denying a motion to dismiss is

interlocutory and cannot be the subject of the

extraordinary petition for certiorari or

mandamus. The remedy of the aggrieved party

is to file an answer and to interpose as defenses

the objections raised in his motion to dismiss,

proceed to trial, and in case of an adverse

decision, to elevate the entire case by appeal in

due course. xxx Under certain situations,

recourse to certiorari or mandamus is

considered appropriate, i.e., (a) when the trial

court issued the order without or in excess of

jurisdiction; (b) where there is patent grave

abuse of discretion by the trial court; or (c)

appeal would not prove to be a speedy and

adequate remedy as when an appeal would not

promptly relieve a defendant from the injurious

effects of the patently mistaken order

maintaining the plaintiff’s baseless action and

compelling the defendant needlessly to go

through a protracted trial and clogging the court

dockets by another futile case.”

Records show that the trial court acted within its

jurisdiction when it issued the assailed Order denying

petitioners’ motion to dismiss. Does the denial of the

motion to dismiss constitute a patent grave abuse of

discretion? Would appeal, under the circumstances, not

prove to be a speedy and adequate remedy? We will

resolve said questions in conjunction with the issues raised

by the parties.

First issue. Did the trial court commit grave abuse of

discretion in refusing to dismiss the complaint on the

ground that plaintiffs have no cause of action against

defendants since plaintiffs are merely stockholders of the

corporations which are the registered owners of the vessels

and the borrowers of petitioners?

No. Petitioners’ argument that private respondents,

being mere stockholders of the foreign corporations, have

no personalities to sue, and therefore, the complaint should

be dismissed, is untenable. A case is dismissible for lack of

personality to sue upon proof that the plaintiff is not the

real party-in-interest. Lack of personality to sue can be

used as a ground for a Motion to Dismiss based on the fact

that the complaint, on the face thereof, evidently states no

cause of action. In San Lorenzo Village Association, Inc.

vs. Court of Appeals, this Court clarified that a complaint

states a cause of action where it contains three essential

elements of a cause of action, namely: (1) the legal right of

the plaintiff, (2) the correlative obligation of the defendant,

and (3) the act or omission of the defendant in violation of

said legal right. If these elements are absent, the complaint

becomes vulnerable to a motion to dismiss on the ground of

failure to state a cause of action. To emphasize, it is not the

lack or absence of cause of action that is a ground for

dismissal of the complaint but rather the fact that the

complaint states no cause of action. “Failure to state a

cause of action” refers to the insufficiency of allegation in

the pleading, unlike “lack of cause of action” which refers

to the insufficiency of factual basis for the action. “Failure

to state a cause of action” may be raised at the earliest

stages of an action through a motion to dismiss the

complaint, while “lack of cause of action” may be raised

any time after the questions of fact have been resolved on

the basis of stipulations, admissions or evidence presented.

In the case at bar, the complaint contains the three

elements of a cause of action. It alleges that: (1) plaintiffs,

herein private respondents, have the right to demand for an

accounting from defendants (herein petitioners), as trustees

by reason of the fiduciary relationship that was created

between the parties involving the vessels in question; (2)

petitioners have the obligation, as trustees, to render such

an accounting; and (3) petitioners failed to do the same.

Petitioners insist that they do not have any obligation

to the private respondents as they are mere stockholders of

the corporation; that the corporate entities have juridical

personalities separate and distinct from those of the private

respondents. Private respondents maintain that the

corporations are wholly owned by them and prior to the

incorporation of such entities, they were clients of

petitioners which induced them to acquire loans from said

petitioners to invest on the additional ships.

We agree with private respondents. As held in the

San Lorenzo case,

“xxx assuming that the allegation of facts

constituting plaintiffs’ cause of action is not as

clear and categorical as would otherwise be

desired, any uncertainty thereby arising should

be so resolved as to enable a full inquiry into the

merits of the action.”

As this Court has explained in the San Lorenzo case,

such a course, would preclude multiplicity of suits which

the law abhors, and conduce to the definitive determination

and termination of the dispute. To do otherwise, that is, to

abort the action on account of the alleged fatal flaws of the

complaint would obviously be indecisive and would not

end the controversy, since the institution of another action

upon a revised complaint would not be foreclosed.”

In Enriquez vs. Macadaeg, , upon the denial of a motion to

dismiss based on improper venue, this Court granted the petition for

prohibition and enjoined the respondent judge from taking cognizance of

the case except to dismiss the same.

In Manalo vs. Mariano, , upon the denial of a motion to dismiss

based on bar by prior judgment, this Court granted the petition for

certiorari and directed the respondent judge to dismiss the case.

In Yuviengco vs. Dacuycuy, , upon the denial of a motion to

dismiss based on the Statute of Frauds, the High Court granted the petition

for certiorari and dismissed the amended complaint.

Take note however, that in Marmo et al., vs. Anacay, , the High

Court ruled that the trial court did not grave abuse its discretion when it

denied the motion to dismiss the complaint for failure of the plaintiff to

implead indispensable parties.

---0---

EFFECT OF DISMISSAL

IF MOTION TO DISMISS IS GRANTED

If the dismissal is based on the grounds that: (1) the cause of action

is barred by a prior judgment or by the statute of limitations (Paragraph

(f), Section 1, Rule 16, 1997 Revised Rules of Civil Procedure); (2) the

claim or demand set forth in the plaintiff’s pleading has been paid, waived,

abandoned, or otherwise extinguished (Paragraph (h), Section 1, Rule 16,

1997 Revised Rules of Civil Procedure); and (3) the claim on which the

action is founded is unenforceable under the provisions of the statute of

frauds; and (Paragraph (i), Section 1, Rule 16, 1997 Revised Rules of

Civil Procedure), subject to the right of appeal, shall bar the refiling of

the same action or claim (Section 5, Rule 16, 1997 Revised Rules of Civil

Procedure). The dismissal is with prejudice.

Strongworld Construction Corporation vs. Judge Perello, said:

“Section 1, Rule 16 of the 1997 Revised Rules of

Civil Procedure enumerates the grounds for which a

Motion to Dismiss may be filed, viz.:

SECTION 1. Grounds. – Within the time

for but before filing the answer to the complaint

or pleading asserting a claim, a motion to

dismiss may be made on any of the following

grounds:

(a) That the court has no jurisdiction over the

person of the defending party;

(b) That the court has no jurisdiction over the

subject matter of the claim;

(c) That venue is improperly laid;That the

plaintiff has no legal capacity to sue;

(d) That there is another action pending

between the same parties for the same

cause;

(e) That the cause of action is barred by a

prior judgment or by the statute of

limitations;

(f) That the pleading asserting the claim states

no cause of action;

(g) That the claim or demand set forth in the

plaintiff’s pleading has been paid, waived,

abandoned, or otherwise extinguished;

(h) That the claim on which the action is

founded is unenforceable under the

provisions of the statute of frauds; and

(i) That a condition precedent for filing the

claim has not been complied with.

Section 5 of the same Rule, recites the effect of a

dismissal under Sections 1(f), (h), and (i), thereof, thus:

SEC. 5. Effect of dismissal. – Subject to

the right of appeal, an order granting a motion to

dismiss based on paragraphs (f), (h), and (i) of

section 1 hereof shall bar the refiling of the

same action or claim.

Briefly stated, dismissals that are based on the

following grounds, to wit: (1) that the cause of action is

barred by a prior judgment or by the statute of limitations;

(2) that the claim or demand set forth in the plaintiff’s

pleading has been paid, waived, abandoned or otherwise

extinguished; and (3) that the claim on which the action is

founded is unenforceable under the provisions of the statute

of frauds, bar the refiling of the same action or claim.

Logically, the nature of the dismissal founded on any of the

preceding grounds is “with prejudice” because the

dismissal prevents the refiling of the same action or claim.

Ergo, dismissals based on the rest of the grounds

enumerated are without prejudice because they do not

preclude the refiling of the same action.

Verily, the dismissal of petitioners’ Complaint by the

court a quo was not based on any of the grounds specified

in Section 5, Rule 16 of the 1997 Revised Rules of Civil

Procedure; rather, it was grounded on what was

encapsulated in Section 1(g), Rule 16 of the 1997 Revised

Rules of Civil Procedure. As the trial court ratiocinated in

its 9 January 1998 Order, the Complaint is not prosecuted

by the proper party in interest. Considering the heretofore

discussion, we can say that the order of dismissal was

based on the ground that the Complaint states no cause of

action. For this reason, the dismissal of petitioners’

Complaint cannot be said to be a dismissal with prejudice

which bars the refiling of the same action.

As has been earlier quoted, Section 1(h), Rule 41 of

the 1997 Revised Rules of Civil Procedure mandates that

no appeal may be taken from an order dismissing an action

without prejudice. The same section provides that in such

an instant where the final order is not appealable, the

aggrieved party may file an appropriate special civil action

under Rule 65.”

-0-0-0-

DISMISSAL ON VARIOUS GROUNDS AND ITS EFFECT

(WITH PREJUDICE AND WITHOUT PREJUDICE)

On filing of initiatory pleading, If act of lawyer or counsel

constitute willful and deliberate forum shopping, shall be a ground for the

summary dismissal of the case with prejudice (Section 5, Rule 7, 1997

Rules of Court).

Regular Procedure (1997 Rules of Court): plaintiff’s failure to

appear at the pre-trial conference, the dismissal is with prejudice, unless

otherwise ordered by the court (Section 5, Rule 18). General Rule:

dismissal is always with prejudice, unless the order says that dismissal is

without prejudice. If the order is silent, the dismissal is with prejudice;

PLAINTIFF’S failed – defendant may be allowed to present evidence on

his counter-claim, if any; DEFENDANT’S failed – plaintiff’s presentation

of ex-parte evidence, and the court to render judgment on the basis

thereof.

Summary Procedure: plaintiff’s failure to appear at the

preliminary conference can cause the dismissal of the action. The

dismissal is with prejudice, unless otherwise ordered by the court (Apply

Section 5, Rule 18, 1997 Rules of Court, suppletory application). General

Rule: dismissal is always with prejudice, unless the order says that

dismissal is without prejudice. If the order is silent, the dismissal is with

prejudice.

Plaintiff’s failure to file pre-trial brief (in ordinary cases) has the

same effect of failure to appear at the pre-trial conference (See: Section 6,

Rule 18, 1997 Rules of Court).

Plaintiff’s failure to file preliminary conference brief (pre-trial

brief under summary procedure) has the same effect with failure to appear

at the preliminary conference (See: Rule 18, 1997 Rules of Court

suppletory application).

Rule on Small Claims: Failure of plaintiff to appear at the trial can

cause the dismissal of the claim. Take note: The dismissal is always

without prejudice (Section 18, Rule on Small Claim).

Rules of Procedure for Environmental Cases – repeated and

unjustified failure of plaintiff to appear at the pre-trial conference, or to

file pre-trial brief, the court shall dismiss the case. The dismissal shall be

without prejudice (Section 7, Rule 3 of the Rule).

Lack of certification against forum shopping under Section 5,

Rule 7, 1997 Rules, shall be cause for the dismissal of the case – dismissal

is without prejudice, unless otherwise provided.

---0---

DEFENDANT MAY FILE MOTION

FOR EXTENSION OF TIME TO ANSWER

This discussion is confined only to all (civil) cases governed by the

rules on regular procedure, excluding the cases governed by the Rules on

Summary Procedure (See separate discussion on the matter).

If after plaintiff has complied with the order of the court requiring

him to submit a bill of particulars, and/or defendant’s motion to dismiss

(and/or the motion for reconsideration thereto) is denied, the defendant

has three (3) more remaining remedial options. The defendant may either:

(1) may still ask for extension of time to file answer; or

(2) file an answer; or

(3) he may deliberately disregard the complaint by inaction.

The defendant may still ask for an extension of time to file answer

or motion to dismiss. This is a non-litigious motion which can be done and

filed ex-parte. Some trial courts do not anymore rule or resolve the

motion for extension of time, and that is, equivalent to allowance. If that is

the case, it is incumbent upon the movant to file the intended pleading on

or before the expiration of the period prayed for within which to file the

same.

The rule says:

1. Upon motion and on such terms as may be just, the court may

extend the time to plead provided in these Rules (Section 11, Rule 11,

1997 Revised Rules of Civil Procedure).

2. The court may also, upon like terms, allow an answer or other

pleading to be filed after the time fixed by the Rules (Paragraph 2,

Section 11, Rule 11, 1997 Revised Rules of Civil Procedure).

---0---

FILING AN ANSWER TO THE COMPLAINT:

If the defendant’s motion to dismiss is denied his other remedial

option aside from filing a certiorari petition (under Rule 65) is to file

answer.

A. When to file answer

The defendant shall file his answer to the complaint within fifteen

(15) days after service of summons, unless a different period is fixed by

the court (Section 1, Rule 11, 1997 Revised Rules of Civil Procedure).

Where the defendant is a foreign private juridical entity and service

of summons is made on the government official designated by law to

receive the same, the answer shall be filed within thirty (30) days after

receipt of summons by such entity (Section 2, Rule 11, 1997 Revised

Rules of Civil Procedure).

When the plaintiff files an amended complaint as a matter of right,

the defendant shall answer the same within fifteen (15) days after being

served with a copy thereof (Section 3, Rule 11, 1997 Revised Rules of

Civil Procedure).

Where its filing is not a matter of right, the defendant shall answer

the amended complaint within ten (l0) days from notice of the order

admitting the same. An answer earlier filed may serve as the answer to the

amended complaint if no new answer is filed (Paragraph 2, Section 3,

Rule 11, 1997 Revised Rules of Civil Procedure).

B. Defenses and objections not pleaded waived

Defenses and objections not pleaded either in a motion to dismiss

or in the answer are deemed waived (Section 1, Rule 9, 1997 Revised

Rules of Civil Procedure).

C. Contents of answer

An answer shall contain in a methodical and logical form, a plain,

concise and direct statement of the ultimate facts on which the defendant

relies for his defense, omitting the statement of mere evidentiary facts

(Section 1, Rule 8, 1997 Revised Rules of Civil Procedure). If a defense

relied on is based on law, the pertinent provisions thereof and their

applicability to him shall be clearly and concisely stated (Paragraph 2,

Section 1, Rule 8, 1997 Revised Rules of Civil Procedure).

A defendant may set forth two or more statements of defense

alternatively or hypothetically, either in one defense or in separate

defenses (Section 2, Rule 8, 1997 Revised Rules of Civil Procedure).

Facts showing the capacity of a party to sue or be sued or the

authority of a party to sue or be sued in a representative capacity or the

legal existence of an organized association of person that is made a party,

must be averred. A party desiring to raise an issue as to the legal existence

of any party or the capacity of any party to sue or be sued in a

representative capacity, shall do so by specific denial, which shall include

such supporting particulars as are peculiarly within the pleader's

knowledge (Section 4, Rule 8, 1997 Revised Rules of Civil Procedure).

In all averments of fraud or mistake the circumstances constituting

fraud or mistake must be stated with particularity. Malice, intent,

knowledge, or other condition of the mind of a person may be averred

generally (Section 5, Rule 8, 1997 Revised Rules of Civil Procedure).

Whenever a defense is based upon a written instrument or

document, the substance of such instrument or document shall be set forth

in the answer, and the original or a copy thereof shall be attached to the

answer as an exhibit, which shall be deemed to be a part of the answer, or

said copy may with like effect be set forth in the pleading (Section 7, Rule

8, 1997 Revised Rules of Civil Procedure).

When a defense is founded upon a written instrument, copied in or

attached to the corresponding pleading as provided in the preceding

section, the genuineness and due execution of the instrument shall be

deemed admitted unless the adverse party, under oath specifically denies

them, and sets forth what he claims to be the facts, but the requirement of

an oath does not apply when the adverse party does not appear to be a

party to the instrument or when compliance with an order for an inspection

of the original instrument is refused (Section 8, Rule 8, 1997 Revised

Rules of Civil Procedure). In pleading an official document or official act,

it is sufficient to aver that the document was issued or the act done in

compliance with law (Section 9, Rule 8, 1997 Revised Rules of Civil

Procedure).

A defendant must specify each material allegation of fact the truth

of which he does not admit and, whenever practicable, shall set forth the

substance of the matters upon which he relies to support his denial. Where

a defendant desires to deny only a part of an averment, he shall specify so

much of it as is true and material and shall deny only the remainder.

Where a defendant is without knowledge or information sufficient to form

a belief as to the truth of a material averment made to the complaint, he

shall so state, and this shall have the effect of a denial (Section 10, Rule 8,

1997 Revised Rules of Civil Procedure).

Material averment in the complaint, other than those as to the

amount of unliquidated damages, shall be deemed admitted when not

specifically denied. Allegations of usury in a complaint to recover

usurious interest are deemed admitted if not denied under oath (Section

11, Rule 8, 1997 Revised Rules of Civil Procedure).

DENIAL AND CONTESTING THE GENUINENESS AND DUE

EXECTION OF ACTIONABLE DOCUMENT

A. How to deny the genuineness and due execution of an

actionable document?

The defendant is required to make an effective specific denial. This

means that the defendant must declare under oath that he did not sign the

document or that it is otherwise false or fabricated. Neither does the

statement of the answer to the effect that the instrument was procured by

fraudulent representation raise any issue as to its genuineness or due

execution. On the contrary such a plea is an admission both of the

genuineness and due execution thereof, since it seeks to avoid the

instrument upon a ground not affecting either (Songco vs. Sellner,).

B. Case No. 1

Plaintiff alleged in the complaint that defendant is indebted to the

former in the amount of P1M attaching in the complaint several annexes

including the photocopy of the promissory note. In his answer, he alleged

that he has never signed the promissory note attached to the complaint in

his personal and/or individual capacity, thus ineffective, unenforceable

and void for lack of valid consideration. In his verification, he stated that:

(1) that the defendant, after having been duly sworn to in accordance with

law, hereby depose and declare that she is the named defendant in the

above-entitled case; he has cause the preparation of the answer upon facts

and figures supplied by his to his retained counsel; have read each and

every allegations contained therein and hereby certify that the same are

true and correct of her own knowledge and information.

At the pre-trial, plaintiff was able to prove that the defendant

received the amount of P1M as loan after signing the Promissory Note

(Annex A), that defendant, upon receipt of the demand letter made a reply

seeking an extension to pay her obligation. On the Formal Offer of

Exhibits Plaintiff sought the admission of the duplicate original of the PN

on the ground that the original copy could no longer be found. The trial

court initially admitted into evidence the duplicate original of the PN, and

allowed Defendant to amend her answer to conform with this new

evidence. Upon Defendant’s motion for reconsideration arguing that the

duplicate Original PN was not properly identified and there were markings

in the photocopy which were not contained in duplicate original, the trial

court granted the MR and dismissed the case on the ground that Plaintiff

had no longer possessed any proof of Defendant’s alleged indebtedness.

The trial court found that there can be no dispute to the fact that the

allegations in the answer of defendant, she denied generally and

specifically under oath the genuineness and due execution of the

promissory note and by way of special and affirmative defenses herein

states that she never signed the promissory note attached to the complaint

in his personal and/or individual capacity. She also deny generally and

specifically the rest of the allegations. It would be considered that there is

a sufficient compliance of the requirement of the law for specific denial.

Is the denial specific?

Solidbank vs. Del Monte Motor Works, Inc., said defendant’s

denial is not specific. Section 8, Rule 8, 1997 Rules of Court provides that

when an action or defense is founded upon a written instrument, copied in

or attached to the corresponding pleading as provided in the preceding

section, the genuineness and due execution of the instrument shall be

deemed admitted unless the adverse party, under oath, specifically

denies them and sets forth what he claims to be the facts.

Is secondary evidence still needed?

Since Defendant failed to deny specifically the execution of the

promissory note, there was no need for the Plaintiff to present the original

of the promissory note in question. Defendant’s judicial admission with

respect to the genuineness and execution of the promissory note

sufficiently established her liability to Plaintiff regardless of the fact that

Plaintiff failed to present the original of said note. Therefore, there is thus

no need of proof of execution and authenticity with respect to the loan

document because of Defendant’s implied admission of loan transaction.

There is no need to comply with what Section 22, Rule 132 of the Rules of

Court which requires that before a private documents can be received in

evidence, presentation and examination of witnesses to testify to prove its

due execution and authenticity (Solidbank vs. Del Monte Motor Works,

Inc.,).

C. Case No. 2

Defendant’s answer contained the following:

1. The allegations in par. 2, Complaint, on the

existence of the alleged loan of P2-M, and the purported

documents evidencing the same, only the signature

appearing at the back of the promissory note, Annex “A”

seems to be that of herein defendant. However, as to any

liability arising therefrom, the receipt of the said amount of

P2-M shows that the amount was received by another

person, not the herein defendant. Hence, no liability

attaches and as further stated in the special and affirmative

defenses that, assuming the promissory note exists, it does

not bind much less is there the intention by the parties to

bind the herein defendant. In other words, the documents

relative to the loan do not express the true intention of the

parties.

2. (Verification) “I, Defendant, of age, am the

defendant in this case, that I caused the preparation of the

complaint and that all the allegations thereat are true and

correct; that the promissory note sued upon, assuming that

it exists and bears the genuine signature of herein

defendant, the same does not bind him and that it did not

truly express the real intention of the parties as stated in

the defenses”

Is this an effective specific denial as contemplated by law?

No. A reading of Defendant’s Answer, shows that he did not

specifically deny that he signed the loan documents. What he merely

stated in his Answer was that the signature appearing at the back of the

promissory note seemed to be his. Defendant also denied any liability on

the promissory note as he allegedly did not receive the amount stated

therein, and the loan documents do not express the true intention of the

parties. Defendant reiterated these allegations in his “denial under oath,”

stating that “the promissory note sued upon, assuming that it exists and

bears the genuine signature of herein defendant, the same does not bind

him and that it did not truly express the real intention of the parties as

stated in the defenses.

His answer amounts to an implied admission of the due execution

and genuineness of the promissory note. The admission of the genuineness

and due execution of a document means that the party whose signature it

bears admits that he voluntarily signed the document or it was signed by

another for him and with his authority; that at the time it was signed it was

in words and figures exactly as set out in the pleading of the party relying

upon it; that the document was delivered; and that any formalities required

by law, such as a seal, an acknowledgment, or revenue stamp, which it

lacks, are waived by him. Also, it effectively eliminated any defense

relating to the authenticity and due execution of the document, e.g., that

the document was spurious, counterfeit, or of different import on its face

as the one executed by the parties; or that the signatures appearing thereon

were forgeries; or that the signatures were unauthorized.

Therefore, Defendant is deemed to have admitted the loan

documents and acknowledged his obligation with Plaintiff; and with

Defendant’s implied admission, it was not necessary for Plaintiff to

present further evidence to establish the due execution and authenticity of

the loan documents sued upon (Permanent Savings and Loan Bank vs.

Mariano Velarde,).

D. Adoptive admission concept and meaning

A party may, by his words or conduct, voluntarily adopt or ratify

another's statement.

Where it appears that a party clearly and

unambiguously assented to or adopted the statements of another, evidence

of those statements is admissible against him. This is the essence of the

principle of adoptive admission (Republic vs. Kenrick Development

Corporation,).

An adoptive admission is a party's reaction to a statement or action

by another person when it is reasonable to treat the party's reaction as an

admission of something stated or implied by the other person. By adoptive

admission, a third person's statement becomes the admission of the party

embracing or espousing it (Republic vs. Kenrick Development

Corporation, G.R. No. 149576, August 8, 2006).

E. Adoptive admission concept and meaning

Adoptive admission may occur when a party:

(a) expressly agrees to or concurs in an oral statement made

by another;

(b) hears a statement and later on essentially repeats it;

(c) utters an acceptance or builds upon the assertion of

another;

(d) replies by way of rebuttal to some specific points raised

by another but ignores further points which he or she

has heard the other make or

(e) reads and signs a written statement made by another

(Republic vs. Kenrick Development Corporation,).

F. Answer must be signed

An unsigned answer is invalid and produces no legal effect.

Defendant may be declared in default and allow plaintiff to present

evidence ex-parte (Republic vs. Kenrick Development Corporation,).

---0---

COUNTERCLAIM

A. What is a counterclaim?

A counterclaim is any claim which a defending party may have

against an opposing party (Section 6, Rule 6, 1997 Revised Rules of Civil

Procedure). This may be a claim for money or other relief (See Sapugay

vs. CA,).

B. Kinds, Principles and its Limitations

Counterclaim may be compulsory or permissive.

Counterclaims are generally allowed in order to avoid a

multiplicity of suits and to facilitate the disposition of the whole

controversy in a single action, such that the defendant’s demand may be

adjudged by a counterclaim rather than by an independent suit (Lafarge

Cement Philippines, Inc., et al, vs. Continental Cement Corporation, et

al,).

The only limitations to this principle are (1) that the court should

have jurisdiction over the subject matter of the counterclaim, and (2) that

it could acquire jurisdiction over third parties whose presence is essential

for its adjudication suit (Lafarge Cement Philippines, Inc., et al, vs.

Continental Cement Corporation,).

COMPULSORY COUNTERCLAIM

A compulsory counterclaim is one which, being cognizable by the

regular courts of justice, arises out of or is connected with the transaction

or occurrence constituting the subject matter of the opposing party's claim

and does not require for its adjudication the presence of third parties of

whom the court cannot acquire jurisdiction. Such a counterclaim must be

within the jurisdiction of the court both as to the amount and the nature

thereof, except that in an original action before the Regional Trial Court,

the counter-claim may be considered compulsory regardless of the amount

(Section 7, Rule 6, 1997 Revised rules of Civil Procedure).

It is compulsory in the sense that it is within the jurisdiction of the

court. It does not require for its adjudication the presence of third parties

over whom the court cannot acquire jurisdiction, and will be barred in the

future if not set up in the answer to the complaint in the same case. Any

other counterclaim is permissive (Cruz-Agana vs. Judge Santiago-

Lagman,).

A compulsory counterclaim set up in the answer is not an

“initiatory” or similar pleading. The initiatory pleading is the plaintiff’s

complaint. A defendant has no choice but to raise a compulsory

counterclaim the moment the plaintiff files the complaint, otherwise

defendant waives the compulsory counterclaim. In short, the compulsory

counterclaim is a reaction or response, mandatory upon pain of waiver, to

an initiatory pleading which is the complaint (Cruz-Agana vs. Judge

Santiago-Lagman,).

A. Criteria or Tests of Compulsoriness

The following criteria to determine whether a counterclaim is

compulsory or permissive:

(1) Are issues of fact and law raised by the claim and by the

counterclaim largely the same?

(2) Would res judicata bar a subsequent suit on defendant’s claim,

absent the compulsory counterclaim rule?

(3) Will substantially the same evidence support or refute

plaintiff’s claim as well as defendant’s counterclaim?

(4) Is there any logical relation between the claim and the

counterclaim? A positive answer to all four questions would indicate that

the counterclaim is compulsory (NAMARCO vs. Federation of United

Namarco Distributors,).

In short, if (1) the issues of fact or law raised by the claim and the

counterclaim largely the same; (2) res judicata will bar a subsequent suit

on defendant’s claim absent the compulsory counterclaim rule; (3) the

same evidence will substantially support or refute plaintiff’s claim as well

as the defendant’s counterclaim; and (4) there is any logical relation

between the claim and the counterclaim, the claim is compulsory.

A counterclaim to be considered as compulsory it must passed the

so-called “compelling test of compulsoriness”, and that is, the test of

“logical relationship” between the main claim and the counterclaim must

exist. There exists such a relationship when conducting separate trials of

the respective claims of the parties would entail substantial duplication of

time and effort by the parties and the court; when the multiple claims

involve the same factual and legal issues; or when the claims are offshoots

of the same basic controversy between the parties (Quintanilla vs. CA).

B. Situationer

If for example, X moves to intervene as party defendant in a suit

between Plaintiff and Defendant, and in his answer he asks plaintiff for

attorney’s fees, moral and exemplary damages as counterclaim for, it is

humbly opines that his counterclaim is permissive, because, he voluntarily

participated in the case without waiting first to be impeladed as party

defendant. It would have been different had Plaintiff later on amends his

complaint by impleading X as party defendant (indispensable or

necessary) and X files his answer pursuant to summons served upon him,

this time, it is humbly opines that counter claim for attorney’s fees, moral

and exemplary damages pleaded in his answer is compulsory. Therefore, if

the attorney’s fees, moral and exemplary damges is permissive, then he is

obliged to pay docket fees because, payment of correct docket fees is

jurisdictional.

C. Compulsory Counterclaim not set up on time is barred

A compulsory counterclaims not set up shall be barred (Section 2,

Rule 9, 1997 Revised Rules of Civil Procedure). The same should be set

up in the same action, otherwise, they would be barred forever (Lafarge

Cement Philippines, Inc., et al, vs. Continental Cement Corporation,).

If it is not set up shall be barred if not raised on time and the party

in error is precluded from setting it up in a subsequent litigation on the

ground of res judicata, the theory being that what are barred by prior

judgment are not only the matters actually raised and litigated upon, but

also such matters as could have been raised but were not (Metals

Engineering Resources Corporation vs. CA,).

It cannot be made the subject of a separate action but should be

asserted in the same suit involving the same transaction or occurrence

giving rise to it (Metals Engineering Resources Corp. vs. Court of

Appeals). This is because it is an auxiliary to the proceeding in the original

suit and derives its jurisdictional support therefrom, inasmuch as it arises

out of or is necessarily connected with the transaction or occurrence that is

the subject matter of the complaint (Metals Engineering Resources Corp.

vs. Court of Appeals.), and if made the subject of a separate suit, it may be

abated upon a plea of auter action pendant or litis pendentia, and/or

dismissed on the ground of res judicata (Visayan Packing Corporation

vs. Reparations Commission,).

D. Compulsory Counterclaim is not an initiatory pleading

A compulsory counterclaim does not require a certificate of non-

forum shopping because a compulsory counterclaim is not an initiatory

pleading. Lack of certificate of non-forum shopping is immaterial (Cruz-

Agana vs. Judge Santiago-Lagman,).

E. Effect of Dismissal of Plaintiff’s Complaint

The dismissal of the complaint due to the fault of plaintiff does not

necessarily carry with it the dismissal of the counterclaim, compulsory or

otherwise. In fact, the dismissal of the complaint is without prejudice to

the right of defendants to prosecute the counterclaim (Pinga vs. Heirs of

Santiago,).

In short, the defendant’s counterclaims must be disposed of based

on the merit of the counterclaim itself and not on the survival of the main

complaint; if the counterclaim is palpably without merit or suffers

jurisdictional flaws which stand independent of the complaint, the trial

court is not precluded from dismissing it under the amended rules,

provided that the judgment or order dismissing the counterclaim is

premised on those defects; and if the counterclaim is justified,

counterclaim is protected by the rules from peremptory dismissal by

reason of the dismissal of the complaint (Pinga vs. Heirs of Santiago,).

It is worthy to note that what was given emphasis in Pinga is

dismissal of actions under Sections 2 and 3 of Rule 17 of the 1997 Revised

Rules of Civil Procedure. Dismissal of plaintiff’s complaint by reason of

defendant’s motion to dismiss is not touched in Pinga. So, by analogy

application of the ruling in Pinga, if the complaint is dismissed by reason

of defendant’s motion (to dismiss), the defendant’s compulsory

counterclaim survives, as complaint’s dismissal does not result in the

automatic dismissal of the counterclaim.

Pinga case abandoned the ruling in Financial Building

Corporation vs. Forbes Park Association,) that dismissal of the main

action results in the dismissal of the counterclaim already filed. Therefore,

the rationale that “it stands to reason that the filing of a motion to dismiss

the complaint is an implied waiver of the compulsory counterclaim

because the grant of the motion ultimately results in the dismissal of the

counterclaim” in Financial Building Corporation case is no longer

controlling.

It also abandoned the ruling in Metals Engineering Resources

Corp. vs. CA). In that case the complaint was dismissed through

defendant’s motion on the ground of lack of jurisdiction for non-payment

of docket fees which dismissal was affirmed by the High Court holding

that that by reason of said dismissal, it is as if no claim was filed against

the defendant, hence the counterclaim has no leg to stand on. The doctrinal

pronouncement in Metals that “if the court does not have jurisdiction to

entertain the main action of the case and dismisses the same, the

compulsory counterclaim, being ancillary to the principal controversy,

must likewise be dismissed since no jurisdiction remained for any grant of

relief under the counterclaim” is also no longer controlling.

Now, it is no longer tree that dismissal of the complaint necessarily

carries the dismissal of the compulsory counterclaim.

F. Remedy to file by defendant if he has a compulsory

counterclaim against the Plaintiff: Either Answer or Motion to

Dismiss

The filing of a motion to dismiss and the setting up of a

compulsory counterclaim are incompatible remedies (Financial Building

Corporation vs. Forbes Park Association).

In the event that a defending party has a ground for dismissal and a

compulsory counterclaim at the same time, he must choose only one

remedy. If he decides to file a motion to dismiss, he will lose his

compulsory counterclaim. But if he opts to set up his compulsory

counterclaim, he may still plead his ground for dismissal as an affirmative

defense in his answer (Financial Building Corporation vs. Forbes Park

Association, Inc.,).

In short, the defendant may either file a motion to dismiss or file an

answer and asserts therein his motion to dismiss as an affirmative defense.

This of course, goes without saying that in the event that if defendant opts

to file a motion to dismiss, he may still file an answer with compulsory

counterclaim if his motion to dismiss will be denied.

G. Effects of plaintiff’s failure to appear during pre-trial

conference to defendant’s compulsory counterclaim

The defendant’s compulsory counterclaim is not dismissible by

reason of dismissal of the plaintiff’s complaint for plaintiff’s failure to

appear during the pre-trial. According to Spouses Corpuz vs. Citibank,

N.A,), plaintiff’s failure to appear during the pre-trial conference pursuant

to Section 3, Rule 17, 1997 Revised Rules of Civil Procedure amounts to

failure to comply with the Rules or any Order of the Court.

---0---

PERMISSIVE COUNTERCLAIM

A counterclaim is permissive if it does not arise out of or is not

necessarily connected with the subject matter of the opposing party’s

claim (Lopez v. Gloria,). It is essentially an independent claim that may

be filed separately in another case (Lafarge Cement Philippines, Inc., et

al, vs. Continental Cement Corporation,), or is capable of proceeding

independently of the main case (Bungcayao, Sr. vs. Fort Ilocandia

Property Holdings and Development Corporation,).

A. Set-off and Recoupment as grounds for counterclaims

A set-off and recoupment can be a subject of counterclaims (Korea

Exchange Bank vs. Judge Gonzales,) provided that the following must

concur: (1) the same be essentially a genuine action of the defendant

against the plaintiff; (2) the same should have as its object to neutralize,

wholly or partially, that which the plaintiff is trying to obtain; (3) the same

does not have for its object to destroy directly the action of the plaintiff;

and (4) the same ought not to pray for a positive remedy distinct from the

payment of money (Lopez vs. Gloria).

B. Set-off (Compensacion) and recoupment (reconvencion)

distinguished

A set-off (compensacion) is a money demand by the defendant

against the plaintiff arising upon contract and constituting a debt

independent of and unconnected with the cause of actions set forth in the

complaint, and may be used to offset a plaintiff’s claim but not to recover

affirmatively (Lopez vs. Gloria).

A recoupment (reconvencion) differs from a counterclaim

(contrarreclamacion) in that, under a counterclaim, the defendant may

have an affirmative judgment where he is able to prove a demand in

excess of the plaintiff’s demand, whereas in the case of recoupment,

whatever the damages proved by the defendant, they can go only to reduce

or extinguish the claim against him. Recoupment must arise out of the

contract or transaction upon which the plaintiff’s claim is founded.

Recoupment is of French origin and means the “cutting back of the

plaintiff’s claim by the defendant.” It thus implies an admission of the

plaintiff’s claim (Lopez vs. Gloria).

C. Payment of docket fees for permissive counterclaim is

jurisdictional

The counterclaimant is bound to pay the prescribed docket fees

(Sandejas v. Ignacio, Jr.,).

A permissive counterclaim also requires payment of the prescribed

docket fees, because it is not only the filing of the complaint or

appropriate initiatory pleading, but the payment of the prescribed docket

fees that vests the court with jurisdiction over the subject matter of the

action (Tan vs. Planters Products,).

The non-payment of which is a ground for the dismissal of the

permissive counterclaim on ground of lack of jurisdiction over it, and the

court by authority of Section 1, Rule 9 of the 1997 Revised Rules of Civil

Procedure may dismiss it motu proprio or sans motion to dismiss (Tan vs.

Planters Products, Inc,).

Any decision rendered without jurisdiction with respect to

permissive counterclaim is a total nullity and may be struck down at any

time, even on appeal before this Court (Sandejas v. Ignacio, Jr.,).

In Sapugay vs. CA , it was held that payment of docket fees is

specifically applicable to permissive counterclaims only. Compulsory

counterclaim is excluded from its purview.

D. Counterclaimant should be ordered to pay the prescribed

docket fees

If the trial court made a mistake by saying that the counterclaim is

compulsory and on appeal it found out that the counterclaim is permissive,

the counterclaim is not dismissible, the counterclaimant should be given a

reasonable period of time, but in no case beyond the reglementary period,

within which to pay the prescribed docket fees because the

counterclaimant should not be made to suffer due to the mistake of the

trial court (Calibre Traders, Inc. vs. Bayer Philippines,).

E. Permissive Counterclaim requires a certificate of non-forum

shopping

A permissive counterclaim requires a certificate of non-forum

shopping because it is an initiatory pleading. Lack of it is fatal (Cruz-

Agana vs. Judge Santiago-Lagman,), and shall be cause for the dismissal

of the counterclaim. The dismissal is always without prejudice, unless

otherwise provided (See Section 5(2) Rule 7, 1997 Revised Rules of Civil

Procedure).

F. Summons is not required to file answer in counterclaim

Well settled is the rule that the purpose of summons is to enable

the court to acquire jurisdiction over the person of the defendant. But as

regards counterclaim, the Rules of Court does not require that summons

should first be served on the defendant before an answer to counterclaim

must be made. Although a counterclaim is treated as an entirely distinct

and independent action, the defendant in the counterclaim, being the

plaintiff in the original complaint, has already submitted to the jurisdiction

of the court (Francisco Motors Corporation vs. CA,).

G. Rule on default in counterclaim

If the defendant (plaintiff in the complaint) fails within the

reglementary period, that is, ten (10) days from service (Section 4, Rule

11, 1997 Revised Rules of Civil Procedure) within which to file an answer,

he, upon motion of the counterclaimant (or the plaintiff in the

counterclaim) may be declared in default (Section 3, Rule 9, 1997 Rules of

Civil Procedure) (Francisco Motors Corporation vs. CA,).

H. Moral, actual and exemplary damages can either be compulsory

or permissive

Counterclaim for moral, actual and exemplary damages and

attorney’s fees on account of plaintiff’s malicious and unfounded

complaint is compulsory (Tiu Po vs. Bautista,), but in Korea Exchange

Bank vs. Judge Gonzales), a counterclaim for moral and exemplary

damages is considered permissive. Therefore, not all claims for moral and

exemplary damages is always compulsory, because this may also be in the

nature of permissive counterclaim. It can be compulsory or permissive

depending upon the circumstances by which the same is claimed.

---0---

PARTIES’ REMEDIES AFTER

ANSWER IS FILED

The following are the remedies available to the parties after an

answer is filed:

1. File a Motion for Judgment Based on Pleadings;

2. File a Motion for Summary Judgment; or

3. (Defendant) to file Motion for Preliminary Hearing on his

affirmative defense asserted in the answer (under Section 6, Rule

16, 1997 Revised Rules of Civil Procedure).

I. FILE A MOTION FOR JUDGMENT BASED ON

PLEADINGS

A. The rule

Where an answer fails to tender an issue, or otherwise admits the

material allegations of the adverse party's pleading, the court may, on

motion of that party, direct judgment on such pleading. xxx xxx xxx

(Section 1, Rule 34, 1997 Revised Rules of Civil Procedure).

B. Concept

Judgment on the pleadings is improper when the answer to the complaint

tenders several issues. It is proper when the answer admits all the material

averments of the complaint. But where several issues are properly

tendered by the answer, a trial on the merits must be resorted to in order to

afford each party his day in court (Municipality of Tiwi, vs. Betito, ).

Where a motion for judgment on the pleadings is filed, the

essential question is whether there are issues generated by the pleadings.

In a proper case for judgment on the pleadings, there is no ostensible issue

at all because of the failure of the defending party’s answer to raise an

issue. The answer would fail to tender an issue, of course, if it does not

deny the material allegations in the complaint or admits said material

allegations of the adverse party’s pleadings by confessing the truthfulness

thereof and/or omitting to deal with them at all (Tan v. De la Vega,).

When what is left are not genuinely issues requiring trial but

questions concerning the proper interpretation of the provisions of some

written contract attached to the pleadings, judgment on the pleadings is

proper (Philippine National Bank vs. Utility Assurance & Surety Co.,

Inc.,).

A motion for judgment on the pleadings admits the truth of all the

material and relevant allegations of the opposing party and the judgment must

rest on those allegations taken together with such other allegations as are

admitted in the pleadings. It is proper when an answer fails to tender an issue, or

otherwise admits the material allegations of the adverse party’s pleading.

However, when it appears that not all the material allegations of the complaint

were admitted in the answer for some of them were either denied or disputed,

and the defendant has set up certain special defenses which, if proven, would

have the effect of nullifying plaintiff’s main cause of action, judgment on the

pleadings cannot be rendered (Municipality of Tiwi, vs. Betito,).

The trial court has the discretion to grant a motion for judgment on

the pleadings filed by a party if there is no controverted matter in the case

after the answer is filed. A judgment on the pleadings is a judgment on the

facts as pleaded, and is based exclusively upon the allegations appearing

in the pleadings of the parties and the accompanying annexes (Sunbanun

vs. Go,).

If it is the defendant who moves for a judgment on the pleadings

without offering proof as to the truth of her own allegations and without

giving plaintiff the opportunity to introduce evidence, defendant is deemed

to have admitted the material and relevant averments of the complaint, and

to rest her motion for judgment based on the pleadings of the parties

(Rodriguez vs. Llorente,).

If the plaintiff asks the court for judgment on the pleadings and

defendant has consented to it, and if judgment is not favorable to the

defendant, he can no longer question the validity of the judgment. In

Tropical Homes, Inc. vs. CA, , the High Court said:

“As to the amount of damages awarded as a

consequence of this violation of plaintiff’s rights, the lower

court based its award from the allegations and prayer

contained in the complaint. The defendant, however,

questions this award for the reason that, according to the

defendant, the plaintiff, in moving for judgment on the

pleadings, did not offer proof as to the truth of his own

allegations with respect to the damages claimed by him,

and gave no opportunity for the appellant to introduce

evidence to refute his claims. We find this objection

without merit. It appears that when the plaintiff moved to

have the case decided on the pleadings, the defendant

interposed no objection and has practically assented

thereto. The defendant, therefore, is deemed to have

admitted the allegations of fact of the complaint, so that

there was no necessity for plaintiff to submit evidence of

his claim.”

If the court can resolve the issues presented by the complaint and

the answer can be resolved within the four corners of said pleadings

without need to conduct further hearings, and what remains to be done is

the proper interpretation of the contracts or documents attached to the

pleadings, then judgment on the pleadings is proper (Pacific Rehouse

Corporation vs. EIB Securities, Inc.,).

In an action for sum of money and defendants have admitted that

they obtained the loan; have admitted the due execution of the loan

documents and their receipt of the demand letter made by the plaintiff, and

only issue is whether the obligation is matured or not, the High Court held

that the case can be decided summarily, because when plaintiff made its

demand, the obligation matured. The matter proferred as a defense could

be resolved judiciously by plain resort to the stipulations in the promissory

note (Wood Technology vs. Equitable Bank,).

II. FILE A MOTION FOR SUMMARY JUDGMENT

A. The rules

A party seeking to recover upon a claim, counterclaim, or cross-

claim or to obtain a declaratory relief may, at any time after the pleading

in answer thereto has been served, move with supporting affidavits,

depositions or admissions for a summary judgment in his favor upon all or

any part thereof (Section 1, Rule 35, 1997 Revised Rules of Civil

Procedure).

A party against whom a claim, counterclaim, or cross-claim is

asserted or a declaratory relief is sought may, at any time, move with

supporting affidavits, depositions or admissions for a summary judgment

in his favor as to all or any part thereof (Section 2, Rule 35, 1997 Revised

Rules of Civil Procedure).

The motion shall be served at least ten (10) days before the time

specified for the hearing. The adverse party may serve opposing affidavits,

depositions, or admissions at least three (3) days before the hearing. After

the hearing, the judgment sought shall be rendered forthwith if the

pleadings, supporting affidavits, depositions, and admissions on file, show

that, except as to the amount of damages, there is no genuine issue as to

any material fact and that the moving party is entitled to a judgment as a

matter of law (Section 3, Rule 35, 1997 Revised Rules of Civil

Procedure).

If on motion under this Rule, judgment is not rendered upon the

whole case or for all the reliefs sought and a trial is necessary, the court at

the hearing of the motion, by examining the pleadings and the evidence

before it and by interrogating counsel shall ascertain what material facts

exist without substantial controversy and what are actually and in good

faith controverted. It shall thereupon make an order specifying the facts

that appear without substantial controversy, including the extent to which

the amount of damages or other relief is not in controversy, and directing

such further proceedings in the action as are just. The facts so specified

shall be deemed established, and the trial shall be conducted on the

controverted facts accordingly (Section 4, Rule 35, 1997 Revised Rules of

Civil Procedure).

Supporting and opposing affidavits shall be made on personal

knowledge, shall set forth such facts as would be admissible in evidence,

and shall show affirmatively that the affiant is competent to testify to the

matters stated therein. Certified true copies of all papers or parts thereof

referred to in the affidavit shall be attached thereto or served therewith (Section 5, Rule 35, 1997 Revised Rules of Civil Procedure).

Should it appear to its satisfaction at any time that any of the

affidavits presented pursuant to this Rule are presented in bad faith, or

solely for the purpose of delay, the court shall forthwith order the

offending party or counsel to pay to the other party the amount of the

reasonable expenses which the filing of the affidavits caused him to incur

including attorney's fees, it may, after hearing further adjudge the

offending party or counsel guilty of contempt (Section 6, Rule 35, 1997

Revised Rules of Civil Procedure).

B. Concept and purpose

A summary judgment is granted to settle expeditiously a case if, on

motion of either party, there appears from the pleadings, depositions,

admissions, and affidavits that no important issues of fact are involved,

except the amount of damages. In such event, the moving party is entitled

to a judgment as a matter of law. Trial courts have limited authority to

render summary judgments and may do so only when there is clearly no

genuine issue as to any material fact ().

In Calubaquib vs. Republic, , the High Court explained the concept

of summary judgment, thus:

“Summary judgments are proper when, upon motion of

the plaintiff or the defendant, the court finds that the answer filed

by the defendant does not tender a genuine issue as to any

material fact and that one party is entitled to a judgment as a

matter of law. A deeper understanding of summary judgments is

found in Viajar v. Estenzo:

Relief by summary judgment is intended to

expedite or promptly dispose of cases where the

facts appear undisputed and certain from the

pleadings, depositions, admissions and affidavits.

But if there be a doubt as to such facts and there be

an issue or issues of fact joined by the parties,

neither one of them can pray for a judgment. Where

the facts pleaded by the parties are disputed or

contested, proceedings for a judgment cannot take

the place of a trial.

An examination of the Rules will readily show

that a judgment is by no means a hasty one. It

assumes a scrutiny of facts in a summary hearing

after the filing of a motion for judgment by one party

supported by affidavits, depositions, admissions, or

other documents, with notice upon the adverse party

who may file an opposition to the motion supported

also by affidavits, depositions, or other documents

x x x. In spite of its expediting character, relief by

judgment can only be allowed after compliance with

the minimum requirement of vigilance by the court

in a summary hearing considering that this remedy is

in derogation of a party's right to a plenary trial of his

case. At any rate, a party who moves for judgment

has the burden of demonstrating clearly the absence

of any genuine issue of fact, or that the issue posed

in the complaint is so patently unsubstantial as not to

constitute a genuine issue for trial, and any doubt as

to the existence of such an issue is resolved against

the movant.

“A summary judgment is permitted only if there is no

genuine issue as to any material fact and [the] moving party is

entitled to a judgment as a matter of law.” The test of the

propriety of rendering summary judgments is the existence of a

genuine issue of fact, “as distinguished from a sham, fictitious,

contrived or false claim.” “[A] factual issue raised by a party is

considered as sham when by its nature it is evident that it cannot

be proven or it is such that the party tendering the same has

neither any sincere intention nor adequate evidence to prove it.

This usually happens in denials made by defendants merely for

the sake of having an issue and thereby gaining delay, taking

advantage of the fact that their answers are not under oath

anyway.”

In determining the genuineness of the issues, and hence the

propriety of rendering a summary judgment, the court is obliged

to carefully study and appraise, not the tenor or contents of the

pleadings, but the facts alleged under oath by the parties and/or

their witnesses in the affidavits that they submitted with the

motion and the corresponding opposition. Thus, it is held that,

even if the pleadings on their face appear to raise issues, a

summary judgment is proper so long as “the affidavits,

depositions, and admissions presented by the moving party

show that such issues are not genuine.”

In Nocom vs. Camerino, , a summary judgment is explained as a

procedural device resorted to in order to avoid long drawn out litigations

and useless delays. When the pleadings on file show that there are no

genuine issues of fact to be tried, the Rules allow a party to obtain

immediate relief by way of summary judgment, that is, when the facts are

not in dispute, the court is allowed to decide the case summarily by

applying the law to the material facts. Conversely, where the pleadings

tender a genuine issue, summary judgment is not proper. A “genuine

issue” is such issue of fact which requires the presentation of evidence as

distinguished from a sham, fictitious, contrived or false claim. Section 3

of the said rule provides two (2) requisites for summary judgment to be

proper: (1) there must be no genuine issue as to any material fact, except

for the amount of damages; and (2) the party presenting the motion for

summary judgment must be entitled to a judgment as a matter of law. A

summary judgment is permitted only if there is no genuine issue as to any

material fact and a moving party is entitled to a judgment as a matter of

law. A summary judgment is proper if, while the pleadings on their face

appear to raise issues, the affidavits, depositions, and admissions presented

by the moving party show that such issues are not genuine.

The law itself determines when a summary judgment is proper.

Under the rules, summary judgment is appropriate when there are no

genuine issues of fact which call for the presentation of evidence in a full-

blown trial. Even if on their face the pleadings appear to raise issues,

when the affidavits, depositions and admissions show that such issues are

not genuine, then summary judgment as prescribed by the rules must

ensue as a matter of law. What is crucial for determination, therefore, is

the presence or absence of a genuine issue as to any material fact (Vadel

Realty and Development Corporation vs. Spouses Soriano,).

The “the amount of damages” is excluded in the coverage,

because in order to prove the issue as to amount of damages, presentation

of evidence is necessary.

In Bungcayao, Sr. vs. Fort Ilocandia Property Holdings, the trial

court in an action for declaration of nullity of contract noted the stipulated

issues and admissions been made by both parties, made to confirm the

agreement of the parties to cancel the Deed of Assignment, Release,

Waiver and Quitclaim and the return of the money to the respondent, and

summarily decided the case dismissing the plaintiff’s claim for damages

for want of legal basis, and directed to immediately vacate the premises

subject of the case.

C. Whether a case is appropriate for summary judgment,

determining guide

In other words, in a motion for summary judgment, the crucial

question is: are the issues raised in the pleadings genuine, sham or

fictitious, as shown by affidavits, depositions or admissions accompanying

the motion? (Manufacturers Hanover Trust Co. vs. Guerrero, ).

So, if there is genuine issue to be resolved and necessitates the

presentation of evidence to resolve the issue, then summary judgment is

not proper.

A. Genuine issue, concept/definition

A “genuine issue” is an issue of fact which requires the

presentation of evidence as distinguished from a sham, fictitious,

contrived or false claim. When the facts as pleaded appear uncontested or

undisputed, then there is no real or genuine issue or question as to the

facts, and summary judgment is called for. The party who moves for

summary judgment has the burden of demonstrating clearly the absence of

any genuine issue of fact, or that the issue posed in the complaint is

patently unsubstantial so as not to constitute a genuine issue for trial. Trial

courts have limited authority to render summary judgments and may do so

only when there is clearly no genuine issue as to any material fact. When

the facts as pleaded by the parties are disputed or contested, proceedings

for summary judgment cannot take the place of trial (Evadel Realty and

Development Corporation v. Soriano,).

In Cotabato Timberland Co., Inc. vs. C. Alcantara and Sons,

Inc., , it was held that summary judgment is not proper because there is a

necessity of trial on the merits and presentation of evidence for the trial

court to properly determine which among the parties must shoulder the

loss. In that case, the special and affirmative defenses raised by private

respondents invoking, inter alia, the alleged fault and negligence of

petitioner as the proximate cause of the loss of the subject logs indubitably

tender a genuine and factual issue as regards the proximate cause of the

loss. A full-blown trial on the merits and presentation of additional

evidence is called for.

When variance in the allegations of the parties in the pleadings is evident,

and the facts as pleaded by the parties are disputed or contested, the case on

appeal should be remanded to the trial court for further proceedings and proper

disposition according to the rudiments of a regular trial on the merits and not

through an abbreviated termination of the case by summary judgment, because

proceedings for summary judgment cannot take the place of trial (Atty. Ferrer

vs. Spouses Diaz,).

B. Sham issue or claim, concept

A factual issue raised by a party is considered as sham when by its nature

it is evident that it cannot be proven or it is such that the party tendering the same

has neither any sincere intention nor adequate evidence to prove it. This

usually happens in denials made by defendants merely for the sake of having an

issue and thereby gaining delay, taking advantage of the fact that their answers

are not under oath anyway (Calubaquib vs. Republic, ).

F. Filing of motion and conduct of hearing necessary, non-compliance

warrants setting aside of the summary judgment

The filing of a motion and the conduct of a hearing on the motion are

therefore important because these enable the court to determine if the parties’

pleadings, affidavits and exhibits in support of, or against, the motion are

sufficient to overcome the opposing papers and adequately justify the finding

that, as a matter of law, the claim is clearly meritorious or there is no defense to

the action. The non-observance of the procedural requirements of filing a motion

and conducting a hearing on the said motion warrants the setting aside of the

summary judgment (Eland Philippines, Inc. vs. Garcia, ).

In Calubaquib vs. Republic, , it was held that when the trial court

proceeded to render summary judgment with neither of the parties filing a motion

therefor it disragarded and ignored the guidelines and safeguards for the rendition

of a summary judgment thereby violating the defendant’s due process right to a

trial where they can present their evidence and prove their defense. In this case, it

was the trial court that directed the plaintiff to file a motion for summary

judgment which was opposed by the defendant.

Also in Calubaquib vs. Republic, , it was held that it was improper for

the trial court to have persisted in rendering summary judgment; to make a

baseless assumption is premature and unfair.

III. DEFENDANT TO FILE MOTION FOR PRELIMINARY

HEARING ON HIS AFFIRMATIVE DEFENSE ASSERTED

IN THE ANSWER

A. The rule

Preliminary hearing is not mandatory, but subject to the discretion

of the trial court.1

If no motion to dismiss has been filed, any of the grounds for

dismissal provided for in this Rule may be pleaded as an affirmative

defense in the answer and, in the discretion of the court, a preliminary

hearing may be had thereon as if a motion to dismiss had been filed

(Section 6, Rule 16, 1997 Revised Rules of Civil Procedure).

The allowance for a preliminary hearing, while left in the

discretion of the court, is authorized only if no motion to dismiss has been

filed but any of the grounds for a motion to dismiss had been pleaded as

an affirmative defense in the answer (Rasdas vs. Estenor,).

Section 6, Rule 16 of the 1997 Revised Rules of Civil Procedure is

explicit in stating that the defendant may reiterate any of the grounds for

dismissal provided under Rule 16 of the Rules of Court as affirmative

defenses but that a preliminary hearing may no longer be had thereon if a

motion to dismiss had already been filed (Abrajano vs. Heirs of Salas,

Jr.,).

1Municipality of Biñan, Laguna vs. CA, 219 SCRA 69, February 17, 1993 and Regalado,

Remedial Law Compendium, Vol. 1, pp. 163-164 (1993) cited in California and

Hawaiian Sugar Company vs. Pioneer Insurance and Surety Corporation, G.R. No.

139273, November 28, 2000.

B. Preliminary hearing may no longer be had thereon if a motion

to dismiss had already been filed, exception

The rule does not contemplate a situation where there are several

defendants but only one filed a motion to dismiss. Other defendants who

did not file a motion to dismiss may still ask and may be allowed for a

preliminary hearing on their own affirmative defenses, otherwise it will be

prejudicial to other defendants who did not file a motion to dismiss

(Abrajano vs. Heirs of Salas,).

Not only that, it will violate the due process and equal protection

clauses enshrined in our Constitution.

B. Preliminary hearing allowed even an answer is filed after a

motion to dismiss was filed and deferred its resolution

When a motion to dismiss is filed and its resolution is deferred,

and thereafter directed the defendant to file answer, and if answer is filed

defendant is still entitled to a preliminary hearing on his affirmative

defenses asserted in the Answer (California and Hawaiian Sugar

Company vs. Pioneer Insurance and Surety Corporation,).

In California and Hawaiian Sugar Company vs. Pioneer

Insurance and Surety Corporation, G it was held that defendant is still

entitled to preliminary hearing despite of filing a motion to dismiss,

because the trial court deferred the resolution of the motion to dismiss and

required the defendant to file answer instead.

The High Court said in California:

“True, Section 6, Rule 16 of the 1997 Rules,

specifically provides that a preliminary hearing on the

affirmative defenses may be allowed only when no motion

to dismiss has been filed. Section 6, however, must be

viewed in the light of Section 3 of the same Rule, which

requires courts to resolve a motion to dismiss and prohibits

them from deferring its resolution on the ground of

indubitability. Clearly then, Section 6 disallows a

preliminary hearing of affirmative defenses once a motion

to dismiss has been filed because such defense should have

already been resolved. In the present case, however, the

trial court did not categorically resolve petitioners’ Motion

to Dismiss, but merely deferred resolution thereof.”

The case of Rasdas vs. Estenor is quite peculiar, because the High

Court affirmed the order of dismissal of the case on ground of res judicata

pleaded in the answer after a preliminary hearing was conducted. The

rationale advanced by the High Court is that disallowing the dismissal

would violate the primordial objective of procedural law to secure a just,

speedy and inexpensive disposition of every action and proceeding. The

High Court said in Rasdas:

“We observe at the onset that it does appear that the

RTC’s act of staging preliminary hearing on the affirmative

defense of lack of jurisdiction and res judicata is not in

regular order. Under Section 6, Rule 16 of the 1997 Rules

of Civil Procedure, the allowance for a preliminary hearing,

while left in the discretion of the court, is authorized only if

no motion to dismiss has been filed but any of the grounds

for a motion to dismiss had been pleaded as an affirmative

defense in the answer. In this case, respondents had filed a

motion to dismiss on the ground of res judicata, but the

same was denied. They thus filed an answer alleging res

judicata as a special affirmative defense, but later presented

a Motion for Preliminary Hearing which was granted,

leading to the dismissal of the case.

The general rule must be reiterated that the

preliminary hearing contemplated under Section 6, Rule 16

applies only if no motion to dismiss has been filed. This is

expressly provided under the rule, which relevantly states

“[i]f no motion to dismiss has been filed, any of the

grounds for dismissal provided for in [Rule 16] may be

pleaded as an affirmative defense in the answer and, in the

discretion of the court, a preliminary hearing may be had

thereon as if a motion to dismiss had been filed.” An

exception was carved out in California and Hawaiian Sugar

Company v. Pioneer Insurance, wherein the Court noted

that while Section 6 disallowed a preliminary hearing of

affirmative defenses once a motion to dismiss has been

filed, such hearing could nonetheless be had if the trial

court had not categorically resolved the motion to dismiss.

Such circumstance does not obtain in this case, since the

trial court had already categorically denied the motion to

dismiss prior to the filing of the answer and the motion for

preliminary hearing.

We observe in this case that the judge who had earlier

denied the motion to dismiss, Hon. Teodulo E. Mirasol,

was different from the judge who later authorized the

preliminary hearing, Hon. Isaac R. de Alban, a

circumstance that bears some light on why the RTC

eventually changed its mind on the motion to dismiss. Still,

this fact does not sanction the staging of a preliminary

hearing on affirmative defenses after the denial of the

motion to dismiss. If a judge disagrees with his/her

predecessor’s previous ruling denying a motion to dismiss,

the proper recourse is not to conduct a preliminary hearing

on affirmative defenses, but to utilize the contested ground

as part of the basis of the decision on the merits.

On the part of the movant whose motion to dismiss

had already been filed and denied, the proper remedy is to

file a motion for reconsideration of the denial of the

motion. If such motion for reconsideration is denied, the

ground for the dismissal of the complaint may still be

litigated at the trial on the merits.

Clearly, the denial of a motion to dismiss does not

preclude any future reliance on the grounds relied

thereupon. However, nothing in the rules expressly

authorizes a preliminary hearing of affirmative defenses

once a motion to dismiss has been filed and denied. Thus,

the strict application of Section 6, Rule 16 in this case

should cause us to rule that the RTC erred in conducting

the preliminary hearing.

However, there is an exceptional justification for us to

overlook this procedural error and nonetheless affirm the

dismissal of the complaint. The complaint in question is so

evidently barred by res judicata, it would violate the

primordial objective of procedural law to secure a just,

speedy and inexpensive disposition of every action and

proceeding should the Court allow this prohibited

complaint from festering in our judicial system. Indeed, the

rule sanctioning the liberal construction of procedural rules

is tailor-made for a situation such as this, when a by-the-

numbers application of the rule would lead to absurdity,

such as the continued litigation of an obviously barred

complaint.”

C. Though discretionary, denial of preliminary hearing sometimes

constitutes grave abuse of discretion

Also in California and Hawaiian Sugar Company vs. Pioneer

Insurance and Surety Corporation, , it was held that the trial court

gravely abused its discretion when it denied the motion for preliminary

hearing, thus:

“The more crucial question that we must settle here is

whether the trial court committed grave abuse of discretion

when it denied petitioners’ Motion for a Preliminary

Hearing on their affirmative defense of lack of cause of

action. Undeniably, a preliminary hearing is not mandatory,

but subject to the discretion of the trial court. In the light of

the circumstances in this case, though, we find that the

lower court committed grave abuse of discretion in refusing

to grant the Motion.

We note that the trial court deferred the resolution of

petitioners’ Motion to Dismiss because of a single issue. It

was apparently unsure whether the charter party that the

bill of lading referred to was indeed the Baltimore Berth

Grain Charter Party submitted by petitioners.

Considering that there was only one question, which

may even be deemed to be the very touchstone of the whole

case, the trial court had no cogent reason to deny the

Motion for Preliminary Hearing. Indeed, it committed

grave abuse of discretion when it denied a preliminary

hearing on a simple issue of fact that could have possibly

settled the entire case. Verily, where a preliminary hearing

appears to suffice, there is no reason to go on to trial. One

reason why dockets of trial courts are clogged is the

unreasonable refusal to use a process or procedure, like a

motion to dismiss, which is designed to abbreviate the

resolution of a case.”

---0---

RULE ON DEFAULT

There are times that a defendant after service of summons fails to

file answer to the complaint. This maybe because he does not know what

to do, or maybe because he is not really interested in the suit, he will

deliberately disregards the complaint by his inaction because he opt not to

be bothered by it.

If this happens, the plaintiff is on the advantageous position

because if the case is governed by the Rules on Summary Procedure

judgment can be rendered as may be warranted by the facts alleged in the

complaint under Section 6 of the 1991 Revised Rules on Summary

Procedure; and if the case is governed by the regular the defendant can be

declared in default under Section 3, Rule 9 of the 1997 Revised Rules of

Civil Procedure. In either, his case can be expeditiously disposed of.

A. Cases governed by the Revised Rules on Summary

Procedure;remedy when defendant did not file his answer

Section 6 of the 1991 Revised Rules on Summary Procedure

provides:

“Sec. 6. Effect of failure to answer. – Should the

defendant fail to answer the complaint within the period

above provided, the court, motu proprio, or on motion of the

plaintiff, shall render judgment as may be warranted by the

facts alleged in the complaint and limited to what is prayed

for therein: xxx xxx xxx.”

B. Basic principles and guidelines; interpretation of the rules, strict

rather than liberal; exception

The use of the word “shall” in the Rule on Summary Procedure

underscores mandatory character, not directory. Giving the provisions a

directory application would subvert the nature of the Rule on Summary

Procedure and defeat its objective of expediting the adjudication of suits

(Gachon vs. Devera,).

Liberal interpretation and application of the rules applies only in

proper cases and under justifiable causes and circumstances (Gachon vs.

Devera, Jr.,).

Filing an answer beyond the prescribed period is a ground for the

court to render judgment under Section 6 of the 1991 Revised Rules on

Summary Procedure, except when the late filing for justifiable causes and

circumstances. This is where the liberal interpretation and application of

Rules applies (Gachon vs. Devera, Jr., ().

The case of Leynes vs. CA is somehow peculiar. Defendant’s last

day to file answer fell on Saturday. When his answer was filed the

following working day, a Monday, the trial court granted plaintiff’s

motion to render judgment pursuant to Section 6 of the 1991 Revised

Rules on Summary Procedure holding that on that Saturday there were

employees present in court as required by A.M. Circular No. 2-99

primarily to act on petitions for bail and other urgent matters. The High

Court ordered that defendant’s answer be admitted.

C. Summary Judgment or Judgment on the Pleading not proper

Motions for Summary Judgment or Judgment on the Pleading are

not proper. While it is true that motion for motions for summary judgment

or judgment on the pleading are not prohibited (Section 16, 1991 Revised

Rules on Summary Procedure). The reason is obvious. For summary

judgment to apply there must exists a genuine issue (Narra Integrated

Corporation vs. CA,). Section 6 of the 1991 Revised Rules on Summary

Section applies because defendant’s inaction to the complaint. Same is

true with motion for judgment on the pleading. It presupposes that an

answer is filed, because a judgment on the pleadings is proper when an

answer fails to tender an issue, or otherwise admits the material allegations

of the adverse party’s pleading (Municipality of Tiwi vs. Betito,).

D. Cases governed by regular procedure, remedy when defendant

did not file his answer

A declaration or order of default is issued as a punishment for

unnecessary delay in joining issues (Vlason Enterprises Corporation vs.

CA,).

Section 3, Rule 9 of the 1997 Revised Rules of Civil Procedure

provides:

Section 3. Default; declaration of. — If the defending

party fails to answer within the time allowed therefor, the

court shall, upon motion of the claiming party with notice

to the defending party, and proof of such failure, declare

the defending party in default. Thereupon, the court shall

proceed to render judgment granting the claimant such

relief as his pleading may warrant, unless the court in its

discretion requires the claimant to submit evidence. Such

reception of evidence may be delegated to the clerk of

court.

From the foregoing procedure, if defendant did not file an answer

the trial court cannot motu proprio declare him in default. There must be a

motion to that effect.

The case of Spouses delos Santos vs. Judge Carpio) enumerated

the three requirements to which claiming party must be complied with

before the court may declare the defending party in default, to wit: (1) the

claiming party must file a motion asking the court to declare the defending

party in default; (2) the defending party must be notified of the motion to

declare him in default; (3) the claiming party must prove that the

defending party has failed to answer within the period provided by the

Rule.

E. The claiming party must file a motion asking the court to

declare the defending party in default; the defending party

must be notified of the motion to declare him in default

The claiming party must file a motion asking the court to declare

the defending party in default. In short, there must be a motion. The

plaintiff must file a motion to that effect. And considering that the trial

court cannot resolve the motion without asking the defendant of his

comment and/or opposition, the same is considered as motion of litigious

character. And therefore, the same must comply with the rules on motions,

particularly the requisite notice, service and hearing (See Sections 4, 5 and

6, Rule 15 of the 1997 Revised Rules of Civil Procedure; see also

discussions on Rules on Motions).

The case of Spouse delos Santos vs. Judge Carpio ) clarified that

present rule expressly requires that the motion of the claiming party

should be with notice to the defending party. The purpose of a notice of a

motion is to avoid surprises on the opposite party and to give him time to

study and meet the arguments. The notice of a motion is required when the

party has the right to resist the relief sought by the motion and principles

of natural justice demand that his right be not affected without an

opportunity to be heard (Spouse delos Santos vs. Judge Carpio,).

Therefore, as the present rule on default requires the filing of a motion and

notice of such motion to the defending party, it is not enough that the

defendant failed to answer the complaint within the reglementary period to

be a sufficient ground for declaration in default. The motion must also be

heard (Spouse delos Santos vs. Judge Carpio, ).

F. The claiming party must prove that the defending party has

failed to answer within the period provided by the Rule

To prove this element, the plaintiff must prove that the fifteen day

period, or the period fixed by the court for the defendant to file responsive

pleading has expired or lapsed. This can be done by giving emphasis to the

Sheriff’s Return of Summons as proof that the fifteen day period, or the

period fixed by the court for the defendant to file responsive pleading has

expired or lapsed.

G. Effect to defendant if declared in default

A party in default is entitled to notice of subsequent proceedings

but shall not take part in the trial (Section 3 (a), Rule 9, 1997 Revised

Rules of Civil Procedure). He loses his standing in court and his right to

adduce evidence and to present his defense (Rural Bank of Sta. Catalina

vs. Land Bank of the Philippines,); and if pleading is filed, he should

expect the trial court to act upon their pleadings ((Vlason Enterprises

Corporation vs. CA,).

H. Plaintiff not automatically entitled to the relief prayed for when

defendant is declared in default;

I. A declaration of default not an admission of the truth or the

validity of the plaintiff’s claims; and

J. The trial court has no authority to order plaintiff to present

evidence ex parte without declaring first the defendant in

default

Declaration of default is somehow like a foul shot in a basketball

game wherein the free-thrower is allowed to throw his shot free to garn a

point with no player allowed to defend the shot.

Like a free-throw in a basketball game, the plaintiff in default

proceedings is not automatically entitled to the relief prayed for in the

complaint.

Though plaintiff is allowed to present evidence ex-parte, he is

still duty bound to present sufficient evidence to establish his cause of

action because failure to do so will result in the denial of his claim or

dismissal of his action. The plaintiff is duty bound to establish by

competent evidence all the material allegations of his complaint upon

which he bases his prayer for relief (Lopez vs. Mendezona, ;Luxuria

Homes and/or Posadas vs. CA,), because a judgment by default against a

defendant does not imply a waiver of rights except that of being heard and

of presenting evidence in his favor (De los Santos v. De la Cruz, cited in

Luxuria Homes and/or Posadas vs. CA,). Declaration of default does not

imply admission by the defendant of the facts and causes of action of the

plaintiff, because the codal section requires the latter to adduce his

evidence in support of his allegations as an indispensable condition before

final judgment could be given in his favor. Nor could it be interpreted as

an admission by the defendant that the plaintiff’s causes of action finds

support in the law or that the latter is entitled to the relief prayed for

(Luxuria Homes and/or Posadas vs. CA,).

A declaration of default is not an admission of the truth or the

validity of the plaintiff’s claims (Macondray & Co. vs. Eustaquio,).

Although defendant may be declared in default, the plaintiff is still

required to substantiate his allegations in the complaint because the

judgment of default against defendant who has not appeared or filed his

answer does not imply a waiver of all his rights, except his right to be

heard and to present evidence in his favor; the defendant’s failure to

answer does not imply his admission of the facts and the causes of action

of the plaintiff, because plaintiff is still required to adduce evidence to

support his allegation, thus the defaulting defendant is still protected by

law (Vlason Enterprises Corporation vs. CA,).

Not only that, the only evidence that may be allowed by the trial

court are those evidence that will tend to support the relief sought and

specified in the complaint (Vlason Enterprises Corporation vs. CA), and

no award greater than or different in kind from that specified in the

complaint can be granted (Vlason Enterprises Corporation vs. CA,), and

if a relief other than that specifically prayed for in the complaint is

awarded in a judgment by default, such an award is null and void because

the court is bereft of jurisdiction to grant such an award (Policarpio vs.

RTC of Quezon City,).

Vlason Enterprises Corporation vs. CA (made to emphasize that

without a declaration that petitioner is in default as required in Section 1,

Rule 18 of the Rules of Court, the trial court has no authority to order the

presentation of evidence ex parte against petitioner to render judgment

against it by default.

Once the defendant is declared in default the plaintiff is not

automatically entitled to the relief prayed for. The court can grant the

favorable relief sought only after it has ascertained that the evidence

offered and the facts proven by the presenting party warrant the grant of

the same (Pascua vs. Florendo cited in Luxuria Homes and/or Posadas

vs. CA,), because it would be meaningless to require presentation of

evidence if everytime the other party is declared in default, a decision

would automatically be rendered in favor of the non-defaulting party and

exactly according to the tenor of his prayer (Luxuria Homes and/or

Posadas vs.).

A defaulted defendant is not actually thrown out of court. The

rules see to it that any judgment against him must be in accordance with

law. The evidence to support the plaintiff’s cause is, of course, presented

in his absence, but the court is not supposed to admit that which is

basically incompetent. Although the defendant would not be in a position

to object, elementary justice requires that only legal evidence should be

considered against him. If the evidence presented should not be sufficient

to justify a judgment for the plaintiff, the complaint must be dismissed.

And if an unfavorable judgment should be justifiable, it cannot exceed the

amount or be different in kind from what is prayed for in the complaint

(Luxuria Homes and/or Posadas vs. CA,).

K. Plaintiff must prove his cause by preponderance of evidence

In civil cases, the party having the burden of proof must establish

his case by a preponderance of evidence. In determining where the

preponderance or superior weight of evidence on the issues involved lies,

the court may consider all the facts and circumstances of the case, the

witnesses’ manner of testifying, their intelligence, their means and

opportunity of knowing the facts to which they are testifying, the nature of

the facts to which they testify, the probability or improbability of their

testimony, their interest or want of interest, and also their personal

credibility so far as the same may legitimately appear upon the trial. The

court may also consider the number of witnesses, though the

preponderance is not necessarily with the greater number (Section 1, Rule

133, Rules of Court).

Plaintiff is not excused from not establishing his claims alleged in

the complaint by the required quantum of proof under Section 1 of Rule

133; any advantage they he have gained from the ex parte presentation of

evidence does not lower the degree of proof required (Gajudo, et al., vs.

Traders Royal Bank,).

L. Section 3 of Rule 9 and Section 1 of Rule 133 of the Rules of

Court are not incompatible with each other

In one case the quantum of evidence for judgments flowing from a

default order under Section 3 of Rule 9 was raised as it is not the same as

that provided for in Section 1 of Rule 133. This issue had already been

clarified by the High Court in Gajudo, et al., vs. Traders Royal Bank

holding that Section 3 of Rule 9 and Section 1 of Rule 133 of the Rules of

Court are not incompatible with each other.

The High Court said:

“For ease of discussion, these two rules will be

reproduced below, starting with Section 3 of Rule 9 of the

Rules of Court:

“Sec. 3. Default; declaration of. – If the

defending party fails to answer within the time

allowed therefor, the court shall, upon motion of

the claiming party with notice to the defending

party, and proof of such failure, declare the

defending party in default. Thereupon, the court

shall proceed to render judgment granting the

claimant such relief as his pleading may

warrant, unless the court in its discretion

requires the claimant to submit evidence. Such

reception of evidence may be delegated to the

clerk of court.

“(a) Effect of order of default. – A party

in default shall be entitled to notice of

subsequent proceedings but not to take part in

the trial.

“(b) Relief from order of default. – A party

declared in default may at any time after notice

thereof and before judgment file a motion under

oath to set aside the order of default upon proper

showing that his failure to answer was due to

fraud, accident, mistake or excusable negligence

and that he has a meritorious defense. In such

case, the order of default may be set aside on

such terms and conditions as the judge may

impose in the interest of justice.

“(c) Effect of partial default. – When a

pleading asserting a claim states a common

cause of action against several defending

parties, some of whom answer and the others

fail to do so, the court shall try the case against

all upon the answers thus filed and render

judgment upon the evidence presented.

“(d) Extent of relief to be awarded. – A

judgment rendered against a party in default

shall not exceed the amount or be different in

kind from that prayed for nor award

unliquidated damages.

“(e) Where no defaults allowed. – If the

defending party in an action for annulment or

declaration of nullity of marriage or for legal

separation fails to answer, the court shall order

the prosecuting attorney to investigate whether

or nor a collusion between the parties exists, and

if there is no collusion, to intervene for the State

in order to see to it that the evidence submitted

is not fabricated.”

We now quote Section 1 of Rule 133:

“SECTION 1. Preponderance of evidence,

how determined. – In civil cases, the party

having the burden of proof must establish his

case by a preponderance of evidence. In

determining where the preponderance or

superior weight of evidence on the issues

involved lies, the court may consider all the

facts and circumstances of the case, the

witnesses’ manner of testifying, their

intelligence, their means and opportunity of

knowing the facts to which they are testifying,

the nature of the facts to which they testify, the

probability or improbability of their testimony,

their interest or want of interest, and also their

personal credibility so far as the same may

legitimately appear upon the trial. The court

may also consider the number of witnesses,

though the preponderance is not necessarily

with the greater number.”

Between the two rules, there is no incompatibility that

would preclude the application of either one of them. To

begin with, Section 3 of Rule 9 governs the procedure

which the trial court is directed to take when a defendant

fails to file an answer. According to this provision, the

court “shall proceed to render judgment granting the

claimant such relief as his pleading may warrant,” subject

to the court’s discretion on whether to require the

presentation of evidence ex parte. The same provision also

sets down guidelines on the nature and extent of the relief

that may be granted. In particular, the court’s judgment

“shall not exceed the amount or be different in kind from

that prayed for nor award unliquidated damages.”

As in other civil cases, basic is the rule that the party

making allegations has the burden of proving them by a

preponderance of evidence. Moreover, parties must rely on

the strength of their own evidence, not upon the weakness

of the defense offered by their opponent. This principle

holds true, especially when the latter has had no

opportunity to present evidence because of a default order.

Needless to say, the extent of the relief that may be granted

can only be as much as has been alleged and proved with

preponderant evidence required under Section 1 of Rule

133.” (Citations Omitted).

M. Judgment by default cannot grant relief not prayed for;

exception, when filed an answer but were absent during trial

Vlason Enterprises Corporation vs. CA, , said that judgment by

default cannot grant relief not prayed for; exception, when filed an answer

but were absent during trial.

The High Court in Vlason:

“A declaration or order of default is issued as a

punishment for unnecessary delay in joining issues. In

such event, defendants lose their standing in court, they

cannot expect the trial court to act upon their pleadings, and

they are not entitled to notice of the proceeding until the

final termination of the case. Thus, the trial court proceeds

with the reception of the plaintiff’s evidence upon which a

default judgment is rendered.

Section 1 of Rule 18 provides that after the defendant

has been declared in default, “the court shall proceed to

receive the plaintiff’s evidence and render judgment

granting him such relief as the complaint and the facts

proven may warrant.” The reliefs that may be granted,

however, are restricted by Section 5, which provides that a

judgment entered against a party in default shall not exceed

the amount or be different in kind from that prayed for.

In other words, under Section 1, a declaration of

default is not an admission of the truth or the validity of the

plaintiff’s claims. The claimant must still prove his claim

and present evidence. In this sense the law gives defaulting

parties some measure of protection because plaintiffs,

despite the default of defendants, are still required to

substantiate their allegations in the complaint. The

judgment of default against defendants who have not

appeared or filed their answers does not imply a waiver of

all their rights, except their right to be heard and to present

evidence in their favor. Their failure to answer does not

imply their admission of the facts and the causes of action

of the plaintiffs, because the latter are required to adduce

evidence to support their allegations.

Moreover, the trial court is not allowed by the Rules

to receive evidence that tends to show a relief not sought or

specified in the pleadings. The plaintiff cannot be granted

an award greater than or different in kind from that

specified in the complaint.

This case should be distinguished, however, from that

of defendants, who filed an answer but were absent during

trial. In that case, they can be held liable for an amount

greater than or different from that originally prayed for,

provided that the award is warranted by the proven facts.

This rule is premised on the theory that the adverse party

failed to object to evidence relating to an issue not raised in

the pleadings.” (Underscoring Supplied)

N. A default judgment only affects defaulted defendant

A default judgment against several defendants cannot affect the

rights of the other defendants who are not declared in default. The trial

court has no authority to allow plaintiff to evidence ex parte against a

defendant is not declared in default. To do otherwise will result to a

manifest failure or miscarriage of justice (Vlason enterprises Corporation

vs. CA,).

O. A default judgment cannot includean award not prayed for in

the complaint

A judgment in default proceeding cannot include an award not

prayed for in the complaint, even if proven ex parte, because the trial court

is not allowed by the Rules to receive evidence that tends to show a relief

not sought or specified in the pleadings and the plaintiff cannot be granted

an award greater than or different in kind from that specified in the

complaint (Vlason enterprises Corporation vs. CA,).

P. Remedy of defendant if plaintiff’s motion todeclare defendant

in default is granted

Settled is the rule that a party in default is entitled to notice of

subsequent proceedings but shall not take part in the trial; that he loses his

standing in court and his right to adduce evidence and to present his

defense, and if pleading is filed he should expect the trial court to act upon

their pleadings (Section 3 (a), Rule 9, 1997 Revised Rules of Civil

Procedure; Rural Bank of Sta. Catalina vs. Land Bank of the

Philippines,). Nevertheless, a party in default has still available remedies.

The following are the remedial measures available to defendants in

case he is ordered declared in default:

(1) a motion to set aside the order of default under Section

3(b), Rule 9 of the Rules of Court, if the default was

discovered before judgment could be rendered;

(2) a motion for new trial under Section 1(a) of Rule 37, if

the default was discovered after judgment but while

appeal is still available;

(3) a petition for relief under Rule 38, if judgment has

become final and executory; and

(4) an appeal from the judgment under Section 1, Rule 41,

even if no petition to set aside the order of default has

been resorted to (Lina vs. CA, 135 SCRA 637 cited in

Indiana Aerospace University vs. CHED, G.R. No.

139371, April 4, 2001).

Martinez vs. Republic () allows the filing of appeal under Section

1, Rule 41 of the 1997 Revised Rules of Civil Procedure on the ground

that the plaintiff failed to prove the material allegations of the complaint,

or that the decision is contrary to law, even without need of the prior filing

of a motion to set aside the order of default. In short, there is no need to

secure an order lifting an order of default.

In Rural Bank of Sta. Catalina vs. LBP, the High Court said:

It bears stressing that a defending party declared in

default loses his standing in court and his right to adduce

evidence and to present his defense. He, however, has the

right to appeal from the judgment by default and assail said

judgment on the ground, inter alia, that the amount of the

judgment is excessive or is different in kind from that

prayed for, or that the plaintiff failed to prove the material

allegations of his complaint, or that the decision is contrary

to law. Such party declare in default is proscribed from

seeking a modification or reversal of the assailed decision

on the basis of the evidence submitted by him in the Court

of Appeals, for if it were otherwise, he would thereby be

allowed to regain his right to adduce evidence, a right

which he lost in the trial court when he was declare in

default, and which he failed to have vacated. In this case,

the petitioner sought the modification of the decision of the

trial court based on the evidence submitted by it only in the

Court of Appeals.”

(Author’s observation: there is no need to secure an order of

lifting an order of default because filing a notice of appeal is considered

as a post judgment remedy).

The foregoing remedial measures is available to party in default if

the order declaring him in default is not a patent nullity or void, because if

the order declaring him in default is a clear and patent nullity, his available

remedy is certiorari under Rule 65.

Certiorari under Rule 65. Why certiorari under Rule 65?

The explanation is so simple.

An order granting a motion to declare defendant in default is

interlocutory nature (Indiana Aerospace University vs. CHED,). This is

because the order granting a motion to declare defendant in default does

not disposes of the case. Such an order is not a final order, because it

leaves something to be done by the court before the case is finally decided

on the merits (Philgreen Trading Construction Corporation vs. CA,).

Therefore the same cannot be a subject of an appeal under Section 1, Rule

41 of the 1997 Revised Rules of Civil Procedure.

Paragraph 2(c), Section 1, Rule 41, 1997 Revised Rules of Civil

Procedure provides that no appeal may be taken from an interlocutory

order. Therefore, the only available remedy is Certiorari under Rule 65

(Cerezo vs. David Tuazon,).

Certiorari is a remedy when any tribunal, board or officer

exercising judicial or quasi-judicial functions has acted without or in

excess its or his jurisdiction, or with grave abuse of discretion amounting

to lack or excess of jurisdiction, and there is no appeal, or any plain,

speedy, and adequate remedy in the ordinary course of law, a person

aggrieved thereby may file a verified petition in the proper court, alleging

the facts with certainty and praying that judgment be rendered annulling or

modifying the proceedings of such tribunal, board or officer, and granting

such incidental reliefs as law and justice may require (Section 1, Rule 65,

1997 Revised Rules of Civil Procedure).

In Guanzon vs. Arradaza, G., the High Court said that a petition

for certiorari under Rule 65 to declare the nullity of judgment by default is

also available if the trial court improperly decalred a party in default, or

even if the trial court properly decalred a party in default, if grave abuse of

discretion attended such declaration.

However, there is no harm if he will avail of the foregoing

remedial measures laid down in Lina case, but he may no longer later on

question the patent nullity of the order because he may deemed to have

waive his right to question the fact of nullity.

Same remedial measure also is available to plaintiff if his motion

to declare defendant in default is denied, or against order setting aside

order of default, because an order setting aside order of default is

interlocutory (Denso (Phils.), Inc., vs. IAC,).

In Indiana Aerospace University vs. CHED,), the subject of the

action is CHEd’s regulation or administration of educational institutions,

and therefore, treated as action imbued with public interest, hence, the

filing of the requisite motion for reconsideration was dispensed with

(Liberty Insurance Corporation vs. CA,).

The case of Spouse delos Santos vs. Judge Carpio) is also peculiar

in the sense that in this case, the trial judge motu proprio issued an order

declared defendant in default without waiting for the scheduled hearing on

the motion to declare defendant in default. That’s foul according to the

High Court, and said:

“We could not see any justifiable reason why the

trial court chose not to hear the petitioners on the date and

time fixed in Metrobank’s motion, and instead, hastily

granted the motion before it could be heard on the ground

that it had found the motion to be impressed with merit.

Indeed, in totally disregarding the purpose for which the

filing of a motion and notice to defending party are

required by the Rules, the trial court had acted in a despotic

manner that is correctly assailed through a petition for

certiorari which petitioners have seasonably filed with the

CA.”

“Again, respondent Judge acted capriciously when he

totally ignored petitioners’ Opposition to Metrobank’s

Motion to Declare them in Default and denied their Motion

to Admit Answer, both filed on February 15, 2001, a day

before the scheduled hearing, which showed their desire to

be heard before the motion to declare them in default is

resolved by the trial court.”

Q. Resorting of plaintiff to filing of motion for summary judgment

or judgment on the pleading not proper

Motions for summary judgment or judgment on the pleading are

not proper. Again, the reason is obvious. For these remedies to apply, it

presupposes that an answer is filed. For summary judgment to apply there

must exists a genuine issue (Narra Integrated Corporation vs. CA,).

Judgment on the pleadings is proper when an answer fails to tender an

issue, or otherwise admits the material allegations of the adverse party’s

pleading (Municipality of Tiwi vs. Betito,).

R. Requirements must comply with in filing motion to lift order of

default

The defendant in default may, at any time after discovery thereof

and before judgment, file a motion under oath to set aside the order of

default on the ground that his failure to answer was due to fraud, accident,

mistake or excusable negligence, and that he has a meritorious defense

(Section 3(b), Rule 9, 1997 Revised Rules of Civil Procedure).

In short, the motion must be under oath, and must contain an

allegation that the defendant’s failure to file his answer was due to fraud,

accident, mistake, or excusable negligence.

S. Certiorari (under Rule 65) not always grantable against an

order denying motion to lift order of default

In David vs. Judge Gutierrez-Fruelda, , it was held that the trial judge

did not commit grave abuse of discretion when she denied the motion to lift

order of default on ground the motion was not under oath. The motion did not

also contain an allegation that the defendant’s failure to file his answer

was due to fraud, accident, mistake, or excusable negligence.

T. Remedy of plaintiff if his motion to declare defendant in default

is denied

If there is grave abuse of discretion in denying the motion to

declare defendant in default, petition for certiorari under Rule 65 is the

remedy. Reason: an order denying the motion to declare defendant in

default is an interlocutory order.

U. Rule on default in counterclaim

If the defendant (plaintiff in the complaint) fails within the

reglementary period, that is, ten (10) days from service (Section 4, Rule

11, 1997 Revised Rules of Civil Procedure) within which to file an answer,

he, upon motion of the counterclaimant (or the plaintiff in the

counterclaim) may be declared in default (Section 3, Rule 9, 1997 Rules

of Civil Procedure; Francisco Motors Corporation vs. CA,).

---0---

DEMURRER TO EVIDENCE

A. Demurrer to evidence, definition, concept and purpose

It is defined as an objection by one of the parties in an action, to

the effect that the evidence which his adversary produced is insufficient in

point of law, whether true or not, to make out a case or sustain the issue

(Gutib vs. CA, G).

It authorizes a judgment on the merits of the case without the

defendant having to submit evidence on his part as he would ordinarily

have to do, if plaintiff’s evidence shows that he is not entitled to the relief

sought (Dandoy vs. CA,).

It is an aid or instrument for the expeditious termination of an

action, similar to a motion to dismiss, which the court or tribunal may

either grant or deny (Dandoy vs. CA,).

Its purpose is precisely to expeditiously terminate the case without

the need of the defendant’s evidence (Spouses Condes vs. CA,).

B. May be issued, when?

A demurrer to evidence may be issued when, upon the facts

adduced and the applicable law, the plaintiff has shown no right to relief.

Where the totality of plaintiff’s evidence, together with such inferences

and conclusions as may reasonably be drawn therefrom, does not warrant

recovery against the defendant, a demurrer to evidence should be

sustained (Dandoy vs. CA,). It may be granted if, after the presentation of

plaintiff’s evidence, it appears upon the facts and the law that the plaintiff

has shown no right to relief (Republic vs. Tuvera,).

A demurrer to evidence is likewise sustainable when, admitting

every proven fact favorable to the plaintiff and indulging in his favor all

conclusions fairly and reasonably inferable therefrom, the plaintiff has

failed to make out one or more of the material elements of his case, or

when there is no evidence to support an allegation necessary to his claim.

It should be sustained where the plaintiff’s evidence is prima facie

insufficient for a recovery (Dandoy vs. CA,).

It is, therefore, premature to speak of “preponderance of

evidence” in a demurrer to evidence because it is filed before the

defendant presents his evidence (Spouses Condes vs. CA).

C. When to file?

After the plaintiff has completed the presentation of his evidence,

the defendant may move for dismissal on the ground that upon the facts

and the law the plaintiff has shown no right to relief. If his motion is

denied, he shall have the right to present evidence. If the motion is granted

but on appeal the order of dismissal is reversed he shall be deemed to have

waived the right to present evidence (Section 1, Rule 33, 1997 Revised

Rules of Civil Procedure).

D. Resolving a demurrer to evidence, basis

What should be resolved in a motion to dismiss based on a

demurrer to evidence is whether the plaintiff is entitled to the relief based

on the facts and the law. The evidence contemplated by the rule on

demurrer is that which pertains to the merits of the case, excluding

technical aspects such as capacity to sue (Casent Realty Development

Corp. vs. Philbanking Corporation,).

The plaintiff’s evidence should not be the only basis in resolving a

demurrer to evidence. The “facts” referred to in Section 8, Rule 8 of the

Rules of Court should include all the means sanctioned by the Rules of

Court in ascertaining matters in judicial proceedings. These include

judicial admissions, matters of judicial notice, stipulations made during

the pre-trial and trial, admissions, and presumptions, the only exclusion

being the defendant’s evidence (Casent Realty Development Corp. vs.

Philbanking Corporation,). In short, Courts may consider other facts

within the range of judicial notice as well as relevant laws and

jurisprudence which the courts are bound to take into account, and they

are also fairly entitled to examine records/documents duly incorporated

into the complaint by the pleader himself in ruling on the demurrer to the

complaint (U. Bañez Electric Light Company vs. Abra Electric

Cooperative, Inc., et al.,).

E. Section 8, Rule 8 of the Rules of Court

When an action or defense is founded upon a written instrument,

copied in or attached to the corresponding pleading as provided in the

preceding section, the genuineness and due execution of the instrument

shall be deemed admitted unless the adverse party, under oath, specifically

denies them, and sets forth, what he claims to be the facts; but the

requirement of an oath does not apply when the adverse party does not

appear to be a party to the instrument or when compliance with an order

for an inspection of the original instrument is refused2 (See Casent Realty

Development Corp. vs. Philbanking Corporation,).

F. Judicial admission also considered in resolving the demurrer to

evidence

An admission, verbal or written, made by a party in the course of

the proceeding in the same case, does not require proof. The admission

may be contradicted only by showing that it was made through palpable

mistake or that no such admission was made3 (See Casent Realty

Development Corp. vs. Philbanking Corporation,).

In Casent Realty Development Corp. vs. Philbanking

Corporation, , it was held that court must consider the deemed admitted

genuineness and due execution of the Dacion and Confirmation Statement

under oath in resolving the demurrer to evidence.

The High Court said in Casent:

“On appeal to the CA, respondent claimed that even

though it failed to file a Reply, all the new matters alleged

in the Answer are deemed controverted anyway, pursuant

to Rule 6, Section 10:

2Section 8. How to contest such documents.

3Section 4. Judicial admissions.

Section 10. Reply.––A reply is a pleading,

the office or function of which is to deny, or

allege facts in denial or avoidance of new

matters alleged by way of defense in the answer

and thereby join or make issue as to such new

matters. If a party does not file such reply, all

the new matters alleged in the answer are

deemed controverted.

We agree with petitioner. Rule 8, Section 8

specifically applies to actions or defenses founded upon a

written instrument and provides the manner of denying it. It

is more controlling than Rule 6, Section 10 which merely

provides the effect of failure to file a Reply. Thus, where

the defense in the Answer is based on an actionable

document, a Reply specifically denying it under oath must

be made; otherwise, the genuineness and due execution of

the document will be deemed admitted. Since respondent

failed to deny the genuineness and due execution of the

Dacion and Confirmation Statement under oath, then these

are deemed admitted and must be considered by the court

in resolving the demurrer to evidence. We held in

Philippine American General Insurance Co., Inc. v. Sweet

Lines, Inc. that “[w]hen the due execution and genuineness

of an instrument are deemed admitted because of the

adverse party’s failure to make a specific verified denial

thereof, the instrument need not be presented formally in

evidence for it may be considered an admitted fact.”

G. Res judicata ground for demurrer to evidence, inappropriate

Res judicata is an inappropriate ground for sustaining a demurrer

to evidence, even as it stands as a proper ground for a motion to dismiss.

A demurrer may be granted if, after the presentation of plaintiff’s

evidence, it appears upon the facts and the law that the plaintiff has shown

no right to relief. In contrast, the grounds for res judicata present

themselves even before the presentation of evidence, and it should be at

that stage that the defense of res judicata should be invoked as a ground

for dismissal. Properly speaking, the movants for demurral who wish to

rely on a controlling value of a settled case as a ground for demurrer

should invoke the ground of stare decisis in lieu of res judicata (Republic

vs. Tuvera, G).

H. Effect of judgment on demurrer to evidence

Again, after the plaintiff has completed the presentation of his

evidence, the defendant may move for dismissal on the ground that upon

the facts and the law the plaintiff has shown no right to relief. If his

motion is denied, he shall have the right to present evidence. If the motion

is granted but on appeal the order of dismissal is reversed he shall have be

deemed to have waived the right to present evidence (Section 1, Rule 33,

1997 Revised Rules of Civil Procedure).

The general rule is that upon the dismissal of the demurrer in the

appellate court, the defendant loses the right to present his evidence and

the appellate court shall then proceed to render judgment on the merits

on the basis of plaintiff’s evidence (Republic vs. Tuvera,).

The rationale behind the rule and doctrine is simple and logical.

The defendant is permitted, without waiving his right to offer evidence in

the event that his motion is not granted, to move for a dismissal (i.e.,

demur to the plaintiff’s evidence) on the ground that upon the facts as thus

established and the applicable law, the plaintiff has shown no right to

relief. If the trial court denies the dismissal motion, i.e., finds that

plaintiff’s evidence is sufficient for an award of judgment in the absence

of contrary evidence, the case still remains before the trial court which

should then proceed to hear and receive the defendant’s evidence so that

all the facts and evidence of the contending parties may be properly placed

before it for adjudication as well as before the appellate courts, in case of

appeal. Nothing is lost. The doctrine is but in line with the established

procedural precepts in the conduct of trials that the trial court liberally

receive all proffered evidence at the trial to enable it to render its decision

with all possibly relevant proofs in the record, thus assuring that the

appellate courts upon appeal have all the material before them necessary

to make a correct judgment, and avoiding the need of remanding the case

for retrial or reception of improperly excluded evidence, with the

possibility thereafter of still another appeal, with all the concomitant

delays. The rule, however, imposes the condition by the same token that if

his demurrer is granted by the trial court, and the order of dismissal is

reversed on appeal, the movant loses his right to present evidence in his

behalf and he shall have been deemed to have elected to stand on the

insufficiency of plaintiff’s case and evidence. In such event, the appellate

court which reverses the order of dismissal shall proceed to render

judgment on the merits on the basis of plaintiff’s evidence (Republic vs.

Tuvera,).

I. Judicial action on demurrer to evidence is discretionary

A judicial action on a motion to dismiss on demurrer to evidence

rests within the sound discretion of the court. In addition, an order denying

a demurrer to evidence is interlocutory. It is not appealable. Neither can

it be the subject of a petition for certiorari in the absence of grave abuse of

discretion or excess of jurisdiction, or an oppressive exercise of judicial

authority (Katigbak vs. Sandiganbayan,).

As experienced in the Bench, petitions for certiorari under Rule 65

against the denial of demurrer to evidence (in criminal cases) are usually

filed by aggrieved parties arguing that a one or two liner sentences

denying the demurrer and the motion for reconsideration for utter lack of

merit violates Section 1, Rule 36 of the 1997 Revised Rules of Civil

Procedure, and Section 14, Article VIII of the 1987 Constitution.

Section 14, Article VIII of the 1987 Constitution

provides that “No decision shall be rendered by any court

without expressing therein clearly and distinctly the facts

and the law on which it is based.”

Section 1, Rule 36 of the 1997 Revised Rules of Civil

Procedure provides that “A judgment or final order

determining the merits of the case shall be in writing

personally and directly prepared by the judge, stating

clearly and distinctly the facts and the law on which is it

is based, signed by him, and filed with the clerk of court.”

This is not a good ground of filing a petition for certiorari under

Rule 65.

An Order resolving a demurrer to evidence has double-character:

first, a final order, and second, as an interlocutory order. If it is a final

order, the, the remedy available to the aggrieved party is appeal, and if it is

an interlocutory order, the remedy is petition for certiorari under Rule 65.

The relevance of having this matter discussed herein is to determine the

importance of observing the decision writing foundations provided in

Section 14, Article VIII of the 1987 Constitution and Section 1, Rule 36 of

the 1997 Revised Rules of Civil Procedure.

The requirement of specificity of rulings is stringently applied only

to judgments and final orders,4 but not to interlocutory one.

If the order denying a demurrer to evidence and the motion for

reconsideration for the denial do not contain clearly and distinctly the facts

and the law on which it is based, it is believed that constitutional and

procedural requirements are not violated because the order is not in the

nature of a final order or judgment that completely disposing of the case.

Judicial technique is sometimes observed by some courts wherein

they will just issue an order of outright denial of demurrer to evidence in

order bit the pressing deadline to resolve pending incidents because of

tremendous judicial paper works on equally important cases rather than

disposing the case on the merit which undeniably requires more precious

time doing research work.

An order granting a demurrer to evidence, both in civil and

criminal cases, partake the nature of a final order, because nothing is left

to the court that issued the order. The issuance of this type of order is the

order (final character) that the trial judge should comply with the requisite

constitutional and procedural mandates. This, the trial judge must state

clearly and distinctly the facts and the law on which is it is based. The

rationale behind why the final order should state clearly and distinctly the

facts and the law on which is it is based is because for the aggrieved party

to easily know why he lost, so that he can easily evaluate what error or

errors that he can assign or impute to the court that issued it if he desires

4Dandoy vs. CA, G.R. No. 150089, August 28, 2007.

and permitted to elevate the matter on appeal. This is the reason why the

remedy available to the aggrieved party is appeal and not certiorari under

Rule 65.

On the other hand, an order denying a demurrer to evidence is

interlocutory order. The constitutional provision does not apply to

interlocutory order, because after the issuance of the order of denial there

will be continuing proceedings that will takes place, i.e.,

actions/proceedings left to be done by the court, such as such as

presentation of defense evidence. This is the reason why the remedy

available under paragraph 2 (c), in relation to paragraph 3, Section 1,5 Rule

41, 1997 Revised Rules of Civil Procedure is certiorari, and not appeal.

Besides, the order of denial is not subject to the requirement of

Section 14, Article VIII of the 1987 Constitution, because as what the

High Court held in Dandoy vs. CA, the order of denial neither terminate

nor finally dispose of the case as there are proceedings still left to be done

by the court before the case is finally decided on the merits.

I. An order denying the demurrer to evidence reviewable by

Certiorari under Section 1, Rule 65 of the Rules of Court, and

when not

Generally, the order denying the motion for leave of court to file

demurrer to evidence or the demurrer itself shall not be reviewable by

appeal or by certiorari before judgment,6 because action on a demurrer or

on a motion to dismiss rests on the sound exercise of judicial discretion.7 It

5“No appeal may be taken from: (c) An interlocutory order.”

“In all the above instances where judgment or final order is not appealable, the aggrieved

party may file an appropriate special civil action under Rule 65. 6Section 23, Rule 119, 2000 Revised Rules of Criminal Procedure. 7Tan vs. Court of Appeals, 347 Phil. 320, 329 (1997); Bernardo vs. Court of Appeals, 344

Phil. 335, 346 (1997) cited in Nicolas vs. Sandiganbayan and the companion case, G.R.

Nos. 175930-31, February 11, 2008. Also in People vs. Singh, G.R. No. 129782, June 29,

2001, 360 SCRA 404; People vs. Mercado, No. L-33492, March 30, 1988, 159 SCRA

453 cited in People vs. Almendras, G.R. No. 145915, April 24, 2003.

is an interlocutory order, not appealable neither can it be the subject of a

petition for certiorari.8

However, this admits an exception: when the denial of a demurrer

to evidence is attended by grave abuse of discretion, patently erroneous or

issued with grave abuse of discretion.9 Though interlocutory in character,

an order denying a demurrer to evidence may be the subject of a certiorari

proceeding, provided the petitioner can show that it was issued with grave

abuse of discretion; and that appeal in due course is not plain, adequate or

speedy under the circumstances. When the plaintiff’s evidence is utterly

and patently insufficient to prove the complaint, it would be capricious for

a trial judge to deny the demurrer and to require the defendant to present

evidence to controvert a non- existing case. The denial of the demurrer to

evidence will constitute an unwelcome imposition on the court’s docket

and an assault on the defendant’s resources and peace of mind, and if

denied, it effectively denies justice.10

---0---

POST JUDGMENT REMEDIES

Fairness dictates that a party who has not appealed from a

judgment of the trial court is bound by the terms of the judgment.11

Let’s say plaintiff, the party that prevails in the decision, upon

finality of the judgment he should move to execute the same to enjoy the

fruits and result of his litigation.

But, how if the loosing party wants to exhaust other remedies to

protect his interest subject of the suit, what are his available remedies?

8David vs. Rivera, 464 Phil. 1006; Tadeo vs. People, 360 Phil. 914, 919 (1998); Cruz vs.

People, 363 Phil. 156; Katigbak vs. Sandiganbayan, 453 Phil. 515 cited in Nicolas vs.

Sandiganbayan and the companion case, G.R. Nos. 175930-31, February 11, 2008. 9People vs. Ong, G.R. No. 140904, October 9, 2000 citing Cruz vs. People (303 SCRA

533 [1999]). 10Choa vs. Choa, G.R. No. 143376, November 26, 2002. 11

Five Star Bus, Co, Inc. vs. CA, G.R. No. 120496, July 17, 1996.

Depending upon each applicability requirement, the following are

the remedies available to the loosing or aggrieved party, namely:

1. execution of judgment

2. appeal

3. motion for reconsideration

4. motion for new trial

5. petition for relief from judgment

6. annulment of judgment

EXECUTION OF JUDGMENT (Rule 39)

Litigation must end and terminate sometime and somewhere, and it

is essential to an effective and efficient administration of justice that once

a judgment has become final, the winning party be not, through a mere

subterfuge, deprived of the fruits of the verdict (Li Kim Tho vs. Sanchez,).

Once a judgment becomes final, it is basic that the prevailing party

is entitled as a matter of right to a writ of execution the issuance of which

is the trial court’s ministerial duty (Torno vs. IAC,).

Final and executory judgments are enforced by a writ of execution

not by mandamus.12 Execution is the fruit and end of the suit and is very

aptly called the life of the law.13 It is the process which carries into effect

a decree or judgment.14

Execution of a judgment can be issued only against a party to the

action and not against one who did not have his day in court (St. Dominic

Corporation vs. IAC,), and therefore, a strangers to a case are not bound

by the judgment rendered by a court. It will not divest the rights of a party

12

Vital-Gozon vs. CA, G.R. No. 101428, August 5, 1992. 13

Ipekdjian Merchandising Co. vs. Court of Tax Appeals, 8 SCRA 59 cited in PAL, Inc.

vs. CA, G.R. No. L-49188, January 30, 1990. 14

Painter v. Berglund, 31 Cal. App. 2d. 63, 87 P 2d 360, 363; Miller v. London, 294 Mass

300, 1 NE 2d 198, 200; Black's Law Dictionary cited in PAL, Inc. vs. CA, G.R. No. L-

49188, January 30, 1990.

who has not and never been a party to a litigation (Panotes vs. City

Townhouse Development Corporation,).

Execution of judgment vs. Cash

The sheriff shall demand judgment obligor the immediate payment

in CASH of the full stated in the writ of execution and all lawful fees.

He is not required to give the judgment debtor some time to raise

cash because the property may be placed in danger of being lost or

absconded, otherwise the sheriff may be administratively disciplined

(Torres vs. Cabling, July 11, 1997).

The executing sheriff is prohibited from demanding payment by

check payable to him (Section 9 (a), Rule 39, 1997 Revised Rules of Civil

Procedure).

A. Third party claim

When a third person claims interest in the property levied upon, his

remedy is to file a “third party claim” (or “tercera).

When the sheriff seizes property of a third person in which the

judgment debtor holds no right or interest, the supervisory power of the

court which has authorized execution may be invoked by the third person.

Upon due application by the third person, and after summary hearing, the

court may command that the property be released from the mistaken levy

and restored to the rightful owner or possessor. If that is the case, the

court’s duty is limited to determine whether the sheriff has acted rightly or

wrongly in the performance of his duties in the execution of judgment,

more specifically, if the sheriff has indeed taken hold of the property not

belonging to the judgment debtor. The court does not and cannot pass

upon the question of title to the property, with any character of finality; it

can treat that matter only in so far as may be necessary to decide if the

sheriff has acted correctly or not (Quebral vs. Garduno,).

The court can require the sheriff to restore the property to the

claimant’s possession if warranted by the evidence; if claimant’s proofs do

not however persuade the court his title or right of possession thereof, his

claim will be denied (Ong vs. Tating, G).

B. Execution of Judgment vs. Bank Deposits

Execution of judgment against bank deposits is through

garnishment.

Garnishment has been defined as a species of attachment for

reaching any property or credits pertaining or payable to a judgment

debtor. In legal contemplation, it is a forced novation by the substitution

of creditors: the judgment debtor, who is the original creditor of the

garnishee is, through service of the writ of garnishment, substituted by the

judgment creditor who thereby becomes creditor of the garnishee.

Garnishment has also been described as a warning to a person having in

his possession property or credits of the judgment debtor, not to pay the

money or deliver the property to the latter, but rather to appear and answer

the plaintiff's suit (Perla Compania De Seguros, Inc., vs. Judge

Ramolete,).

C. Alias writ of execution be issued without a prior return of the

original writ by the implementing officer

The issue of whether an alias writ of execution be issued without a

prior return of the original writ by the implementing officer had already

been settled by the High Court in PAL, Inc. vs. CA,. Resolving in the

affirmative, the High Court said in PAL:

“We rule in the affirmative and we quote the

respondent court's decision with approval:

"The issuance of the questioned alias writ

of execution under the circumstances here

obtaining is justified because even with the

absence of a Sheriff's return on the original writ,

the unalterable fact remains that such a return is

incapable of being obtained (sic) because the

officer who is to make the said return has

absconded and cannot be brought to the Court

despite the earlier order of the court for him to

appear for this purpose. (Order of Feb. 21, 1978,

Annex C, Petition). Obviously, taking

cognizance of this circumstance, the order of

May 11, 1978 directing the issuance of an alias

writ was therefore issued. (Annex D, Petition).

The need for such a return as a condition

precedent for the issuance of an alias writ was

justifiably dispensed with by the court below

and its action in this regard meets with our

concurrence. A contrary view will produce an

abhorent situation whereby the mischief of an

erring officer of the court could be utilized to

impede indefinitely the undisputed and awarded

rights which a prevailing party rightfully

deserves to obtain and with dispatch. The final

judgment in this case should not indeed be

permitted to become illusory or incapable of

execution for an indefinite and over extended

period, as had already transpired." (Rollo, pp.

35-36)

Judicium non debet esse illusorium; suum effectum

habere debet (A judgment ought not to be illusory; it ought

to have its proper effect).

Indeed, technicality cannot be countenanced to defeat

the execution of a judgment for execution is the fruit and

end of the suit and is very aptly called the life of the law

(Ipekdjian Merchandising Co. v. Court of Tax Appeals, 8

SCRA 59 [1963]; Commissioner of Internal Revenue v.

Visayan Electric Co., 19 SCRA 697, 698 [1967]). A

judgment cannot be rendered nugatory by the unreasonable

application of a strict rule of procedure. Vested rights were

never intended to rest on the requirement of a return, the

office of which is merely to inform the court and the

parties, of any and all actions taken under the writ of

execution. Where such information can be established in

some other manner, the absence of an executing officer's

return will not preclude a judgment from being treated as

discharged or being executed through an alias writ of

execution as the case may be. More so, as in the case at bar.

Where the return cannot be expected to be forthcoming, to

require the same would be to compel the enforcement of

rights under a judgment to rest on an impossibility, thereby

allowing the total avoidance of judgment debts. So long as

a judgment is not satisfied, a plaintiff is entitled to other

writs of execution (Government of the Philippines v.

Echaus and Gonzales, 71 Phil. 318). It is a well known

legal maxim that he who cannot prosecute his judgment

with effect, sues his case vainly.”

G. Execution cannot be equated with satisfaction of a judgment;

execution and satisfaction of judgment, distinguished

PAL, Inc. vs. CA, said that execution is the process which carries

into effect a decree or judgment (Painter v. Berglund, 31 Cal. App. 2d. 63,

87 P 2d 360, 363; Miller v. London, 294 Mass 300, 1 NE 2d 198, 200;

Black's Law Dictionary), whereas the satisfaction of a judgment is the

payment of the amount of the writ, or a lawful tender thereof, or the

conversion by sale of the debtor's property into an amount equal to that

due, and, it may done otherwise, than upon an execution (Section 47, Rule

39). Levy and delivery by an execution officer are not prerequisites to the

satisfaction of a judgment when the same has already been realized in fact

(Section 47, Rule 39). Execution is for the sheriff to accomplish while

satisfaction of the judgment is for the creditor to achieve. Section 15,

Rule 39 merely provides the sheriff with his duties as executing officer

including delivery of the proceeds of his levy on the debtor's property to

satisfy the judgment debt. It is but to stress that the implementing officer's

duty should not stop at his receipt of payments but must continue until

payment is delivered to the obligor or creditor (G.R. No. L-49188,

January 30, 1990).

H. Motion to quash execution when proper

A motion to quash execution is only proper where:

(a) the writ of execution varies the judgment;

(b) there has been a change in the situation of the parties

making execution inequitable or unjust;

(c) execution is sought to be enforced against property

exempt from execution;

(d) it appears that the controversy has never been submitted

to the judgment of the court;

(e) the terms of the judgment are not clear enough and there

remains room for interpretation thereof; or

(f) it appears that the writ of execution has been

improvidently issued, or that it is defective in substance

or is issued against the wrong party, or that the

judgment debt has been paid or otherwise satisfied, or

the writ was issued without authority (Gutierrez vs,

Valiente,).

I. Execution pending appeal

On motion of the prevailing party will notice to the adverse party

filed in the trial court while it has jurisdiction over the case and is in

possession of either the original record or the record on appeal, as the case

may be, at the time of the filing of such motion, said court may, in its

discretion, order execution of a judgment or final order even before the

expiration of the period to appeal.

After the trial court has lost jurisdiction, the motion for execution

pending appeal may be filed in the appellate court.

Discretionary execution may only issue upon good reasons to be

stated in a special order after due hearing (Section 2, Rule 39, 1997

Revised Rules of Civil Procedure).

Discretionary execution of appealed judgments may be allowed

upon concurrence of the following requisites:

(a) there must be a motion by the prevailing party with notice

to the adverse party;

(b) there must be a good reason for execution pending

appeal; and

(c) the good reason must be stated in a special order

(Manacop vs. Equitable Banking Corporation,).

The exercise of the power to grant or deny immediate execution is

addressed to the sound discretion of the court and the existence of good

reasons is precisely what confers such discretionary power upon the court

(Philippine National Bank vs. Puno,).

J. Good Reasons

Good reasons consist of compelling circumstances justifying

immediate execution lest judgment becomes illusory, or the prevailing

party after the lapse of time be unable to enjoy it, considering the tactics of

the adverse party who may have apparently no cause but to delay. Such

reasons must constitute superior circumstances demanding urgency which

will outweigh the injury or damages should the losing party secure a

reversal of the judgment. Execution of a judgment pending appeal is an

exception to the general rule that only a final judgment may be executed.

Thus, the existence of “good reasons” is essential for it is what confers

discretionary power on a court to issue a writ of execution pending appeal

(Villamor vs. National Power Corporation,).

J. Supersedeas Bond

The purpose of the supersedeas bond is to answer for the rents,

damages and costs accruing down to the judgment of the inferior court

appealed from, the amount of which is to be determined from the

judgment of said court. The postulation of complainants and their counsel

that the execution sought was effectively stayed by the filing of a

supersedeas bond was sufficiently refuted and justifiably rejected when we

consider the circumstances then obtaining (Cordova vs. Judge Labayen,

A).

---0---

REVIVAL OF JUDGMENT

A. The principle and Rule

An action for revival of judgment is no more than a procedural

means of securing the execution of a previous judgment which has

become dormant after the passage of five years without it being executed

upon motion of the prevailing party. It is not intended to re-open any issue

affecting the merits of the judgment debtor’s case nor the propriety or

correctness of the first judgment (Panotes, vs. City Townhouse

Development Corporation,).

It is premised on the assumption that the decision to be revived,

either by motion or by independent action, is already final and executory

(Saligumba vs. Palanog,).

A final and executory judgment or order may be executed on

motion within five (5) years from the date of its entry. After the lapse of

such time, and before it is barred by the statute of limitations, a judgment

may be enforced by action. The revived judgment may also be enforced by

motion within five (5) years from the date of its entry and thereafter by

action before it is barred by the statute of limitations (Section 6, Rule 39,

1997 Revised Rules of Civil Procedure [Execution by motion or by

independent action]).

The purpose of the law in prescribing time limitations for

enforcing judgment by action is precisely to prevent the winning parties

from sleeping on their rights (Macias vs. Lim,).

B. It is a new and independent action

An action for revival of judgment is a new and independent action,

different and distinct from either the recovery of property case or the

reconstitution case, wherein the cause of action is the decision itself and

not the merits of the action upon which the judgment sought to be

enforced is rendered (Juco vs. Heirs of Tomas Siy Chung Fu,).

C. Period within which to file action for revival of judgment

An action may be revived by mere motion within the period

between five (5) years from the date of entry of judgment, and by

independent action within the period or between five (5) years and one (1)

day from the date of entry of judgment to ten (10) years.

The guide in filing a revival of action is that: first, observe the 10

year prescription period counted from the date said judgment become final

or from the date of its entry, and second, observe the five (5) year period

mentioned in Section 6, Rule 39 of the Rules of Court – final and

executory judgment or order may be executed on motion within five (5)

years from the date of its entry.

Again, this may be by mere motion or by action. If the action for

revival of judgment is filed after the five (5) year period provided for in

the Rules of Court, but beyond the ten (10) year period provided for in the

Civil Code, the action is considered barred by the statute of limitations,

thus, an action can no longer be enforced by action.

In Quesada vs. CA, , a writ of execution was issued on September

15, 1976, but was not enforced. An action for revival of judgment was

filed on August 26, 1985. The CA ruled that the action was already barred

by prescription. The High Court ruled that the action to revived judgment

was not barred by prescription because it was filed within the 10 year

period.

The period within which to file action for revival of judgment was

explained by the High Court in Shipside Incorporated vs. CA, , thus:

“The action instituted by the Solicitor General in the

trial court is one for revival of judgment which is governed

by Article 1144(3) of the Civil Code and Section 6, Rule 39

of the 1997 Rules on Civil Procedure. Article 1144(3)

provides that an action upon a judgment “must be brought

within 10 years from the time the right of action accrues."

On the other hand, Section 6, Rule 39 provides that a final

and executor judgment or order may be executed on motion

within five (5) years from the date of its entry, but that after

the lapse of such time, and before it is barred by the statute

of limitations, a judgment may be enforced by action.

Taking these two provisions into consideration, it is plain

that an action for revival of judgment must be brought

within ten years from the time said judgment becomes

final.”

In this case (Shipside Incorporated), the High Court

said that:

“From the records of this case, it is clear

that the judgment sought to be revived became

final on October 23, 1973. On the other hand,

the action for revival of judgment was instituted

only in 1999, or more than twenty-five (25)

years after the judgment had become final.

Hence, the action is barred by extinctive

prescription considering that such an action can

be instituted only within ten (10) years from the

time the cause of action accrues.”

0-0-0

In Villeza vs. German Management and Services, Inc, , made to

emphasize that an action for revival of judgment is governed by Article

1144 (3), Article 1152 of the Civil Code and Section 6, Rule 39 of the

Rules of Court; and that judgment must be enforced by the institution of a

complaint in a regular court within ten years from the time the judgment

becomes final.

The High Court said:

“Art. 1144. The following actions must be brought within ten years from the time the right of action accrues:

x x x x

(3) Upon a judgment

Article 1152 of the Civil Code states:

Art. 1152. The period for prescription of

actions to demand the fulfillment of obligations

declared by a judgment commences from the

time the judgment became final.

Apropos, Section 6, Rule 39 of the Rules of Court reads:

Sec. 6. Execution by motion or by

independent action. –A final and executory

judgment or order may be executed on motion

within five (5) years from the date of its entry.

After the lapse of such time, and before it is

barred by the statute of limitations, a judgment

may be enforced by action. The revived

judgment may also be enforced by motion

within five (5) years from the date of its entry

and thereafter by action before it is barred by

the statute of limitations. (emphasis supplied)

The rules are clear. Once a judgment becomes final

and executory, the prevailing party can have it executed as

a matter of right by mere motion within five years from the

date of entry of judgment. If the prevailing party fails to

have the decision enforced by a motion after the lapse of

five years, the said judgment is reduced to a right of action

which must be enforced by the institution of a complaint in

a regular court within ten years from the time the judgment

becomes final.”

“When petitioner Villeza filed the complaint for revival of

judgment on October 3, 2000, it had already been eleven (11) years from

the finality of the judgment he sought to revive. Clearly, the statute of

limitations had set in” said the High Court.15

The High Court added: “xxx xxx xxx it was petitioner Villeza, the

prevailing party himself, who moved to defer the execution of judgment.

The losing party never had any hand in the delay of its execution. Neither

did the parties have any agreement on that matter. After the lapse of five

years (5) from the finality of judgment, petitioner Villeza should have

instead filed a complaint for its revival in accordance with Section 6, Rule

39 of the Rules of Court. He, however, filed a motion to execute the same

which was a wrong course of action. On the 11th

year, he finally sought

its revival but he requested the aid of the courts too late.”16

-0-0-0-

D. Instances that prescriptive period within which to revive action

is interrupted or tolled

(1). If it is impossible for the winning party to have sought the

execution of the judgment because of the dilatory schemes and maneuvers

resorted to by the other party (Republic vs. CA,).

(2). There is an agreement between parties to defer or suspend the

enforcement of the judgment, or when the order of ejectment was not

carried out, however, due to the judgment debtor’s begging to withhold

the execution of judgment because of financial difficulties (Torralba vs.

delos Angeles,).

15Villeza vs. German Management and Services, Inc, G.R. No. 182937, August 8, 2010. 16Villeza vs. German Management and Services, Inc, G.R. No. 182937, August 8, 2010.

(3). When the oblior moves for the suspension of the writ of

execution, or when the obligee was not in delay because he exhausted all

legal means within his power to eject the obligor from his land; or where

the writs of execution issued by the lower court were not complied with

and/or were suspended by reason of acts or causes not of obligee’s own

making and against his objections (Casela vs. CA).

E. Venue of revival of judgment involving personal action (accion

in personam) and real action

In revival of judgment, if the revival of judgment action is a

personal one (accion in personam), the venue of the action is in the

residence of the plaintiff, or at the latter’s option, residence of the

defendant, and if the revival of judgment action involves a real action, the

venue of the action is in the place where the property subject of the revival

action is situated (Infante vs, Aran Builders,).

The High Court in Infante:

“xxx xxx xxx. Petitioner claims that the CA erred in

finding that the complaint for revival of judgment is an

action in rem which was correctly filed with the RTC of the

place where the disputed real property is located.

The petition is unmeritorious.

Petitioner insists that the action for revival of

judgment is an action in personam; therefore, the complaint

should be filed with the RTC of the place where either

petitioner or private respondent resides. Petitioner then

concludes that the filing of the action for revival of

judgment with the RTC of Muntinlupa City, the place

where the disputed property is located, should be dismissed

on the ground of improper venue.

Private respondent is of the opinion that the judgment

it is seeking to revive involves interest over real property.

As such, the present action for revival is a real action, and

venue was properly laid with the court of the place where

the realty is located.

Thus, the question that must be answered is: where is

the proper venue of the present action for revival of

judgment?

Section 6, Rule 39 of the 1997 Rules of Civil

Procedure provides that after the lapse of five (5) years

from entry of judgment and before it is barred by the statute

of limitations, a final and executor judgment or order may

be enforced by action. The Rule does not specify in which

court the action for revival of judgment should be filed.

In Aldeguer v. Gemelo, the Court held that:

x x x an action upon a judgment must be

brought either in the same court where said

judgment was rendered or in the place where the

plaintiff or defendant resides, or in any other

place designated by the statutes which treat

of the venue of actions in general. (Emphasis

supplied)

but emphasized that other provisions in the rules of

procedure which fix the venue of actions in general must be

considered.

Under the present Rules of Court, Sections 1 and 2 of

Rule 4 provide:

Section 1. Venue of real actions. -

Actions affecting title to or possession of real

property, or interest therein, shall be

commenced and tried in the proper court which

has jurisdiction over the area wherein the real

property involved, or a portion thereof, is

situated.

x x x x

Section 2. Venue of personal actions. -

All other actions may be commenced and tried

where the plaintiff or any of the principal

plaintiffs resides, or where the defendant or any

of the principal defendants resides, or in the

case of a non-resident defendant where he may

be found, at the election of the plaintiff.

Thus, the proper venue depends on the determination

of whether the present action for revival of judgment is a

real action or a personal action. Applying the afore-quoted

rules on venue, if the action for revival of judgment affects

title to or possession of real property, or interest therein,

then it is a real action that must be filed with the court of

the place where the real property is located. If such action

does not fall under the category of real actions, it is then a

personal action that may be filed with the court of the place

where the plaintiff or defendant resides.

In support of her contention that the action for revival

of judgment is a personal action and should be filed in the

court of the place where either the plaintiff or defendant

resides, petitioner cites the statements made by the Court in

Aldeguer v. Gemelo and Donnelly v. Court of First Instance

of Manila. Petitioner, however, seriously misunderstood

the Court's rulings in said cases.

In Aldeguer, what the Court stated was that “[t]he

action for the execution of a judgment for damages is a

personal one, and under section 377 [of the Code of Civil

Procedure], it should be brought in any province where the

plaintiff or the defendant resides, at the election of the

plaintiff” (Emphasis and underscoring supplied). Petitioner

apparently took such statement to mean that any action for

revival of judgment should be considered as a personal one.

This thinking is incorrect. The Court specified that the

judgment sought to be revived in said case was a judgment

for damages. The judgment subject of the action for

revival did not involve or affect any title to or possession of

real property or any interest therein. The complaint filed in

the rivival case did not fall under the category of real

actions and, thus, the action necessarily fell under the

category of personal actions.

In Donnelly, the portion of the Decision being relied

upon by petitioner stated thus:

Petitioner raises before this Court two (2)

issues, namely: (a) whether an action for revival

of judgment is one quasi in rem and, therefore,

service of summons may be effected thru

publication; and (b) whether the second action

for revival of judgment (Civil Case No. 76166)

has already prescribed. To our mind, the first

is not a proper and justiciable issue in the

present proceedings x x x. Nevertheless, let it

be said that an action to revive a judgment is a

personal one. (Emphasis supplied)

The Court clearly pointed out that in said case, the

issue on whether an action for revival of judgment is quasi

in rem was not yet proper and justiciable. Therefore, the

foregoing statement cannot be used as a precedent, as it

was merely an obiter dictum. Moreover, as in Aldeguer,

the judgment sought to be revived in Donnelly involved

judgment for a certain sum of money. Again, no title or

interest in real property was involved. It is then

understandable that the action for revival in said case was

categorized as a personal one.

Clearly, the Court's classification in Aldeguer and

Donnelly of the actions for revival of judgment as being

personal in character does not apply to the present case.

The allegations in the complaint for revival of

judgment determine whether it is a real action or a personal

action.

The complaint for revival of judgment alleges that a

final and executor judgment has ordered herein petitioner to

execute a deed of sale over a parcel of land in Ayala

Alabang Subdivision in favor of herein private respondent;

pay all pertinent taxes in connection with said sale; register

the deed of sale with the Registry of Deeds and deliver to

Ayala Corporation the certificate of title issued in the name

of private respondent. The same judgment ordered private

respondent to pay petitioner the sum of P321,918.25 upon

petitioner's compliance with the aforementioned order. It is

further alleged that petitioner refused to comply with her

judgment obligations despite private respondent's repeated

requests and demands, and that the latter was compelled to

file the action for revival of judgment. Private respondent

then prayed that the judgment be revived and a writ of

execution be issued to enforce said judgment.

The previous judgment has conclusively declared

private respondent's right to have the title over the disputed

property conveyed to it. It is, therefore, undeniable that

private respondent has an established interest over the lot in

question; and to protect such right or interest, private

respondent brought suit to revive the previous judgment.

The sole reason for the present action to revive is the

enforcement of private respondent's adjudged rights over a

piece of realty. Verily, the action falls under the category

of a real action, for it affects private respondent's interest

over real property.

The present case for revival of judgment being a real

action, the complaint should indeed be filed with the

Regional Trial Court of the place where the realty is

located.

Section 18 of Batas Pambansa Bilang 129 provides:

Sec. 18. Authority to define territory

appurtenant to each branch. - The Supreme

Court shall define the territory over which a

branch of the Regional Trial Court shall

exercise its authority. The territory thus

defined shall be deemed to be the territorial

area of the branch concerned for purposes of

determining the venue of all suits,

proceedings or actions, whether civil or

criminal, as well as determining the

Metropolitan Trial Courts, Municipal Trial

Courts and Municipal Circuit Trial Courts over

which the said branch may exercise appellate

jurisdiction. The power herein granted shall be

exercised with a view to making the courts

readily accessible to the people of the different

parts of the region and making the attendance of

litigants and witnesses as inexpensive as

possible. (Emphasis supplied)

From the foregoing, it is quite clear that a branch of

the Regional Trial Court shall exercise its authority only

over a particular territory defined by the Supreme

Court. Originally, Muntinlupa City was under the

territorial jurisdiction of the Makati Courts. However,

Section 4 of Republic Act No. 7154, entitled An Act to

Amend Section Fourteen of Batas Pambansa Bilang 129,

Otherwise Known As The Judiciary Reorganization Act of

1981, took effect on September 4, 1991. Said law provided

for the creation of a branch of the Regional Trial Court in

Muntinlupa. Thus, it is now the Regional Trial Court in

Muntinlupa City which has territorial jurisdiction or

authority to validly issue orders and processes concerning

real property within Muntinlupa City.

Thus, there was no grave abuse of discretion

committed by the Regional Trial Court of Muntinlupa City,

Branch 276 when it denied petitioner's motion to dismiss;

and the CA did not commit any error in affirming the same.”

F. Judgment does not attain finality when record is destroyed

during pendency of motion for reconsideration

When the records of the original case were destroyed in the fire

during the pendency of the motion for reconsideration of the disapproval

of the record on appeal, a motion for reconsideration has the effect of

suspending the statutory period after which an order, decision, or

judgment, in connection with which said motion was filed, becomes final.

The motion for reconsideration prevents the decision from attaining

finality. Cannot therefore be a proper subject of an action for revival of

judgment (Juco vs. Heirs of Tomas Siy Chung Fu,).

Juco said:

“As pointed out by the appellate court, an action for

revival of judgment is a new and independent action,

different and distinct from either the recovery of property

case or the reconstitution case, wherein the cause of action

is the decision itself and not the merits of the action upon

which the judgment sought to be enforced is rendered.

However, revival of judgment is premised on the

assumption that the decision to be revived, either by motion

or by independent action, is already final and executory.

Hence, the need to make a determination of whether or not

the decision in Civil Case No. 7281 has indeed become

final and executory. For if the subject decision has already

reached finality, then the conclusion of the appellate court

is correct that the dismissal of the reconstitution case would

not prevent respondents from reviving and thereafter

executing the said decision.

A decision issued by a court is final and executory

when such decision disposes of the subject matter in its

entirety or terminates a particular proceeding or action,

leaving nothing else to be done but to enforce by execution

what has been determined by the court, such as when after

the lapse of the reglementary period to appeal, no appeal

has been perfected. In the case at bar, it is an undisputed

fact that when the records of the original case were

destroyed in the fire there was a pending motion for

reconsideration of the disapproval of the record on appeal

filed by petitioner. A motion for reconsideration has the

effect of suspending the statutory period after which an

order, decision, or judgment, in connection with which said

motion was filed, becomes final. In effect, such motion for

reconsideration has prevented the decision from attaining

finality.

The findings of the Court of Appeals that

notwithstanding the pendency of the motion for

reconsideration, the decision in Civil Case No. 7281 has

become final and executory by reason of laches cannot be

sustained. As discussed above, the doctrine of laches

cannot operate to lend finality to the decision since

petitioner’s failure to pursue the motion for reconsideration

was not due to her negligence or abandonment, but was

rather brought upon by the dismissal of the reconstitution

case. Therefore, it is clear that the case has not reached

finality at the time the records of the case were burnt.”

---0---

APPEAL

A. Right to appeal is a statutory privilege

The right to appeal is a statutory privilege and of statutory origin,

not a constitutional, natural or inherent right. Therefore, available only if

granted or as provided by statutes. It may be exercised only in the manner

prescribed by the provisions of the law. The period to appeal is

specifically governed by Section 39 of Batas Pambansa Blg. 129 (BP

129), as amended amd Section 3 of Rule 41 of the 1997 Rules of Civil

Procedure (Yu vs. Judge Samson-Tatad,).

B. When to appeal?

The period for appeal from final orders, resolutions, awards,

judgments, or decisions of any court in all cases shall be fifteen (15) days

counted from the notice of the final order, resolution, award, judgment, or

decision appealed from (Section 39, BP 129, as amended).

Under Section 39 of Batas Pambansa Bilang (B.P. Blg.) 129, the

petitioners had fifteen (15) days within which to file their notice of appeal,

from the time their counsel received notice or was served a copy of the

trial court’s decision. The fifteen-day period provided therein is mandatory

and jurisdictional. It bears stressing that the right to appeal is not a natural

right or a part of due process. It is a procedural remedy of statutory origin

and, as such, may be exercised only in the manner and within the time

frame provided by the provisions of law authorizing its exercise. Failure

of a party to perfect an appeal within the period fixed by law renders the

decision sought to be appealed final and executory. As a result, no court

could exercise appellate jurisdiction to review the decision. After a

decision is declared final and executory, vested rights are acquired by the

winning party who has the right to enjoy the finality of the case (Alon vs.

CA, ).

Section 39, BP 129, as amended, is the substantive source of the

remedy of appeal, while Section 3, Rule 41 of the 1997 Rules of Civil

Procedure is the procedural source.

In Ortigas and Company Limited Partnership v. Velasco, this Court

expounded on the interpretation of certain phrases used by the courts in

their judgments or resolutions –

C. Denial of motion for reconsideration with words “with finality”

is of no consequence

While the denial of a motion for reconsideration of a judgment or

final order is normally accompanied by the modifier, “final,” or “with

finality,” there may be a denial not so qualified. That is of no

consequence. By no means may it be taken as indicating any uncertainty

or indecisiveness on the part of the Court regarding its denial of

reconsideration, or an encouragement or expectation of a second motion

for reconsideration. The modifier serves simply to emphasize the import

and effect of the denial of the motion for reconsideration, i.e., that the

Court will entertain and consider no further arguments or submissions

from the parties respecting its correctness; that in the Court’s considered

view, nothing more is left to be discussed, clarified or done in the case, all

issues raised having been passed upon and definitely resolved, and any

other which could have been raised having been waived and no longer

being available as ground for a second motion. A denial with finality

stresses that the case is considered closed (Ortigas and Company Limited

Partnership v. Velasco,).

APPEALS

FROM FIRST TO SECOND LEVEL COURT

A. How, where and when to appeal?

The appeal is taken by filing a notice of appeal with the court that

rendered the judgment or final order appealed from. The notice of appeal

shall indicate the parties to the appeal, the judgment or final order or part

thereof appealed from, and state the amterial dates showing the timeliness

of the appeal (Section 3, Rule 40, 1997 Revised Rules of Civil

Procedure).

An appeal from a judgment or final order of a Municipal Trial

Court may be taken to the Regional Trial Court exercising jurisdiction

over the area to which the former pertains. The title of the case shall

remain as it was in the court of origin, but the party appealing the case

shall be further referred to as the appellant and the adverse party as the

appellee (Section 1, Rule 40, 1997 Revised Rules of Civil Procedure).

An appeal may be taken within fifteen (15) days after notice to the

appellant of the judgment or final order appealed from. Where a record on

appeal is required, the appellant shall file a notice of appeal and a record

on appeal within thirty (30) days after notice of the judgment or final order

(Section 2, Rule 40, 1997 Revised Rules of Civil Procedure).

The period of appeal shall be interrupted by a timely motion for

new trial or reconsideration. No motion for extension of time to file a

motion for new trial or reconsideration shall be allowed (Paragraph 2,

Section 2, Rule 40, 1997 Revised Rules of Civil Procedure).

B. Perfection of appeals and its effects

According to Section 4 of Rule 40, 1997 Revised Rules of Civil

Procedure, the perfection of the appeal and the effect thereof shall be

governed by the provisions of Section 9, Rule 41.

Section 4 of Rule 40, 1997 Revised Rules of Civil Procedure says:

Section 9. Perfection of appeal; effect thereof. — A

party's appeal by notice of appeal is deemed perfected as to

him upon the filing of the notice of appeal in due time.

A party's appeal by record on appeal is deemed

perfected as to him with respect to the subject matter

thereof upon the approval of the record on appeal filed in

due time.

In appeals by notice of appeal, the court loses

jurisdiction over the case upon the perfection of the appeals

filed in due time and the expiration of the time to appeal of

the other parties.

In appeals by record on appeal, the court loses

jurisdiction only over the subject matter thereof upon the

approval of the records on appeal filed in due time and the

expiration of the appeal of the other parties.

In either case, prior to the transmittal of the original

record or the record on appeal, the court may issue orders

for the protection and preservation of the rights of the

parties which do not involve any matter litigated by the

appeal, approve compromises, permit appeals of indigent

litigants, order execution pending appeal in accordance

with 2 of Rule 39, and allow withdrawal of the appeal.

C. Payment of appellate and another lawful fees is a requirement

The general rule is that appeal is perfected by filing a notice of

appeal and paying the requisite docket fees and other lawful fees

(Baniqued v. Ramos, cited in Tanenglian vs.. Lorenzo, G.R. No. 173415,

March 28, 2008).

Within the period for taking an appeal, the appellant shall pay to

the clerk of the court which rendered the judgment or final order appealed

from the full amount of the appellate court docket and other lawful fees.

Proof of payment thereof shall be transmitted to the appellate court

together with the original record or the record on appeal, as the case may

be (Section 5, Rule 40, 1997 Revised Rules of Civil Procedure).

D. Duty of clerk of court of the first level court upon perfection of

appeal

Within fifteen (15) days from the perfection of the appeal, the clerk

of court or the branch clerk of court of the lower court shall transmit the

original record or the record on appeal, together with the transcripts and

exhibits, which he shall certify as complete, to the proper Regional Trial

Court. A copy of his letter of transmittal of the records to the appellate

court shall be furnished the parties (Section 5, Rule 40, 1997 Revised

Rules of Civil Procedure).

E. Duty of clerk of court of the second level court (appellate court)

upon receipt of record

Upon receipt of the complete record or the record on appeal, the

clerk of court of the Regional Trial Court shall notify the parties of such

fact (Section 7 (a), Rule 40, 1997 Revised Rules of Civil Procedure).

Within fifteen (15) days from such notice, it shall be the duty of

the appellant to submit a memorandum which shall briefly discuss the

errors imputed to the lower court, a copy of which shall be furnished by

him to the adverse party. Within fifteen (15) days from receipt of the

appellant's memorandum, the appellee may file his memorandum. Failure

of the appellant to file a memorandum shall be a ground for dismissal of

the appeal (Section 7 (b), Rule 40, 1997 Revised Rules of Civil

Procedure).

Upon the filing of the memorandum of the appellee, or the

expiration of the period to do so, the case shall be considered submitted

for decision. The Regional Trial Court shall decide the case on the basis of

the entire record of the proceedings had in the court of original and such

memoranda as are filed (Section 7 (c), Rule 40, 1997 Revised Rules of

Civil Procedure).

F. Appeal of dismissed case without trial or for lack of jurisdiction

If an appeal is taken from an order of the lower court dismissing

the case without a trial on the merits, the Regional Trial Court may affirm

or reverse it, as the case may be. In case of affirmance and the ground of

dismissal is lack of jurisdiction over the subject matter, the Regional Trial

Court, if it has jurisdiction thereover, shall try the case on the merits as if

the case was originally filed with it. In case of reversal, the case shall be

remanded for further proceedings (Section 8, Rule 40, 1997 Revised Rules

of Civil Procedure).

If the case was tried on the merits by the lower court without

jurisdiction over the subject matter, the Regional Trial Court on appeal

shall not dismiss the case if it has original jurisdiction thereof, but shall

decide the case in accordance with the preceding section, without

prejudice to the admission of amended pleadings and additional evidence

in the interest of justice (Paragraph 2, Section 8, Rule 40, 1997 Revised

Rules of Civil Procedure).

APPEALS FROM SECOND LEVEL COURT

TO COURT OF APPEALS

A. How and when to appeal?

The appeal is taken by filing a notice of appeal with the court that

rendered the judgment or final order appealed from. The notice of appeal

shall indicate the parties to the appeal, the judgment or final order or part

thereof appealed from, and state the amterial dates showing the timeliness

of the appeal (Section 3, Rule 40, 1997 Revised Rules of Civil

Procedure).

The appeal shall be taken within fifteen (15) days from notice of

the judgment or final order appealed from. Where a record on appeal is

required, the appellant shall file a notice of appeal and a record on appeal

within thirty (30) days from notice of the judgment or final order (Section

3, Rule 41, 1997 Revised Rules of Civil Procedure).

The period of appeal shall be interrupted by a timely motion for

new trial or reconsideration. No motion for extension of time to file a

motion for new trial or reconsideration shall be allowed (Paragraph 2,

Section 3, Rule 41, 1997 Revised Rules of Civil Procedure).

The notice of appeal shall indicate the parties to the appeal, specify

the judgment or final order or part thereof appealed from, specify the court

to which the appeal is being taken, and state the material dates showing

the timeliness of the appeal (Section 5, Rule 41, 1997 Revised Rules of

Civil Procedure).

B. Subject of appeal

An appeal may be taken from a judgment or final order that

completely disposes of the case, or of a particular matter therein when

declared by these Rules to be appealable (Section 1, Rule 41, 1997

Revised Rules of Civil Procedure).

C. Appeal is not permitted

No appeal may be taken from:

(a) An order denying a motion for new trial or reconsideration;

(b) An order denying a petition for relief or any similar motion

seeking relief from judgment;

(c) An interlocutory order;

(d) An order disallowing or dismissing an appeal;

(e) An order denying a motion to set aside a judgment by

consent, confession or compromise on the ground of fraud,

mistake or duress, or any other ground vitiating consent;

(f) An order of execution;

(g) A judgment or final order for or against one or more of

several parties or in separate claims, counterclaims, cross-

claims and third-party complaints, while the main case is

pending, unless the court allows an appeal therefrom; and

(h) An order dismissing an action without prejudice.

In all the above instances where the judgment or final order is not

appealable, the aggrieved party may file an appropriate special civil action

under Rule 65 (Paragraph 2, Section 1, Rule 41, 1997 Revised Rules of

Civil Procedure).

D. The modes of appeal

The appeal to the Court of Appeals in cases decided by the

Regional Trial Court in the exercise of its original jurisdiction shall be

taken by filing a notice of appeal with the court which rendered the

judgment or final order appealed from and serving a copy thereof upon the

adverse party. No record on appeal shall be required except in special

proceedings and other cases of multiple or separate appeals where law on

these Rules so require. In such cases, the record on appeal shall be filed

and served in like manner (Section 2 (a), Rule 41, 1997 Revised Rules of

Civil Procedure).

This is the so-called “ordinary appeal”.

The appeal to the Court of Appeals in cases decided by the

Regional Trial Court in the exercise of its appellate jurisdiction shall be by

petition for review in accordance with Rule 42 (Section 2 (b), Rule 41,

1997 Revised Rules of Civil Procedure).

This is the so-called “petition for review”.

In all cases where only questions of law are raised or involved, the

appeal shall be to the Supreme Court by petition for review on certiorari

in accordance with the Rule 45 (Section 2 (c), Rule 41, 1997 Revised

Rules of Civil Procedure).

This is the so-called “appeal by certiorari”.

E. Perfection of appeal and its effect

A party's appeal by notice of appeal is deemed perfected as to him

upon the filing of the notice of appeal in due time (Section 9, Rule 41,

1997 Revised Rules of Civil Procedure).

A party's appeal by record on appeal is deemed perfected as to him

with respect to the subject matter thereof upon the approval of the record

on appeal filed in due time (Paragraph 2, Section 9, Rule 41, 1997

Revised Rules of Civil Procedure).

In appeals by notice of appeal, the court loses jurisdiction over the

case upon the perfection of the appeals filed in due time and the expiration

of the time to appeal of the other parties (Paragraph 3, Section 9, Rule

41, 1997 Revised Rules of Civil Procedure).

In appeals by record on appeal, the court loses jurisdiction only

over the subject matter thereof upon the approval of the records on appeal

filed in due time and the expiration of the appeal of the other parties

(Paragraph 4, Section 9, Rule 41, 1997 Revised Rules of Civil

Procedure).

In either case, prior to the transmittal of the original record or the

record on appeal, the court may issue orders for the protection and

preservation of the rights of the parties which do not involve any matter

litigated by the appeal, approve compromises, permit appeals of indigent

litigants, order execution pending appeal in accordance with 2 of Rule 39,

and allow withdrawal of the appeal (Paragraph 5, Section 9, Rule 41,

1997 Revised Rules of Civil Procedure).

F. Duty of clerk of court of the lower court upon perfection of

appeal

Within thirty (30) days after perfection of all the appeals in

accordance with the preceding section, it shall be the duty of the clerk of

court of the lower court:

(a) To verify the correctness of the original record or the record on

appeal, as the case may be aid to make certification of its

correctness;

(b) To verify the completeness of the records that will be,

transmitted to the appellate court;

(c) If found to be incomplete, to take such measures as may be

required to complete the records, availing of the authority that he

or the court may exercise for this purpose; and

(d) To transmit the records to the appellate court.

If the efforts to complete the records fail, he shall indicate in his

letter of transmittal the exhibits or transcripts not included in the records

being transmitted to the appellate court, the reasons for their non-

transmittal, and the steps taken or that could be taken to have them

available.

The clerk of court shall furnish the parties with copies of his letter

of transmittal of the records to the appellate court (Section 10, Rule 41,

1997 Revised Rules of Civil Procedure).

Upon the perfection of the appeal, the clerk shall immediately

direct the stenographers concerned to attach to the record of the case five

(5) copies of the transcripts of the testimonial evidence referred to in the

record on appeal. The stenographers concerned shall transcribe such

testimonial evidence and shall prepare and affix to their transcripts an

index containing the names of the witnesses and the pages wherein their

testimonies are found, and a list of the exhibits and the pages wherein each

of them appears to have been offered and admitted or rejected by the trial

court. The transcripts shall be transmitted to the clerk of the trial court

who shall thereupon arrange the same in the order in which the witnesses

testified at the trial, and shall cause the pages to be numbered

consecutively (Section 10, Rule 41, 1997 Revised Rules of Civil

Procedure).

The clerk of the trial court shall transmit to the appellate court the

original record or the approved record on appeal within thirty (30) days

from the perfection of the appeal, together with the proof of payment of

the appellate court docket and other lawful fees, a certified true copy of

the minutes of the proceedings, the order of approval, the certificate of

correctness, the original documentary evidence referred to therein, and the

original and three (3) copies of the transcripts. Copies of the transcripts

and certified true copies of the documentary evidence shall remain in the

lower court for the examination of the parties (Section 12, Rule 41, 1997

Revised Rules of Civil Procedure).

G. Payment of appellate and another lawful fees is a requirement

Within the period for taking an appeal, the appellant shall pay to

the clerk of the court which rendered the judgment or final order appealed

from the full amount of the appellate court docket and other lawful fees.

Proof of payment thereof shall be transmitted to the appellate court

together with the original record or the record on appeal, as the case may

be (Section 5, Rule 40, 1997 Revised Rules of Civil Procedure).

H. Duty of clerk of court of the first level court upon perfection of

appeal

I. Record on appeal; form and contents

Again, the Notice of Appeal shall indicate the parties to the appeal,

specify the judgment or final order or part thereof appealed from, specify

the court to which the appeal is being taken, and state the material dates

showing the timeliness of the appeal (Section 5, Rule 41, 1997 Revised

Rules of Civil Procedure).

The record on appeal shall contain the full names of all the parties

to the proceedings shall be stated in the caption of the record on appeal

and it shall include the judgment or final order from which the appeal is

taken and, in chronological order, copies of only such pleadings, petitions,

motions and all interlocutory orders as are related to the appealed

judgment or final order for the proper understanding of the issue involved,

together with such data as will show that the appeal was perfected on time.

If an issue of fact is to be raised on appeal, the record on appeal shall

include by reference all the evidence, testimonial and documentary, taken

upon the issue involved. The reference shall specify the documentary

evidence by the exhibit numbers or letters by which it was identified when

admitted or offered at the hearing, and the testimonial evidence by the

names of the corresponding witnesses. If the whole testimonial and

documentary evidence in the case is to be included, a statement to that

effect will be sufficient without mentioning the names of the witnesses or

the numbers or letters of exhibits. Every record on appeal exceeding

twenty (20) pages must contain a subject index (Section 6, Rule 41, 1997

Revised Rules of Civil Procedure).

Copies of the notice of appeal and the record on appeal where

required, shall be served on the adverse party (Paragraph 4, Section 3,

Rule 40, 1997 Revised Rules of Civil Procedure).

J. Approval of record on appeal; and when both parties are

appellants

Upon the filing of the record on appeal for approval and if no

objection is filed by the appellee within five (5) days from receipt of a

copy thereof, the trial court may approve it as presented or upon its own

motion or at the instance of the appellee, may direct its amendment by the

inclusion of any omitted matters which are deemed essential to the

determination of the issue of law or fact involved in the appeal. If the trial

court orders the amendment of the record, the appellant, within the time

limited in the order, or such extension thereof as may be granted, or if no

time is fixed by the order within ten (10) days from receipt thereof, shall

redraft the record by including therein, in their proper chronological

sequence, such additional matters as the court may have directed him to

incorporate, and shall thereupon submit the redrafted record for approval,

upon notice to the appellee, in like manner as the original draft (Section 7,

Rule 41, 1997 Revised Rules of Civil Procedure).

Where both parties are appellants, they may file a joint record on

appeal within the time fixed by section 3 of this Rule, or that fixed by the

court (Section 8, Rule 41, 1997 Revised Rules of Civil Procedure).

K. Payment of appellate and another lawful fees is a requirement

Within the period for taking an appeal, the appellant shall pay to

the clerk of the court which rendered the judgment or final order appealed

from, the full amount of the appellate court docket and other lawful fees.

Proof of payment of said fees shall be transmitted to the appellate court

together with the original record or the record on appeal (Section 4, Rule

41, 1997 Revised Rules of Civil Procedure).

The appellate docket and other lawful fees must be paid within the

same period for taking an appeal (Enriquez vs. Enriquez, G.R. No.

139303, August 25, 2005).

The High Court in Enriquez vs. Enriquez, G.R. No. 139303,

August 25, 2005, said:

“Prior to the effectivity of the 1997 Rules of Civil

Procedure, as amended, payment of appellate court docket

fee is not a prerequisite for the perfection of an appeal. In

Santos vs. Court of Appeals, this Court held that although

an appeal fee is required to be paid in case of an appeal

taken from the Municipal Trial Court to the Regional Trial

Court, it is not a prerequisite for the perfection of an appeal

under Sections 20 and 23 of the Interim Rules and

Guidelines issued by this Court on January 11, 1983

implementing the Judiciary Reorganization Act of 1981

(B.P. Blg. 129). Under these sections, there are only two

requirements for the perfection of an appeal, to wit: (a) the

filing with the trial court of a notice of appeal within the

reglementary period; and (b) the expiration of the last day

to appeal by any party.

However, the 1997 Rules of Civil Procedure, as

amended, which took effect on July 1, 1997, now require

that appellate docket and other lawful fees must be paid

within the same period for taking an appeal. This is clear

from the opening sentence of Section 4, Rule 41 of the

same Rules that, “(W)ithin the period for taking an appeal,

the appellant shall pay to the clerk of the court which

rendered the judgment or final order appealed from, the full

amount of the appellate court docket and other lawful fees.”

The use of the word “shall” underscores the

mandatory character of the Rule. The term “shall” is a

word of command, and one which has always or which

must be given a compulsory meaning, and it is generally

imperative or mandatory. Petitioners cannot give a

different interpretation to the Rule and insist that payment

of docket fee shall be made only upon their receipt of a

notice from the trial court to pay. For it is a rule in

statutory construction that every part of the statute must be

interpreted with reference to the context, i.e., that every

part of the statute must be interpreted together with the

other parts, and kept subservient to the general intent of the

whole enactment. Indeed, petitioners cannot deviate from

the Rule.

Also under Rule 41 of the same Rules, an appeal to

the Court of Appeals from a case decided by the RTC in the

exercise of the latter’s original jurisdiction, shall be taken

within fifteen (15) days from the notice of judgment or

final order appealed from. Such appeal is made by filing a

notice thereof with the court that rendered the judgment or

final order and by serving a copy of that notice upon the

adverse party. Furthermore, within this same period,

appellant shall pay to the clerk of court which rendered the

judgment or final order appealed from, the full amount of

the appellate court docket and other lawful fees. The

payment of docket fee within this period is mandatory for

the perfection of appeal. Otherwise, the appellate court

would not be able to act on the subject matter of the action,

and the decision sought to be appealed from becomes final

and executory.

Time and again, this Court has consistently held that

payment of docket fee within the prescribed period is

mandatory for the perfection of an appeal. Without such

payment, the appellate court does not acquire jurisdiction

over the subject matter of the action and the decision

sought to be appealed from becomes final and executory.”

The appeal shall be taken within fifteen (15) days from notice of

the judgment or final order appealed from. Where a record on appeal is

required, the appellant shall file a notice of appeal and a record on appeal

within thirty (30) days from notice of the judgment or final order (Section

3, Rule 41, 1997 Rules of Civil Procedure).

The period of appeal shall be interrupted by a timely motion for

new trial or reconsideration. No motion for extension of time to file a

motion for new trial or reconsideration shall be allowed (Paragraph 2,

Section 3, Rule 41, 1997 Rules of Civil Procedure).

L. Failure to pay the appellate docket fee within the reglementary

period bestows on the appellate court a directory power to

dismiss an appeal

The appellant’s failure to pay the appellate docket fee within the

fifteen-day reglementary period bestows on the appellate court a directory,

not a mandatory, power to dismiss an appeal (Fontanar vs. Bonsubre).

The High Court’s explanation in the Badillo consolidated cases,

thus:

“We agree with the RTC that, insofar as appeals from

the MTC to the RTC are concerned, the 1997 Rules of Civil

Procedure do not mandate the dismissal of an appeal as a

consequence of the nonpayment of the required fee.

Martinez v. Court of Appeals holds that in such

appeals, “the failure to pay the appellate docket fees does

not automatically result in the dismissal of the appeal, the

dismissal being discretionary on the part of the appellate

court.” While that case was governed by Sections 20 and

23 of the Interim Rules and Guidelines issued by the Court

on January 11, 1983 to implement the Judiciary

Reorganization Act of 1981 (BP Blg. 129), the present

Rules lead to a similar conclusion.

Under the 1997 Rules of Civil Procedure, parties

perfect an appeal from the judgment of the MTC to the

RTC by filing a notice of appeal within the fifteen day

reglementary period, as provided under Section 4 of Rule

40 and Section 9 of Rule 41:

Rule 40 --

“SEC. 4. Perfection of appeal; effect thereof. – The

perfection of the appeal and the effect thereof shall be

governed by the provisions of section 9, Rule 41.

Rule 41--

“SEC. 9. Perfection of appeal; effect thereof. - A party’s

appeal by notice of appeal is deemed perfected as to him

upon filing of the notice of appeal in due time.

xxx xxx xxx

“In appeals by notice of appeal, the court loses

jurisdiction over the case upon the perfection of the appeals

filed in due time and the expiration of the time to appeal of

the other party.”

Fontanar v. Bonsubre is a case in point. It holds that

in appeals from the MTC to the RTC, failure to pay the

appellate docket fee within the fifteen-day reglementary

period bestows on the appellate court a directory, not a

mandatory, power to dismiss an appeal. The Court

ratiocinated as follows:

“x x x [T]his Court restated the importance

and real purpose of the remedy of appeal as an

essential part of our judicial system and advised

the courts to proceed with caution so as not to

deprive a party of a right to appeal with the

instruction that every party-litigant should be

afforded the amplest opportunity for the proper

and just disposition if his cause, freed from the

constraints of technicalities. Rightly so, for the

payment of the appellate docket fee is not a

requirement for the protection of the prevailing

party, and non-compliance therewith within the

time prescribed causes no substantial prejudice

to anyone.”

On the other hand, the cases cited by petitioners

involve appeals -- not from the MTC to the RTC -- but

from the RTC to the CA and from the CA to the SC, for

which the payment of appellate fees is indeed mandatory

according to the Rules. We quote Manalili v. Arsenio and

De Leon:

“Appeal is not a right, but a mere statutory

privilege. Corollary to this principle is that the

appeal must be exercised strictly in accordance

with provisions set by law. x x x

“x x x [T]he payment of the appellate

docket fee is not a mere technicality of law or

procedure. It is an essential requirement,

without which the decision or final order

appealed from would become final and

executory as if no appeal was filed at all.”

In the instant cases, when the NHA filed a Notice of

Appeal on February 22, 2000 -- two days before the appeal

period lapsed – it perfected its appeal and the MTC thereby

lost its jurisdiction. The MTC therefore acted without

jurisdiction in issuing the May 23, 2000 Order and the May

30, 2000 Writ of Execution.”

-0-0-0-

A notice of appeal was timely filed but the appeal docket fee was

paid after three months when the notice of appeal because of the

messenger’s inadvertence in securing postal money order. The trial court

approved the notice of appeal basing the order in the phrase “in the

interest of substantial justice.”

Is the trial court correct?

No. The bare invocation of the phrase “in the interest of

substantial justice” is not a magic spell that will automatically allow the

court to suspend procedural rules, despite jurisdictional bar. The rule may

be relaxed only in exceptionally meritorious cases. The messenger’s

alleged inadvertence to secure a postal money order for appellate docket

fees is not a meritorious reason to justify as exception in our jurisprudence

(Ilusorio vs. Ilusorio-Yap,). The finality of a decision is a jurisdictional

event which cannot be made to depend on the convenience of the parties

(Ocampo vs. CA,).

M. Appellate court may motu proprio dismiss the appeal

Prior to the transmittal of the original record or the record on

appeal to the appellate court, the trial court may motu propio or on motion

dismiss the appeal for having been taken out of time (Section 13, Rule 41,

1997 Revised Rules of Civil Procedure).

N. A final judgment or order is one that finally disposes of a case

A final judgment or order is one that finally disposes of a case,

leaving nothing more for the court to do in respect thereto, such as an

adjudication on the merits which, on the basis of the evidence presented at

the trial, declares categorically what the rights and obligations of the

parties are and which party is in the right, or a judgment or order that

dismisses an action on the ground of res judicata or prescription, for

instance (Intramuros Tennis Club, Inc. vs. Philippine Tourism

Authority,).

O. Questions of fact and questions of law distinguished

A question of law arises when there is doubt as to what the law is

on a certain state of facts while there is a question of fact when the doubt

arises as to the truth or falsity of the alleged facts. A question of law may

be resolved by the court without reviewing or evaluating the evidence. No

examination of the probative value of the evidence would be necessary to

resolve a question of law. The opposite is true with respect to questions of

fact, which necessitate a calibration of the evidence (Macababbad vs.

Masirag,).

The nature of the issues to be raised on appeal can be gleaned from

the appellant’s notice of appeal filed in the trial court and in his or her

brief as appellant in the appellate court. In their Notice of Appeal, the

respondents manifested their intention to appeal the assailed RTC order on

legal grounds and “on the basis of the environmental facts.” Further, in

their Brief, the petitioners argued that the RTC erred in ruling that their

cause of action had prescribed and that they had “slept on their rights.” All

these indicate that questions of facts were involved, or were at least raised,

in the respondents’ appeal with the CA.

P. Issue on prescription may be a question of fact or question of

law

Issue on prescription may either be a question of law or fact; it is a

question of fact when the doubt or difference arises as to the truth or

falsity of an allegation of fact; it is a question of law when there is doubt

or controversy as to what the law is on a given state of facts. The test of

whether a question is one of law or fact is not the appellation given to the

question by the party raising the issue; the test is whether the appellate

court can determine the issue raised without reviewing or evaluating the

evidence. Prescription, evidently, is a question of fact where there is a

need to determine the veracity of factual matters such as the date when the

period to bring the action commenced to run (Crisostomo vs. Garcia, G.R.

No. 164787, January 31, 2006 cited in Macababbad vs. Masirag, G.R.

No. 161237, January 14, 2009).

The High Court in Macababbad said:

“Ingjug-Tiro v. Casals instructively tells us too that a

summary or outright dismissal of an action is not proper

where there are factual matters in dispute which require

presentation and appreciation of evidence. In this cited case

whose fact situation is similar to the present case, albeit

with a very slight and minor variation, we considered the

improvident dismissal of a complaint based on prescription

and laches to be improper because the following must still

be proven by the complaining parties:

first, that they were the co-heirs and co-owners

of the inherited property; second, that their co-

heirs-co-owners sold their hereditary rights

thereto without their knowledge and consent;

third, that forgery, fraud and deceit were

committed in the execution of the Deed of

Extrajudicial Settlement and Confirmation of

Sale since Francisco Ingjug who allegedly

executed the deed in 1967 actually died in 1963,

hence, the thumbprint found in the document

could not be his; fourth, that Eufemio Ingjug

who signed the deed of sale is not the son of

Mamerto Ingjug, and, therefore, not an heir

entitled to participate in the disposition of the

inheritance; fifth, that respondents have not paid

the taxes since the execution of the sale in 1965

until the present date and the land in question is

still declared for taxation purposes in the name

of Mamerto Ingjug, the original registered

owner, as of 1998; sixth, that respondents had

not taken possession of the land subject of the

complaint nor introduced any improvement

thereon; and seventh, that respondents are not

innocent purchasers for value.

As in Ingjug-Tiro, the present case involves factual

issues that require trial on the merits. This situation rules

out a summary dismissal of the complaint.”

Proper Mode of Appeal

Since the appeal raised mixed questions of fact and

law, no error can be imputed on the respondents for

invoking the appellate jurisdiction of the CA through an

ordinary appeal. Rule 41, Sec. 2 of the Rules of Court

provides:

Modes of appeal.

(a) Ordinary appeal - The appeal to the Court

of Appeals in cases decided by the

Regional Trial Court in the exercise of its

original jurisdiction shall be taken by

filing a notice of appeal with the court

which rendered the judgment or final order

appealed from and serving a copy thereof

upon the adverse party.

In Murillo v. Consul, this Court had the occasion to

clarify the three (3) modes of appeal from decisions of the

RTC, namely: (1) ordinary appeal or appeal by writ of

error, where judgment was rendered in a civil or criminal

action by the RTC in the exercise of original jurisdiction,

covered by Rule 41; (2) petition for review, where

judgment was rendered by the RTC in the exercise of

appellate jurisdiction, covered by Rule 42; and (3) petition

for review to the Supreme Court under Rule 45 of the Rules

of Court. The first mode of appeal is taken to the CA on

questions of fact or mixed questions of fact and law. The

second mode of appeal is brought to the CA on questions of

fact, of law, or mixed questions of fact and law. The third

mode of appeal is elevated to the Supreme Court only on

questions of law.”

Q. Perfected is the appeal by filing a notice of appeal and paying

the requisite docket fees, exception

The general rule is that appeal is perfected by filing a notice of

appeal and paying the requisite docket fees and other lawful fees, but

admits an exception (Tanenglian vs. Lorenzo,).

Tanenglian said:

“However, all general rules admit of certain

exceptions. In Mactan Cebu International Airport

Authority v. Mangubat where the docket fees were paid six

days late, we said that where the party showed willingness

to abide by the rules by immediately paying the required

fees and taking into consideration the importance of the

issues raised in the case, the same calls for judicial

leniency, thus:

In all, what emerges from all of the above

is that the rules of procedure in the matter of

paying the docket fees must be followed.

However, there are exceptions to the stringent

requirement as to call for a relaxation of the

application of the rules, such as: (1) most

persuasive and weighty reasons; (2) to relieve a

litigant from an injustice not commensurate with

his failure to comply with the prescribed

procedure; (3) good faith of the defaulting party

by immediately paying within a reasonable time

from the time of the default; (4) the existence of

special or compelling circumstances; (5) the

merits of the case; (6) a cause not entirely

attributable to the fault or negligence of the

party favored by the suspension of the rules; (7)

a lack of any showing that the review sought is

merely frivolous and dilatory; (8) the other party

will not be unjustly prejudiced thereby; (9)

fraud, accident, mistake or excusable negligence

without appellant’s fault; (10) peculiar legal and

equitable circumstances attendant to each case;

(11) in the name of substantial justice and fair

play; (12) importance of the issues involved;

and (13) exercise of sound discretion by the

judge guided by all the attendant circumstances.

Concomitant to a liberal interpretation of the

rules of procedure should be an effort on the

part of the party invoking liberality to

adequately explain his failure to abide by the

rules. Anyone seeking exemption from the

application of the Rule has the burden of

proving that exceptionally meritorious instances

exist which warrant such departure.

We have not been oblivious to or unmindful of the

extraordinary situations that merit liberal application of the

Rules, allowing us, depending on the circumstances, to set

aside technical infirmities and give due course to the

appeal. In cases where we dispense with the technicalities,

we do not mean to undermine the force and effectivity of

the periods set by law. In those rare cases where we did not

stringently apply the procedural rules, there always existed

a clear need to prevent the commission of a grave injustice.

Our judicial system and the courts have always tried to

maintain a healthy balance between the strict enforcement

of procedural laws and the guarantee that every litigant be

given the full opportunity for the just and proper

disposition of his cause. If the Highest Court of the land

itself relaxes its rules in the interest of substantive justice,

then what more the administrative bodies which exercise

quasi-judicial functions? It must be emphasized that the

goal of courts and quasi-judicial bodies, above else, must

be to render substantial justice to the parties.

In this case, petitioner was only one day late in paying

the appeal fee, and he already stands to lose his titles to the

subject properties. We find this too harsh a consequence for

a day’s delay. Worthy to note is the fact that petitioner

actually paid the appeal fee; only, he was a day late. That

petitioner immediately paid the requisite appeal fee a day

after the deadline displays his willingness to comply with

the requirement therefor.”

---0---

MOTION FOR RECONSIDERATION

(Rule 37)

A. Two-fold nature of motion for reconsideration, purpose and

importance

As a rule, not all orders denying or granting motions for

reconsideration are final orders. It depends upon the kind of incident to

which the motion for reconsideration is filed or applied. If it is filed

arising out of an interlocutory order, let say for instance, against a motion

for bill of particulars, the order granting or denying the same is

interlocutory in character, because the court has still something to do with

the case. It does not finally dispose of the case.

If it is filed against a judgment, the order granting or denying the

same is in the nature of a final order. This is the so-called the two-fold

functional character of a motion for reconsideration – the interlocutory

and final, depending upon its usage and applicability.

The importance of knowing whether an order granting or denying a

motion for reconsideration is to determine what will be the next applicable

remedy is available to the aggrieved party.

The aggrieved party is not allowed to file a second motion for

reconsideration of a judgment or final order (Paragraph 2, Section 5, rule

37, 1997 Revised Rules of Civil Procedure).

If an aggrieved party files a second motion for reconsideration of a

judgment or final order, thus allowing the period to appeal to lapsed.

Therefore, he shall loose his right to appeal, and the consequence of

which, he cannot resort to the filing of a petition for certiorari under Rule

65, because it is not a substitute for a lost appeal (Zarate vs. Maybank

Philippines, Inc,).

A. Motion for reconsideration that fails to comply with the rules

on motion not toll the period to file appeal

The motion for reconsideration which is filed without compliance

with the rules on motion is treated as a mere scrap of paper and will not

toll the period to file appeal.

In People vs. Court of Appeals, , the High Court held that a motion

without a notice of hearing is pro forma, a mere scrap of paper that does

not toll the period to appeal.

Said the High Court:

“Not only did the defect render the motion for

reconsideration itself unworthy of consideration, it more

crucially failed to toll the period to appeal. A motion

without a notice of hearing is pro forma, a mere scrap of

paper that does not toll the period to appeal, and upon the

expiration of the 15-day period, the questioned order or

decision becomes final and executory. The rationale behind

this rule is plain: unless the movant sets the time and place

of hearing, the court will be unable to determine whether

the adverse party agrees or objects to the motion, and if he

objects, to hear him on his objection, since the rules

themselves do not fix any period within which he may file

his reply or opposition.”

---0---

MOTION FOR NEW TRIAL

(Rule 37)

A. Concept, purpose and its allowance

A new trial is not a refuge for the obstinate.17

New trial is a remedy that seeks to ‘temper the severity of a

judgment or prevent the failure of justice.’ The Rules allows the courts to

grant a new trial when there are errors of law or irregularities prejudicial

to the substantial rights of the accused committed during the trial, or when

there exists newly discovered evidence. (Ybiernas vs. Tanco-Gabaldon,).

B. Grant or denial is discretionary

The grant or denial of a new trial is addressed to the sound

discretion of the court which cannot be interfered with unless a clear abuse

thereof is shown (Ybiernas vs. Tanco-Gabaldon,).

17

Viking Industrial Corporation vs. Court of Appeals, G.R. No. 143794, July 13, 2004

cited in Atlas Consolidated Mining and Development Corporation vs. CIR, G.R. Nos.

141104 & 148763, June 8, 2007.

C. Only on newly discovered evidence, requirements

A new trial may be granted on the ground of newly discovered

evidence, provided that the following must be shown:

(1) that the evidence was discovered after trial;

(2) that such evidence could not have been discovered and

produced at the trial even with the exercise of reasonable

diligence;

(3) that it is material, not merely cumulative, corroborative, or

impeaching; and

(4) that the evidence is of such weight that it would probably

change the judgment if admitted (Custodio vs.

Sandiganbayan,).

If the alleged newly discovered evidence could have been very

well presented during the trial with the exercise of reasonable diligence,

the same cannot be considered newly discovered (Custodio vs.

Sandiganbayan,).

The movant for a new trial must not only act in a timely fashion in

gathering evidence in support of the motion; he must act reasonably and in

good faith as well (Custodio vs. Sandiganbayan,).

Only a final judgment or order may be the subject of a motion for

new trial (Ybiernas vs. Tanco-Gabaldon,).

D. Ignorance, inexperience or incompetence of counsel do not

qualify as a ground for new trial.

Blunders and mistakes in the conduct of the proceedings in the trial

court as a result of the ignorance, inexperience or incompetence of counsel

do not qualify as a ground for new trial. If such were to be admitted as

valid reasons for re-opening cases, there would never be an end to

litigation so long as a new counsel could be employed to allege and show

that the prior counsel had not been sufficiently diligent, experienced or

learned. This will put a premium on the willful and intentional

commission of errors by counsel, with a view to securing new trials in the

event of of conviction, or an adverse decision (Uy vs. First Metro

Integrated Steel Corp,).

E. Remedy of adverse party if motion for new trial is granted by the

trial court.

If a motion for new trial is granted by the trial court, the remedy of

the adverse party if the act of granting the motion is believed to be tainted

with grave abuse of discretion is to file a petition for certiorari under Rule

65 of the 1997 Revised Rules of Civil Procedure, because: first, this is

authorized by Section 1, paragaraph 2 (a) of Rule 41 of the same Rules of

Procedure says that no appeal may be taken from an order denying a

motion for new trial or reconsideration; second, this is because an order

granting a motion for new trial is interlocutory character (See Section 1,

paragraph 2 (c), Rule 41, 1997 Revised Rules of Civil Procedure). It is an

order which does not dispose of the case completely because it leaves

something for the trial court to do (Tan vs. Republic, ), as the original

judgment or final order is vacated (See Section 6, Rule 37, 1997 Revised

Rules of Civil Procedure, Effect of granting of motion for new trial). The

trial court in trial de novo shall re-try the case and to decide it after the re-

trial; and third, it is only when such interlocutory order was rendered

without or in excess of jurisdiction or with grave abuse of discretion that

certiorari under Rule 65 may be resorted to (Silverio, Jr. vs. CA, G.R. No.

178933, September 16, 2009 citing 1 F. Regalado, Remedial Law

Compendium 540 (8th revised ed.).

If it is believed that the act of the trial court in granting the motion

for new trial is not tainted with grave abuse of discretion, the remedy of

the adverse party is to wait for the new judgment to be rendered after trial

de novo, and if the judgment is not favorable, then file a notice of appeal.

Again, this is because an interlocutory order generally cannot be appealed

separately from the judgment. It is only after a judgment has been

rendered in the case that the ground for the appeal of the interlocutory

order may be included in the appeal of the judgment itself (Silverio, Jr. vs.

CA,).

F. Remedy of the aggrieved party if motion for new trial is denied

by the trial court.

The remedy available is not petition for certiorari under Rule 65,

but appeal under Rule 41 of the 1997 Revised Rules of Civil Procedure.

This is because:

First, Section 9 of Rule 37 of the 1997 Revised Rules of Civil

Procedure is very clear on the matter that an order denying a motion for

new trial is not appealable (the order itself denying a motion for new trial

is not appealable), the remedy being an appeal from the judgment or final

order;

Second, an order denying a motion for new trial is no longer an

interlocutory character;

Third, certiorari is only available if there is no appeal, or any plain,

speedy, and adequate remedy in the ordinary course of law (See Rule 65).

Appeal is available.

Lastly, it is a final order, because it decisively puts to a close, or

disposes of a case or a disputed issue leaving nothing else to be done by

the court in respect thereto (Santo Tomas University Hospital vs. Surla,

G).

Section 9, Rule 37 of the Rules of Court provides:

“SEC. 9. Remedy against order denying a motion

for new trial or reconsideration. –An order denying a

motion for new trial or reconsideration is not

appealable, the remedy being an appeal from the

judgment or final order.”

Section 1, Rule 41 of the Rules of Court provides:

“SECTION 1. Subject of appeal. –An appeal may

be taken from a judgment or final order that completely

disposes of the case, or of a particular matter therein

when declared by these Rules to be appealable.

No appeal may be taken from:

(a) An order denying a motion for new

trial or reconsideration;

“xxx

xxx

In all the above instances where the

judgment or final order is not appealable, the

aggrieved party may file an appropriate special

civil action under Rule 65.”

Section 1, Rule 65 of the Rules of Court provides:

“SECTION 1. Petition for certiorari. –When any

tribunal, board or officer exercising judicial or quasi-

judical functions has acted with grave abuse of

discretion amouting to lack or excess of jurisdiction,

and there is no appeal, or any plain, speedy, and

adequate remedy in the course of law, a person

aggrieved thereby may file a verified petition in the

proper court, alleging the facts with certainty and

prayuing that judgment be rendered annulling or

modifying the proceedings of such tribunal, board or

officer, and granting such incidental releifs as law

and justice may require.”

Section 9 of Rule 39, 1997 Revised Rules of Civil Procedure says

that an order denying a motion for new trial or reconsideration is not

appealable, the remedy being an appeal from the judgment or final order,

while, Section 1, paragaraph 2 (a) of Rule 41 of the same Rules of

Procedure says that no appeal may be taken from an order denying a

motion for new trial or reconsideration.

Santo Tomas University Hospital case had the occasion to

distinguish the concepts of a final judgment or order from an interlocutory

issuance, thus: a final judgment or order is that the former decisively puts

to a close, or disposes of a case or a disputed issue leaving nothing else to

be done by the court in respect thereto. Once that judgment or order is

rendered, the adjudicative task of the court is likewise ended on the

particular matter involved;18 and it is interlocutory if its effects would only

be provisional in character and would still leave substantial proceedings to

be further had by the issuing court in order to put the controversy to rest.19

While it is true that Section 9 of Rule 39, 1997 Revised Rules of

Civil Procedure says an order denying a motion for new trial or

reconsideration is not appealable, however, it also says that the remedy is

appeal from the judgment or final order. Therefore, the aggrieved party

should concentrate to the decision or judgment itself after the motion for

new trial is denied for purposes of filing an appeal (via notice of appeal,

See Section 3, Rule 40 on how to appeal) under Rule 41, and not to

elevate the order of denial (of motion for new trial) via certiorari under

Rule 65. Thus, applying Section 9, Rule 37 in relation to Section 1,

paragaraph 2 (a), Rule 41 and Section 1, Rule 65 of the 1997 Rules of

Civil Procedure it is humbly opined that the available remedy to the

aggrieved party is not certiorari under Rule 65, but appeal.

However, the case of Uy vs. First Metro Integrated Steel Corp, is

different. In this case, it was held that if a motion for new trial is denied,

the aggrieved party may resort to the filing of a petition for certiorari

under Rule 65 of the 1997 Revised Rules of Civil Procedure from the

denial of the motion for new trial. Certiorari was allowed by the High

Court because the denial of the motion for new trial is not on the substance

of the motion but on ground of technicality. In that case the trial court

denied the motion for new trial because the same was filed out of time,

18

Santo Tomas University Hospital vs. Surla, G.R. No. 129718, August 17, 1998 citing

Investments, Inc. vs. Court of Appeals, 147 SCRA 334; Denso (Phils.,) Inc. vs.

Intermediate Appellate Court, 148 SCRA 280. 19

Santo Tomas University Hospital vs. Surla, G.R. No. 129718, August 17, 1998 citing

Bairan vs. Tan Siu Lay, 18 SCRA 1235.

and was found out by the High Court that the same was filed within the

reglementary period. According to the High Court, citing Section 1, Rule

41, that certiorari under Rule 65 is applicable because no appeal may be

taken from an order denying a motion for new trial or

reconsideration.

The High Court’s observation in Uy:

“A scrutiny of the records discloses that while the

Motion for New Trial was received by the trial court on

April 28, 2003, the date on the Registry Receipt attached to

the Affidavit of Service as well as that stamped on the

envelope which contained the copy of the motion, reveals

that it was filed and served by registered mail on April 21,

2003, a Monday, because April 19, 2003, the last day for

filing the same was a Saturday. Section 1, Rule 22 of the

Rules of Court states in no uncertain terms that if the last

day of the period thus computed falls on a Saturday, a

Sunday, or a legal holiday in the place where the court sits,

the time shall not run until the next working day. Thus, the

motion was actually filed on time it having been filed on

April 21, 2003, the next working day, following the last day

for filing which fell on a Saturday.

Section 9, Rule 37 of the Rules of Court which provides

that the remedy to an order denying a motion for new trial is

to appeal the judgment or final order, must be read in

conjunction with Section 1, Rule 41 which provides that:

SEC. 1. Subject of appeal. – An appeal

may be taken from a judgment or final order

that completely disposes of the case, or of a

particular matter therein when declared by these

rules to be appealable.

No appeal may be taken from:

(a) An order denying a motion for

new trial or reconsideration;

x x x x

In all the above instances where the

judgment or final order is not appeasable, the

aggrieved party may file an appropriate special

civil action under Rule 65. (Emphasis

supplied)

Thus, the filing by the petitioner of a petition for

certiorari with the Court of Appeals from the denial of the

motion for new trial by the trial court is proper.”

G. Failure of counsel to attend the hearings for the reception of

evidence is inexcusable negligence

In Uy vs. First Metro Integrated Steel Corp, it was held that:

“xxx xxx xxx, we find that the trial court correctly

denied petitioner’s motion for new trial.

Section 1, Rule 37 provides that a motion for new

trial may be filed within the period for taking an appeal

based on the following grounds:

(a) Fraud, accident, mistake or

excusable negligence which ordinary prudence

could not have guarded against and by reason of

which such aggrieved party has probably been

impaired in his rights; or

x x x x

Negligence to be excusable must be one which

ordinary diligence and prudence could not have guarded

against.

In the instant case, we find the negligence of

petitioner’s counsel in failing to attend the hearings for the

reception of evidence inexcusable. The trial court

scheduled the hearing for the reception of petitioner’s

evidence seven times. The initial hearing set on February

28, 2001 was cancelled because petitioner allegedly had

influenza. The hearings scheduled on April 26, 2001 and

May 10, 2001 were cancelled and moved to October 25,

2001 and December 13, 2001. Petitioner was represented

by Atty. Carpio, Jr. as collaborating counsel during the

hearing on October 25, 2001 but no evidence was

presented. Instead, the hearing was cancelled. On

December 13, 2001, Atty. Bañares, petitioner’s new

counsel, appeared but he requested for a resetting. On

February 14, 2002, Atty. Bañares moved to postpone the

hearing to February 28, 2002 as previously scheduled. On

February 28, 2002, Atty. Bañares arrived late.

Scrutiny of the records disclose that the hearings were

postponed or cancelled without any justification. However,

the trial court accommodated the requests for postponement

or resetting in order to accord petitioner due process.

Under the circumstances, we find petitioner’s counsel’s

failure to attend the seven scheduled hearings without

justifiable reason tantamount to inexcusable neglect. As

such, it cannot be a ground for new trial.”

H. Affidavit of merit is required in filing a motion for new trial

Also in Uy vs. First Metro Integrated Steel Corp, , it was held

that:

“In addition, the Rule requires that motions for new

trial founded on fraud, accident, mistake or excusable

negligence must be accompanied by affidavits of merits,

i.e., affidavits showing the facts (not mere conclusions or

opinions) constituting the valid cause of action or defense

which the movant may prove in case a new trial is granted,

because a new trial would serve no purpose and would just

waste the time of the court as well as the parties if the

complaint is after all groundless or the defense is nil or

ineffective.

Under the Rules, the moving party must show that he

has a meritorious defense. The facts constituting the

movant’s good and substantial defense, which he may

prove if the petition were granted, must be shown in the

affidavit which should accompany the motion for a new

trial.20 We examined petitioner’s Affidavit of Merit and

find that it did not contain clear statements of the facts

constituting a good and valid defense which he might prove

if given the chance to introduce evidence. The allegations

that he has a “meritorious defense” and a “good cause” are

mere conclusions which did not provide the court with any

basis for determining the nature and merit of the case. An

affidavit of merit should state facts, and not mere opinion

or conclusions of law. Petitioner’s motion for new trial and

affidavit of merit did not mention the evidence which he

was prevented from introducing, nor did it allege that such

evidence would change the outcome of the case.”

---0---

PETITION FOR RELIEF FROM JUDGMENT (Rule 38)

A. Concept, nature and principles

Section 3, Rule 38 of the 1997 Rules of Civil Procedure provides

that a verified petition for relief must be filed within sixty (60) days after

the petitioner learns of the judgment, final order, or other proceeding to be

set aside and not more than six (6) months after such judgment or final

order has been entered or such proceeding has been taken.

Rule 38 of the Rules of Court only applies when the one deprived

of his right is a party to the case. If the movant is not a party or never been

a party to the case or even summoned to appear in case the remedy of

petition for relief from judgment under Rule 38 of the Rules of Court is

not proper. The word used is “against a party” (Lagula vs. Casimiro,).

A petition for relief from judgment is an exception to the public

policy of immutability of final judgments (Madarang vs. Morales,). It is

premised on equity; an act of grace and not regarded with favor, and

granted only in exceptional cases (Dirige vs. Biranya,), especially to any

person against whom a decision or order is entered into through fraud,

accident, mistake or excusable negligence (Somoso vs. CA,). It is

dismissible outright if filed beyond the reglementary period (Madarang

vs. Morales,). Good and substantial defense are also included as

requirements sine qua non conditions for its proper allowance (Dirige vs.

Biranya,).

When a party has another remedy available to him, which may be

either a motion for new trial or appeal from an adverse decision of the trial

court, and he was not prevented by fraud, accident, mistake or excusable

negligence from filing such motion or taking such appeal, he cannot avail

himself of this petition. Indeed, relief will not be granted to a party who

seeks avoidance from the effects of the judgment when the loss of the

remedy at law was due to his own negligence; otherwise the petition for

relief can be used to revive the right to appeal which had been lost thru

inexcusable negligence (Tuason vs. Court of Appeals,).

If a party has another adequate remedy available to him, which was

either a motion for new trial or appeal from adverse decisions of the lower

court, and he was not prevented by fraud, accident, mistake or excusable

negligence from filing such motion or taking the appeal he cannot avail

himself of the relief provided in Rule 38 (Somoso vs. CA,).

A party is not entitled to relief under Rule 38, Section 2, of the

Rules of Court if he was not prevented from filing his notice of appeal by

fraud, accident, mistake, or excusable negligence. Such relief will not be

granted to a party who seeks to be relieved from the effects of the

judgment, when the loss of the remedy at law was due to his own

negligence or to a mistaken mode of procedure for that matter; otherwise,

the petition for relief will be tantamount to reviving the right of appeal,

which has already been lost either due to inexcusable negligence or due to

a mistake of procedure by counsel (Fukuzumi vs. Sanritsu Great

International Corporation,).

The petition for relief from judgment must be accompanied with

affidavits showing the fraud, accident, mistake, or excusable negligence

relied upon, and the facts constituting petitioner’s good and substantial

cause of action or defense (Public Estates Authority vs. Yujuico,). This is

an indispensable statutory requirement under Section 3, Rule 38 of the

1997 Revised Rules of Civil Procedure which must be strictly observed.

Affidavit of merit serves as the jurisdictional basis for a court to entertain

a petition for relief, and when a petition for relief is flawed by such serious

defect, the court with which the petition is filed is not called upon to

entertain the same (Torno vs. IAC,).

B. Fraud as ground must be extrinsic or collateral

Like annulment of judgment, the fraud as ground for filing a

petition for relief from judgment must be extrinsic or collateral. The facts

upon which the extrinsic fraud is based have not been controverted or

resolved in the case where the judgment sought to be annulled was

rendered. For this purpose, fraud is regarded as extrinsic or collateral

where it has prevented a party from having a trial or from presenting all of

his case to the court. Intrinsic fraud takes the form of acts of the party in a

litigation during the trial, such as the use of forged instruments or perjured

testimony which did not affect the presentation of the case, but did prevent

a fair and just determination of the case (Libudan vs. Gil,).

C. Mistake and excusable negligence as a grounds

The mistake contemplated by Rule 38 of the Rules of Court

pertains generally to mistake of fact, not of law, which relates to the case.

The word “mistake” which grants relief from judgment does not apply and

was never intended to apply to a judicial error which the court might have

committed in the trial. Such error may be corrected by means of an appeal

(Agan vs. Heirs of Sps. Andres Nueva and Diosdada Nueva,), and not by

petition for relief form judgment.

In Madarang vs. Morales, , it was held that a failure of petitioners’

former counsel to file the notice of appeal within the reglementary period

is not excusable negligence. The High Court said that that kind of an

argument stereotypes and demeans senior citizens. According to the High

Court, this cannot be done as it asked to assume that a person with

advanced age is prone to incompetence.

D. Reglementary period, when reckoned

The 60-day period must be counted after petitioner learns of the

judgment or final order, and the 6-month period is counted from the

finality of judgment or final order. These are the so-called double period.

These double periods are both inextendible and uninterruptible; is

jurisdictional and should be strictly complied with (Madarang vs.

Morales,).

The 60-day reglementary period shall reckone from the time the

party’s counsel receives notice of the decision for notice to counsel of the

decision is notice to the party for purposes of Section 3 of Rule 38

(Mercury Drugs Corporation vs. CA,).

The grace period allowed by the rule within which to file petition for

relief from judgment must be taken as absolutely fixed, inextendible,

never interrupted and cannot be subjected to any condition or contingency.

Because the period fixed is itself devised to meet a condition or

contingency, the equitable remedy is an act of grace, as it were, designed

to give the aggrieved party another and last chance and failure to avail of

such last chance within the grace period fixed is fatal (Martinez vs.

Workmen's Compensation Commission,).

E. Must be filed in the same court that rendered the judgment

It must be filed in the same court that that rendered the judgment

and in the same cause wherein the judgment was rendered. If the court

finds the allegations of the petition to be true, it shall set aside the

judgment and try the principal case upon its merits as if a timely motion

for new trial had been granted therein (Braca vs. Tan,).

F. Petition for relief is only available against a final and executory

judgment

In Gomez vs.Montalban, , it was held that petitioner’s petition for

relief from judgment under Rule 38 of the Rules of Court was premature

and inappropriate, because when he filed his petition for relief the

judgment on May 28, 2004 against the judgment dated May 4, 2004 which

he received on May 14, 2004, the judgment had not attained finality, and

the 5-day period within which to file a motion for reconsideration or

appeal had not yet lapsed.

G. Petition is deniable when the loss of the remedy at law was due

to his own negligence or mistake

The petition for relief from judgment will not be granted to a party

who seeks to be relieved from the effects of the judgment when the loss of

the remedy at law was due to his own negligence or to a mistaken mode of

procedure for that matter, because if it will be granted the petition for

relief will be tantamount to reviving the right of appeal, a remedy which

has already been lost either due to inexcusable negligence or due to a

mistake of procedure by counsel (Fukuzumi vs. Sanritsu Great

International Corporation,).

H. Cases:

When a petition for relief is filed 61 days from receipt of the notice

of dismissal or one day late, the petition for relief must be ordered denied.

While it is true that the law gives an exception or ‘last chance’ of a timely

petition for relief from judgment within the reglementary period (within

60 days from knowledge and 6 months from entry of judgment) under

Rule 38 supra, but such grace period must be taken as 'absolutely fixed,

inextendible, never interrupted and cannot be subject to any condition or

contingency. Because the period fixed is itself devised to meet a condition

or contingency (fraud, accident, mistake or excusable neglect), the

equitable remedy is an act of grace, as it were, designed to give the

ag­grieved party another and last chance' "and failure to avail of such last

chance within the grace period fixed by the statute or Rules of Court is

fatal." (Turqueza vs. Hernando,).

A petition for relief which was filed 61 days from receipt of the

notice of dismissal or one day late was held filed out of time (Philippine

Rabbit Bus Lines vs. Judge Arciaga,).

The High Court said in Philippine Rabbit Bus Lines:

“It is undisputed that the Petition for Relief in this

case was filed 61 days from receipt of the notice of

dismissal or one day late. In fact, the records show that

counsel for private respondent learned of the dismissal on

the same day, April 29, 1967, when he arrived late for the

hearing so that the Petition for Relief was at least eight (8)

days late. The records further show that counsel for private

respondent did not move for reconsideration of the Order of

dismissal, nor for new trial. Neither did he appeal, thereby

allowing the decision to become final and executory. As a

last resort, he could have availed of the sixty day period

provided for by Rule 38 to file a Petition for Relief from

judgment but again he allowed this opportu­nity to lapse.

Indeed, to him is applicable, the well known maxim that

"equity aids the vigilant, not those who slumber on their

rights." (Henson v. Director of Lands, 55 Phil. 586).

In the case of Turqueza v. Hernando (97 SCRA 488

[1980]) the Supreme Court in disallowing the reopening of

the case which has become final, ruled that there is no

justification in law and in fact, for respondent judge's void

act of ordering the reopening of the case which has become

final and executory.

Thus, the Court held:

"The Court has said time and again that

the doctrine of finality of judgments is grounded

on fundamental considerations of public policy

and sound practice that at the risk of occasional

error the judgments of courts must become final

at some definite date fixed by law. The law

gives an exception or 'last chance’ of a timely

petition for relief from judgment within the

reglementary period (within 60 days from

knowledge and 6 months from entry of

judgment) under Rule 38 supra, but such grace

period must be taken as 'absolutely fixed,

inextendible, never interrupted and cannot be

subject to any condition or contingency.

Because the period fixed is itself devised to

meet a condition or contingency (fraud,

accident, mistake or excusable neg­lect), the

equitable remedy is an act of grace, as it were,

designed to give the ag­grieved party another

and last chance' "and failure to avail of such last

chance within the grace period fixed by the

statute or Rules of Court is fatal." (Turqueza v.

Hernando, supra).

In expressly reiterating the above-quoted decision the

Supreme Court in Arcilla v. Arcilla (138 SCRA 56 [1985]),

held that the Rule is that, for a petition for relief under Rule

38 to be entertained by the court, the petitioner must

satisfactorily show that he has faithfully and strictly

complied with the provisions of said Rule. Consequently,

it is incumbent upon the petitioner to show that the said

petition was filed within the reglementary period specified

in Sec. 3, of the same, otherwise on this ground alone, the

petition should be dismissed.”

I. Petition for relief will not lie when plaintiff filed to personally

notify defendant of court processes

Petition for relief will not lie if plaintiff fails to notify the

defendant of court processes and/or fails to search for the address of

defendant so that he may be properly notified by the Court. Plaintiff is not

duty bound to personally notify the defendant of court processes. It is not

the plaintiff’s duty to search for the defendant’s address so that they may

be properly notified by the Court. This is not the kind of fraud

contemplated by law. Bad faith cannot be presumed from inaction where

there is no duty to act. The grounds not having been clearly established,

petition for relief will not lie (David vs. CA,).

J. Counsel’s negligence not a ground for petition for relief from

judgment

The excusable negligence referred to in Rule 38 to warrant the

filing of the petition for relief from judgment is negligence of party NOT

negligence of counsel (Somoso vs. CA,). However, where reckless or

gross negligence of counsel deprives the client of due process of law; or

(2) when its application will result in outright deprivation of the client’s

liberty or property; or (3) where the interests of justice so require, the

courts must step in and accord relief to a client who suffered thereby

(Sarraga, Sr., vs. Banco Filipino Savings and Mortgage Bank,).

K. Petition for relief from judgment filed out of time shall not be

granted, exceptions

The following are the exceptions:

(1) where reckless or gross negligence of counsel

deprives the client of due process of law;

(2) when its application will result in outright

deprivation of the client’s liberty or property; or

(3) where the interests of justice so require (Sarraga,

Sr., vs. Banco Filipino Savings and Mortgage Bank, ).

In People’s Homesite and Housing Corporation vs.), it was held

that the counsel’s client was denied of due process when the lawyer failed

to inform his clients of the scheduled hearing of the case and the hearing

proceeded despite the client’s absence. In that case, the Court sensed that

there was something fishy with the actuations of the lawyer which

deprived client of their day in court.

In Sarraga vs. Banco Filipino Savings and Mortgage Bank, , the

negligence of newly-hired clerk with lack of work experience in the law

office who merely left the court order in her desk and eventually

misplaced it and failed to bring the matter to the attention of the lawyer

before the lawyer was appointed as Senior State Prosecutor in the

Department of Justice is gross negligence on the part of the lawyer, and

because such negligence will result in the deprivation of the client’s

property petition for relief from judgment was allowed. Obtaining these

circumstances, the client should not be made to suffer the consequences.

L. Court can grant the relief different from that what is prayed in

the petition

Obviously, the court can grant relief which is different from that

what is prayed for in the petition. The prayer in a petition for relief from

judgment under Rule 38 is not necessarily the same prayer in the

petitioner's complaint, answer or other basic pleading, because once a

petition for relief is granted and the judgment subject thereof set aside, and

further proceedings are thereafter had, the court in its judgment on the

merits may properly grant the relief sought in the petitioner's basic

pleadings, although different from that stated in his petition for relief

(Cheesman vs. IAC,).

---0---

PETITION FOR RELIEF IS DENIED; REMEDY

A. If denied, petition for certiorari under Rule 65 may be filed

When a petition for relief for judgment is denied, the aggrieved

party may file a petition for certiorari under Rule 65 of the 1997 Revised

Rules of Civil Procedure.

Section 1(b), Rule 41 of the 1997 Revised Rules of Civil Procedure

is clear. It says that no appeal may be taken from an order denying a

petition for relief from judgment, and instead, the aggrieved party may file

an appropriate special civil action under Rule 65.

The normal remedy against a judgment denying relief under Rule

38 is appeal wherein the aggrieved party may assail the judgment on the

merits, upon the ground that it is not supported by the evidence or it is

contrary to law. However, if appeal would not be a remedy adequate under

the circumstances, since it would not promptly relieve the aggrieved party

from the injurious effects of acts of the trial court or tribunal wherein in

the latter for example has authorized execution of the judgment, a resort to

the special civil action of certiorari may exceptionally be allowed.21

---0---

REMEDY OF ADVERSE PARTY

IF PETITION FOR RELIEF IS GRANTED

How if the petition for relief from judgment is granted, what is the

remedy available to the adverse party?

Wait for the judgment, and if the judgment is not favorable, then

file a notice of appeal. This is because an interlocutory order generally

cannot be appealed separately from the judgment. It is only after a

21

PCGG vs. Sandiganbayan, G.R. No. 100733, June 18, 1992 citing Saludes vs. Pajarillo,

78 Phil. 754; Liwanag vs. Castillo, Oct. 20, 1959; Occeña vs. Jabson, Oct. 29, 1986,

Silvestre vs. Torres, 57 Phil. 885; Pachoco vs. Tumungday, May 25, 1960, Lopez vs.

Alvendia, Dec. 24, 1964, cited in Moran, op. cit., Vol. 3, p. 177.

judgment has been rendered in the case that the ground for the appeal of

the interlocutory order may be included in the appeal of the judgment

itself (Silverio, Jr. vs. CA, G.R. No. 178933, September 16, 2009).

But if the order granting the petition for relief is believed to be

tainted with graved abuse of discretion, though not covered by Section

1(b), Rule 41 of the 1997 Revised Rules of Civil Procedure as it only

covers a situation wherein a petition for relief from judgment is denied

allowing the aggrieved party to resort to special civil action under Rule 65,

if tainted with graved abuse of discretion, the adverse party may still resort

to filing a petition certiorari under Rule 65 of the 1997 Revised Rules of

Civil Procedure by authority of Section 1(c), Rule 41 of the same Rules,

because the order interlocutory nature; an an order is one which does not

dispose of the case completely but leaves something to be decided upon

(Tan vs. Republic,). It is only when such interlocutory order was rendered

without or in excess of jurisdiction or with grave abuse of discretion that

certiorari under Rule 65 may be resorted to (Silverio, Jr. vs. CA,).

B. Motion for reconsideration is a requirement

A motion for reconsideration of the order denying the petition for

relief from judgment is the plain, speedy, and adequate remedy in the

ordinary course of law. Therefore it is a mandatory requirement to resort

first to the filing of a motion for reconsideration.

In Madarang vs. Morales, , the petition for certiorari assailing the

denial of the petition for relief from judgment was ordered dismissed for

failure of the petitioner to avail first the remedy of filing a motion for

reconsideration.

---0---

ANNULMENT OF JUDGMENT

(Rule 47)

A. Concept, where to file and grounds

Annulment of judgment under Rule 47 of the 1997 Revised Rules

of Civil Procedure is a recourse equitable in character and allowed only in

exceptional cases where the ordinary remedies of new trial, appeal,

petition for relief or other appropriate remedies are no longer available

through no fault of petitioner.22

Rule 47 of the 1997 Revised Rules of Civil Procedure covers

annulment by the Court of Appeals of judgments or final orders and

resolutions in civil actions of Regional Trial Courts for which the ordinary

remedies of new trial, appeal, petition for relief or other appropriate

remedies could no longer be availed of through no fault of the petitioner. It

is a remedy in law independent of the case where the judgment sought to

be annulled is rendered (Macalalag vs. Ombusdman,).

This remedy may not be invoked:

(1) where the party has availed himself of the remedy of

new trial, appeal, petition for relief or other appropriate

remedy and lost therefrom, or

(2) where he has failed to avail himself of those remedies

through his own fault or negligence (Macalalag vs.

Ombusdman,).

All annulment of judgment shall be within the exclusive original

jurisdiction of the Court of Appeals.

The grounds are extrinsic fraud and lack of jurisdiction (Paulino

vs. CA,). Through jurisprudential pronouncements, denial of due process

is included as an additional ground (Biaco vs. Philippine Countryside

Rural Bank, G).

22

City Government of Tagaytay vs. Judge Guerrero, G.R. Nos. 140743 & 140745 and

Justice Ameurfina Melencio-Herrera vs. Judge Guerrero, G.R. Nos. 141451-52,

September 17, 2009.

B. Fraud as ground, extrinsic or collateral not intrinsic

Fraud has been regarded as extrinsic or collateral. It is only

extrinsic or collateral fraud NOT intrinsic fraud that can serve as a basis

for annulment of judgment (Macabingkil vs. People's Homesite and

Housing Corporation,). To become a basis for annulment of judgment,

fraud has to be extrinsic or actual (Ancheta vs. Guersey-Dalaygon,), and

refers to acts outside the trial (Ybañez vs. CA,).

Fraud is extrinsic or collateral where it is on the effect of which

prevent a party from having a trial, or real contest, or from presenting all

of his case to the court, or where it operates upon matters pertaining, not to

the judgment itself, but to the manner in which it was procured so that

there is not a fair submission of the controversy (Macabingkil vs. People's

Homesite and Housing Corporation,). It takes the form of acts of a party

in a litigation during the trial, such as the use of forged instruments or

perjured testimony, which did not affect the presentation of the case, but

did prevent a fair and just determination of the case (Libudan vs. Palma,).

Extrinsic fraud refers to any fraudulent act of the prevailing party

in the litigation which is committed outside of the trial of the case,

whereby the defeated party has been prevented from exhibiting fully his

side of the case, by fraud or deception practiced on him by his opponent

(Macabingkil vs. People's), or acts of a party at a trial which prevented a

fair and just determination of the case and which could have been litigated

and determined at the trial or adjudication of the case. Examples of

intrinsic fraud are falsification and false testimony (Ybañez vs. CA,).

According to Ancheta vs. Guersey-Dalaygon, , fraud takes on

different shapes and faces. In that case, the High Court stated that “man in

his ingenuity and fertile imagination will always contrive new schemes to

fool the unwary.”

Said the High Court:

“There is extrinsic fraud within the meaning of Sec. 9

par. (2), of B.P. Blg. 129, where it is one the effect of

which prevents a party from hearing a trial, or real contest,

or from presenting all of his case to the court, or where it

operates upon matters, not pertaining to the judgment itself,

but to the manner in which it was procured so that there is

not a fair submission of the controversy. In other words,

extrinsic fraud refers to any fraudulent act of the prevailing

party in the litigation which is committed outside of the

trial of the case, whereby the defeated party has been

prevented from exhibiting fully his side of the case by fraud

or deception practiced on him by his opponent. Fraud is

extrinsic where the unsuccessful party has been prevented

from exhibiting fully his case, by fraud or deception

practiced on him by his opponent, as by keeping him away

from court, a false promise of a compromise; or where the

defendant never had any knowledge of the suit, being kept

in ignorance by the acts of the plaintiff; or where an

attorney fraudulently or without authority connives at his

defeat; these and similar cases which show that there has

never been a real contest in the trial or hearing of the case

are reasons for which a new suit may be sustained to set

aside and annul the former judgment and open the case for

a new and fair hearing.”

C. Extrinsic fraud already availed of as ground in petition for

relief or new trial, barred.

If extrinsic fraud was availed of or could have been availed of in a

motion for new trial or petition for relief, the extrinsic fraud can no longer

be availed of as a ground for annulment of judgment. It is considered

barred.

In Tolentino vs. Judge Leviste, , the High Court said that it is

provided in Section 2 of Rule 47 that extrinsic fraud shall not be a valid

ground if it was availed of, or could have been availed of, in a motion for

new trial or petition for relief. In other words, it is effectively barred if it

could have been raised as a ground in an available remedial measure.

D. Extrinsic fraud, prescriptive period

According to Article 1391 of the Civil Code of the Philippines, in

case of fraud an action for annulment must be brought within four (4)

years from the time of the discovery of the same. It commences to run

from the discovery of the fraud or fraudulent act or acts. Therefore,

annulment of judgment based on extrinsic fraud under Rule 47 of the 1997

Revised Rules of Civil Procedure must be brought within four (4) years

from the discovery of the fraud (Ancheta vs. Guersey-Dalaygon, G).

E. Intrinsic fraud, concept, examples and cases

Intrinsic fraud is not sufficient to attack a judgment (Yatco vs.

Sumagui,).

F. Introduction of false affidavit purely intrinsic

The use of a false affidavit of loss is intrinsic because it is similar

to the use of it during trial of forged instruments or perjured testimony. It

does not constitute extrinsic fraud to warrant the invalidation of a final

judgment (Demetriou vs. CA, The use of a forged instrument constituted

only intrinsic fraud for while perhaps it prevented a fair and just

determination of a case, the use of such instrument or testimony did not

prevent the adverse party from presenting his case fully and fairly

(Palanca vs. Republic,).

G. Lack of jurisdiction as a ground for annulment

Lack of jurisdiction as a ground for annulment of judgment refers

to either lack of jurisdiction over the person of the defending party or over

the subject matter of the claim (Paulino vs. CA, June 04, 2014).

A petition for annulment of judgment must be based on lack of

jurisdiction. Meaning, absence of or no jurisdiction; and that the court

should not have taken cognizance of the petition because the law does not

vest it with jurisdiction over the subject matter. The petitioner must show

not merely an abuse of jurisdictional discretion but an absolute lack of

jurisdiction (Durisol Philippines, Inc. vs. CA,).

If a complaint is filed and there is absence, or lack, of jurisdiction,

and the trial court dismisses the case outright, the dismissal cannot be a

subject of annulment of judgment. Appeal or certiorari under Rule 65, as

the case may be, is the proper remedy, or to re-file the case to the

appropriate court having jurisdiction over the case or subject matter.

The High Court in Paulino vs. CA, , gave emphasis that if the case

is dismissed on ground of lack of jurisdiction, the dismissal cannot be a

subject of an annulment of judgment. And no need to allege in the petition

that the ordinary remedy of new trial or reconsideration of the final order

or judgment or appeal therefrom is no longer available through no fault of

his own, precisely because the judgment rendered or the final order issued

by the RTC without jurisdiction is null and void. The High Court said in

Paulino:

“The Court finds the petitions devoid of merit.

Under Section 2 of Rule 47, the only grounds for

annulment of judgment are extrinsic fraud and lack of

jurisdiction. Lack of jurisdiction as a ground for annulment

of judgment refers to either lack of jurisdiction over the

person of the defending party or over the subject matter of

the claim. In case of absence, or lack, of jurisdiction, a

court should not take cognizance of the case.

In these cases, the petition for annulment was based

on lack of jurisdiction over the subject matter. The rule is

that where there is want of jurisdiction over a subject

matter, the judgment is rendered null and void. A void

judgment is in legal effect no judgment, by which no rights

are divested, from which no right can be obtained, which

neither binds nor bars any one, and under which all acts

performed and all claims flowing out are void. It is not a

decision in contemplation of law and, hence, it can never

become executory. It also follows that such a void

judgment cannot constitute a bar to another case by reason

of res judicata.

Accordingly, the Court agrees with the CA that LRA

was not estopped from assailing the July 20, 2011 RTC

Decision because it never attained finality for being null

and void, having been rendered by a court without

jurisdiction over the reconstitution proceedings.

As early as the case of Strait Times, Inc. v. CA, the

Court has held that when the owner’s duplicate certificate

of title has not been lost, but is, in fact, in the possession of

another person, then the reconstituted certificate is void,

because the court that rendered the decision had no

jurisdiction. Reconstitution can be validly made only in

case of loss of the original certificate. This rule was

reiterated in the cases of Villamayor v. Arante, Rexlon

Realty Group, Inc. v. Court of Appeals, Eastworld Motor

Industries Corporation v. Skunac Corporation, Rodriguez v.

Lim, Villanueva v. Viloria, and Camitan v. Fidelity

Investment Corporation. Thus, with evidence that the

original copy of the TCT was not lost during the

conflagration that hit the Quezon City Hall and that the

owner’s duplicate copy of the title was actually in the

possession of another, the RTC decision was null and void

for lack of jurisdiction.

For the aforecited reason, the Court agrees that the

public respondent correctly availed of the remedy of

petition for annulment of judgment under Rule 47 without

need of exhausting other ordinary remedies of new trial,

appeal, petition for relief, or other appropriate remedies

because the RTC judgment was null and void.

Indeed, where a petition for annulment of a judgment

or a final order of the RTC filed under Rule 47 of the Rules

of Court is grounded on lack of jurisdiction over the person

of the respondent or over the nature or subject of the action,

the petitioner need not allege in the petition that the

ordinary remedy of new trial or reconsideration of the final

order or judgment or appeal therefrom is no longer

available through no fault of his own, precisely because the

judgment rendered or the final order issued by the RTC

without jurisdiction is null and void and may be assailed

any time either collaterally or in a direct action or by

resisting such judgment or final order in any action or

proceeding whenever it is invoked, unless barred by

laches.”

In Ybañez vs. CA, The High Court dismissed the annulment of

judgment case, because what was filed with the CA was an annulment of

judgment and not petition for review under Rule 42. So, when the RTC

ruled on the issue of validity of the substituted service of summons over

the persons of the petitioners in the MTC level, it exercised its appellate

jurisdiction, thus, not a ground for annulment of judgment.

H. Denial of due process, as a ground

Section 2 of the said Rule provides that the annulment may be

based only on the grounds of extrinsic fraud and lack of jurisdiction,

although jurisprudence recognizes denial of due process as an additional

ground.

I. Cases

Demetriou case was permitted by the considered by the High

Court as annulment of judgment case but not on extrinsic fraud but on

lack of jurisdiction.

In Demetriou vs. CA, , petitioners and private respondent became

co-owners of the realty brought about by the execution of deeds of sale

executed by respondents’ co-owners in favor of the petitioner which at

that time the property was then under lease agreement. That time copy of

the title was delivered to petitioners, but because of the existing lease

agreement, petitioners waited for the lease agreement to expire. At the

registry of deeds, upon termination of the lease agreement, petitioners

tried to facilitate the transfer of the title over portions pertaining to them

but they failed because the duplicate owner’s copy of the title had already

been cancelled by virtue of a court order upon petitioner’s co-owners’

petition for issuance of the second owner’s duplicate title by reason of its

loss as it was destroyed by the typhoon. Because the use of a false

affidavit of loss does not constitute extrinsic fraud to warrant the

invalidation of a final judgment, the CA dismissed the action. However,

the High Court reversed the Court of Appeals’ ruling and remanded the

case for further proceedings. The High Court said:

“On the basis of these allegations the appellate court

ruled that the fraud alleged was, if at all, only intrinsic and

not extrinsic in character:

An action to annul a final judgment on the

ground of fraud will lie only if the fraud is

extrinsic or collateral in character. Extrinsic

fraud refers to any fraudulent act of the

prevailing party in the litigation which is

committed outside of the trial of the case,

whereby the defeated party has been prevented

from exhibiting fully his side of the case, by

fraud or deception practiced on him by his

opponent (Macabingkil vs. People's Homesite

and Housing Corporation, 72 SCRA 326 cited in

Canlas vs. CA, 164 SCRA 160). On the other

hand, intrinsic fraud takes the form of "acts of a

party in a litigation during the trial such as the

use of forged or false document or perjured

testimony, which did not affect the presentation

of the case, but did prevent a fair and just

determination of the case" (Libudan vs. Gil, 45

SCRA 17). In the present petition, the allegation

of fraud involves admission by the respondent

court of an alleged false affidavit of loss, which

alleged fraud is intrinsic in character. Thus, as

the alleged fraud committed by the private

respondent is not extrinsic in character, the

instant petition for annulment of the said

December 1, 1990 order of the lower court

should be dismissed.

The appellate court is certainly right in holding that

the use of a false affidavit of loss does not constitute

extrinsic fraud to warrant the invalidation of a final

judgment. The use of the alleged false affidavit of loss by

private respondent is similar to the use during trial of

forged instruments or perjured testimony. In the leading

case of Palanca v. Republic, it was held that the use of a

forged instrument constituted only intrinsic fraud for while

perhaps it prevented a fair and just determination of a case,

the use of such instrument or testimony did not prevent the

adverse party from presenting his case fully and fairly. In

the case at bar, petitioners were not really kept out of the

proceedings because of the fraudulent acts of the private

respondent. They could have rebutted or opposed the use

of the affidavit and shown its falsity since they were

theoretically parties in the case to whom notice had been

duly given.

But a judgment otherwise final may be annulled not

only on the ground of extrinsic fraud but also because of

lack of jurisdiction of the court which rendered it. In Serra

Sera v. Court of Appeals, on facts analogous to those

involved in this case, this Court already held that if a

certificate of title has not been lost but is in fact in the

possession of another person, the reconstituted title is void

and the court rendering the decision has not acquired

jurisdiction. Consequently the decision may be attacked

any time. Indeed, Rep. Act No. 26, § 18 provides that "in

case a certificate of title, considered lost or destroyed be

found or recovered, the same shall prevail over the

reconstituted certificate of title." It was, therefore, error for

the Court of Appeals to dismiss the petition for annulment

of judgment of the petitioners.”

J. Withdrawal of the appeal will not give rise to the filing of

annulment of judgment

Resorting to the filing of a petition for certiorari after the trial court

had issued a writ of execution pending appeals and later on withdrew the

appeal, the withdrawal of the appeal will not give rise to the filing of

annulment of judgment (Philippine Tourism Authority vs. Philippine

Golf Development & Equipment,).

K. Taxpayer is a proper party in annulment of judgment

In Remulla vs. Maliksi, , the High Court ruled that if the petition

for annulment of judgment is filed by a taxpayer who is representing the

interests of the province itself, the petition for annulment of judgment

should not be dismissed on ground of improper party. The petitioner being

a taxpayer who will be either benefited or injured by the execution of the

compromise judgment is a real-party-in-interest.

L. Dismissal on ground of specific reason, duty of the court

If the dismissal is for specific reasons the dismissal shall be clearly

set out so that the High Court will not be at sea if the resolution will be

later on assailed, and to make a definitive determination as to whether the

CA committed a reversible error in dismissing the petition. The dismissal

must contain the details whether the dismissal is on ground of extrinsic

fraud or lack of jurisdiction, procedural or substantial (Castigador vs.

Nicolas,).

M. Remedy if Court of Appeals denies or grants annulment of

judgment

If the Court of Appeals denies/dismisses or grants the petition for

annulment of judgment, the remedy available to the aggrieved party is to

file a Petition for Review on Certiorari under Rule 45 of the Revised Rules

of Civil Procedure with High Court (Supreme Court) within fifteen (15)

days from notice of the judgment or final order or resolution appealed

from, or of the denial of the petitioner's motion for new trial or

reconsideration filed in due time after notice of the judgment (Sections 1 23

and 2 24, Rule 45, 1997 Revised Rules of Civil Procedure).

---0---

---0---

SPECIAL CIVIL ACTIONS

Special civil actions are those actions governed by Rules 62,

63, 64, 65, 66, 67, 68 69, 70 and 71. These are the interpleader

(Rules 62), declaratory relief and similar remedies (Rules 63), review

of judgments and final orders of the Commission on Elections and

the Commission on Audit (Rules 64), certiorari, prohibition and

mandamus (Rules 65); quo warranto (Rules 66); expropriation

(Rules 67); foreclosure of real estate mortgage (Rules 68); partition

(Rules 69); forcible entry and unlawful detainer (Rules 70); and

contempt (Rules 71).

1. CERTIORARI (RULE 65)

23

Section 1. Filing of petition with Supreme Court. — A party desiring to appeal by

certiorari from a judgment or final order or resolution of the Court of Appeals, the

Sandiganbayan, the Regional Trial Court or other courts whenever authorized by law,

may file with the Supreme Court a verified petition for review on certiorari. The petition

shall raise only questions of law which must be distinctly set forth.

24

Section 2. Time for filing; extension. — The petition shall be filed within fifteen (15)

days from notice of the judgment or final order or resolution appealed from, or of the

denial of the petitioner's motion for new trial or reconsideration filed in due time after

notice of the judgment. On motion duly filed and served, with full payment of the docket

and other lawful fees and the deposit for costs before the expiration of the reglementary

period, the Supreme Court may for justifiable reasons grant an extension of thirty (30)

days only within which to file the petition.

A. The Rule

Certiorari under Rule 65 is a remedy designed for the correction of

errors of jurisdiction and not errors of judgment. An error of judgment is

not correctible by certiorari, and as long as the public respondent acted

with jurisdiction, any error committed by him or it in the exercise thereof

will amount to nothing more than an error of judgment which may be

reviewed or corrected only by appeal (Lim vs. Judge Vianzon,).

The acceptance of a petition for certiorari, as well as the grant of

due course thereto is, in general, addressed to the sound discretion of the

court. It must be stressed that certiorari, being an extraordinary remedy,

the party who seeks to avail of the same must strictly observe the rules laid

down by the law and non-observance thereof may not be brushed aside as

mere technicality (Garcia, Jr. vs. CA,).

The rule is that “when any tribunal, board or officer exercising

judicial or quasi-judicial functions has acted without or in excess its or his

jurisdiction, or with grave abuse of discretion amounting to lack or excess

of jurisdiction, and there is no appeal, or any plain, speedy, and adequate

remedy in the ordinary course of law, a person aggrieved thereby may file

a verified petition in the proper court, alleging the facts with certainty and

praying that judgment be rendered annulling or modifying the proceedings

of such tribunal, board or officer, and granting such incidental reliefs as

law and justice may require” (Section 1, Rule 65, 1997 Revised Rules of

Civil Procedure).

The petition shall be accompanied by a certified true copy of the

judgment, order or resolution subject thereof, copies of all pleadings and

documents relevant and pertinent thereto, and a sworn certification of non-

forum shopping as provided in the third paragraph of section 3, Rule 46

(Paragraph 2, Section 1, Rule 65, 1997 Revised Rules of Civil

Procedure).

In Lim vs. Judge Vianzon, , the held that the petition is

procedurally flawed as it is not accompanied by copies of relevant

pleadings mandated by the second paragraph of Section 1, Rule 65 of the

1997 Rules of Civil Procedure.

The High Court said in In Lim:

“Moreover, the instant petition is procedurally flawed

as it is not accompanied by copies of relevant pleadings

mandated by the second paragraph of Section 1, Rule 65 of

the 1997 Rules of Civil Procedure. Said provision reads as

follows:

SECTION 1. Petition for certiorari. –

When any tribunal, board or officer exercising

judicial or quasi-judicial functions has acted

without or in excess of its or his jurisdiction, or

with grave abuse of discretion amounting to

lack or excess of jurisdiction, and there is no

appeal, nor any plain, speedy, and adequate

remedy in the ordinary course of law, a person

aggrieved thereby may file a verified petition in

the proper court, alleging the facts with certainty

and praying that judgment be rendered annulling

or modifying the proceedings of such tribunal,

board or officer, and granting such incidental

reliefs as law and justice may require.

The petition shall be accompanied by a

certified true copy of the judgment, order or

resolution subject thereof, copies of all

pleadings and documents relevant and

pertinent thereto, and a sworn certification of

no-forum shopping as provided on the third

paragraph of Section 3, Rule 46. (Emphasis

supplied.)

Specifically, as pointed out by respondents, the

instant petition is not accompanied by copies of the Motion

to Dismiss and Motion for Reconsideration that petitioners

filed with the trial court. These are documents important for

the Court’s appraisal, evaluation and judicious disposition

of the case. Failing to fully apprise the Court of the relevant

details of the case, we find this egregious error a sufficient

cause for the dismissal of the instant petition. As held in

Santiago, Jr. v Bautista, to wit:

x x x the lower court’s holding that

appellant’s failure to accompany his petition

with a copy of the judgment or order subject

thereof together with copies of all pleadings and

documents relevant and pertinent thereto “is

fatal to his cause” is supported not only by the

provision of that Rule but by precedents as well.

A party who seeks to avail of the extraordinary remedy

of certiorari must observe the rules laid down by law, and

non-observance of the said rules may not be brushed aside

as mere technicality.”

B. Petition must be accompanied with clearly legible

duplicate original or certified copy of the judgment, order,

resolution, or ruling subject

What really required is that a petition shall be accompanied with

clearly legible duplicate original or certified copy of the judgment, order,

resolution, or ruling subject of the petition. Other relevant documents and

pleadings attached to it may be mere machine copies thereof is sufficient

(Garcia, Jr. vs. CA,).

The High Court in Garcia, Jr:

“We dismiss the Petition.

The acceptance of a petition for for certiorari, as well

as the grant of due course thereto is, in general, addressed

to the sound discretion of the court. It must be stressed that

certiorari, being an extraordinary remedy, the party who

seeks to avail of the same must strictly observe the rules

laid down by the law and non-observance thereof may not

be brushed aside as mere technicality.

In the matter of the requirement that a petition for

certiorari be accompanied by a certified true copy of the

judgment, order or resolution subject thereof, , Section 1,

Rule 65 of the 1997 Rules of Civil Procedure, as amended,

provides:

SECTION 1. Petition for certiorari.-

x x x x

The petition shall be accompanied by a

certified true copy of the judgment, order or

resolution subject thereof, copies of all

pleadings and documents relevant and

pertinent thereto, and a sworn certification of

non-forum shopping as provided in the third

paragraph of Section 3, Rule 46.

Significantly, Section 3, Rule 46 of the same Rules,

provides:

SECTION 3. Contents and filing of

petition; effect of non-compliance with

requirements. –

x x x x

It shall be filed in seven (7) clearly

legible copies together with proof of service

thereof on the respondent with the original

copy intended for the court indicated as such

by the petitioner, and shall be accompanied by

a clearly legible duplicate original or

certified true copy of the judgment, order,

resolution, or ruling subject thereof, such

material portions of the record as are referred

to therein, and other document or pertinent

thereto. The certification shall be

accomplished by the proper clerk of court or

by his duly authorized representative, or by

the proper officer of the court, tribunal,

agency or office involved or by his duly

authorized representative. The other requisite

number of copies of the petition shall be

accompanied by clearly legible plain copies of

all documents attached to the original.

x x x x

The failure of the petitioner to comply

with any of the foregoing requirements shall

be sufficient ground for the dismissal of the

petition.”

It is true that Section 3 of Rule 46 does not require

that all supporting papers and documents accompanying a

petition be duplicate originals or certified true copies.

However, it explicitly directs that all cases originally filed

in the Court of Appeals shall be accompanied by a clearly

legible duplicate original or certified true copy of the

judgment, order, resolution or ruling subject thereof.

Similarly, under Rule 65, which covers certiorari,

prohibition and mandamus, petitions need to be

accompanied only by duplicate originals or certified true

copies of the questioned judgment, order or resolution.

Other releval documents and pleadings attached to it may

be mere machine copies thereof. In the case at bar,

petitioners failed to attach duplicate originals or certified

true copies of the assailed Orders of the RTC, dated 22

February 2005 and 7 April 2005. What they affixed were

machine or Xerox copies of the same. Plainly put,

petitioners contravened the obvious rudiments of the rules.

In Circular No. 3-96, we made the following

clarifications and supplemental rules on what is a duplicate

original or certified true copy:

1. The "duplicate original copy shall" be understood

to be that copy of the decision, judgment, resolution or

order which is intended for and furnished to a party in the

case or proceeding in the court or adjudicative body which

rendered and issued the same. The "certified true copy"

thereof shall be such other copy furnished to a party at his

instance or in his behalf, duly authenticated by the

authorized officers or representatives of the issuing entity

as hereinbefore specified.

2. The duplicate original copy must be duly signed or

initialed by the authorities or the corresponding officer or

representative of the issuing entity, or shall at least bear the

dry seal thereof or any other official indication of the

authenticity and completeness of such copy. For this

purpose, all courts, offices or agencies furnishing such

copies which may be used in accordance with Paragraph

(3) of Revised Circular No. 1-88 shall make arrangements

for and designate the personnel who shall be charged with

the implementation of this requirement.

3. The certified true copy must further comply with

all the regulations therefor of the issuing entity and it is the

authenticated original of such certified true copy, and not a

mere Xerox copy thereof, which shall be utilized as an

annex to the petition or other initiatory pleading.

4. Regardless of whether a duplicate original copy or

a certified true copy of the adjudicatory document is

annexed to the petition or initiatory pleading, the same

must be exact and complete copy of the original and all the

pages thereof must be clearly legible and printed on white

bond or equivalent paper of good quality with the same

dimensions as the original copy. Either of the aforesaid

copies shall be annexed to the original copy of the petition

or initiatory pleading filed in court, while plain copies

thereof may be attached to the other copies of the pleading.

5. It shall be the duty and responsibility of the party

using documents required by Paragraph (3) of Circular No.

1-88 to verify and ensure compliance with all the

requirements thereof as detailed in the proceeding

paragraphs. Failure to do so shall result in the rejection of

such annexes and the dismissal of the case. Subsequent

compliance shall not warrant any reconsideration unless the

court is fully satisfied that the noncompliance was not in

any way attributable to the party despite due diligence on

his part, and that there are highly justifiable and compelling

reasons for the court to make such other disposition as it

may deem just and equitable.

Based on the foregoing, it is incontrovertible that a

certified true copy is not a mere Xerox copy. Further, it is

imperative that the duplicate original copyu required by the

rules must be duly signed or initialed by the authorities or

the corresponding officer or representative of the issuing

entity, or shall at least bear the dry seal thereof or any other

official indication of the authenticity and completeness of

such copy. Petitioners’ Xerox copies are wanting in this

respect.

Petitioners seek a liberal application of the procedural

rules. For their failure to attach certified true copies of the

assailed orders of the RTC, petitioners place the blame on

the appellate court. Petitioners brazenly suggest that what

the Court of Appeals should have done was to issue an

Order directing them to comply with the rule on attaching

certified true copies, instead of dismissing the case on its

face. We do not see reason to grant liberality in the

application of the rules. It must be emphasized that the

liberality in the interpretation and application of the rules

applies only in proper cases and under justifiable causes

and circumstances. While it is true that litigation is not a

game of technicalities, it is equally tue that every case must

be prosecuted in accordance with the prescribed procedure

to insure an orderly and speedy administration of justice.

Only strong considerations of equity, which are wanting in

this case, will lead us to allow an exception to the

procedural rule in the interest of substantial justice. To

further suggest petitioners’ impervious attitude towards

rules, they even failed to attach certified true copies or

duplicate original copies of the assailed Orders in their

Motion for Reconsideration filed with the Court of

Appeals. Concomitant to a liberal application of the rules

of procedure should be an effort on the party invoking

liberality to at least explain its failure to comply with the

rules. Circular No. 3-96 is also unequivocal that it shall be

the duty and responsibility of the party to verify and ensure

compliance with all the requirements detailed therein. In

fact, failure to do so shall result in the rejection of such

annexes and the dismissal of the case.”

Take note the requirement provided in A.M. No. 10-3-7-SC

[Rules on e-Filing] and A.M. No. 10-3-7-SC [Rule for the Efficient

Use of Paper] dated September 10, 2013.

A. The assessment of treble costs provided in Section 8 of Rule 65,

1997 Revised Rules of Civil Procedure, as amended by A.M.

No. 07-7-12-SC is not automatic or mandatory

After the comment or other pleadings required by the court are

filed, or the time for the filing thereof has expired, the court may hear the

case or require the parties to submit memoranda. If, after such hearing or

filing of memoranda or upon the expiration of the period for filing, the

court finds that the allegations of the petition are true, it shall render

judgment for such relief to which the petitioner is entitled. However, the

court may dismiss the petition if it finds the same patently without merit or

prosecuted manifestly for delay, or if the questions raised therein are too

unsubstantial to require consideration. In such event, the court may award

in favor of the respondent treble costs solidarily against the petitioner and

counsel, in addition to subjecting counsel to administrative sanctions

under Rule 139 and 139-B of the Rules of Court (First paragraph,

Section 8 of Rule 65, as amended by A.M. No. 07-7-12-SC).

The Court may impose motu proprio, based on res ipso loquitur,

other disciplinary sanctions or measures on erring lawyers for patently

dilatory and unmeritorious petitions for certiorari (Second paragraph,

Section 8 of Rule 65, as amended by A.M. No. 07-7-12-SC).

The use of the word "may" in the last sentence of the second

paragraph or Section 8, Rule 65, indicates that the assessment of treble

costs is not automatic or mandatory. It merely gives the court the

discretion and latitude to impose further sanctions where a petition is

dismissed for being "patently without merit," "prosecuted manifestly for

delay," or upon finding that the questions raised in the petition for

certiorari were "too to require consideration" (City of Davao vs. CA, G.R.

No. 200538, Augsut 13, 2014).

The court is afforded judicial discretion in imposing treble costs,

there remains a need to show that it is sound and with basis - that is

"taking all the pertinent circumstances into due consicleration.'' (Diaz vs.

People, 180677,).

B. When is treble cost imposable or awarded?

The parties and their counsels resort to deplorable dilatory tactics

to frustrate the fruition of justice (City of Davao vs. CA,).

Examples:

1. When the losing litigant repeatedly frustrated the execution of a

final and executory decision (Central Surety and Insurance Company vs.

Planters Product, Inc, 546 Phil. 479 cited in City of Davao vs. CA, G.R.

No. 200538, August 13, 2014).

In Central Surety and Insurance Company vs. Planters Product,

Inc,), the execution was delayed for more than five years because of his

dilatory tactics. When the winning party sought the execution by motion

beyond the period, he still opposed it despite the fact that the period was

suspended because of reasons attributable to him.

2. Treble costs were awarded because of the deplorable course

resorted to by the losing litigants in the hope of evading manifest

obligations. The Court stated that it viewed with disfavor the unjustified

delay in the enforcen1ent or the final decision and orders in the said case.

Once a judgment becomes tinal and executory, the prevailing party should

not be denied the fruits of his victory by some subterfuge devised by the

losing party. Unjustified delay in the enforcement of a judgment sets at

naught the role of courts in disposing justiciable controversies with finality

(Spouses Aguilar vs. The Manila Banking Corporation,).

3. Treble costs were imposed because the parties took the law into

their own hands or resorted to a wrong remedy (Uypitching vs.

Quiamco,).

In Uypitching vs. Quiamco, , instead of bringing the proper civil

action necessary to acquire legal possession of a motorcycle, the petitioner

took the law into his own hands and seized it without a search warrant or

court order. Worse, in the course or the illegal seizure or the motorcycle,

the petitioner even mouthed a slanderous statement. By doing so, he

transgressed the proper norms of human relations.

4. In Solen vs. A-1 Investors., the party claimed that he was denied

due process when the court adjudged a case against him even if he was not

served the summons. He then resorted to filing a complaint for injunction

to stop the execution of the final judgment. The Court said that he should

have resorted to an action for annulment under Rule 47 and so awarded

treble costs against him.

In City of Davao vs. CA, G, the imposition of treble costs was

striken off by the High Court for failure of the Court of Appeals to explain

and/or to give any reason for such imposition.

C. Timeliness of the petition for certiorari

The petition shall be filed not later than sixty (60) days from notice

of judgment, order or resolution. In case a motion for reconsideration or

new trial is timely filed, whether such motion is required or not, the sixty

(60) day period shall be counted from notice of the denial of said motion

(Section 4, Rule 65, 1997 Revised Rules of Civil Procedure).

The 60-day period starts to run from the date petitioner receives

the assailed judgment, final order or resolution, or the denial of the motion

for reconsideration or new trial timely filed, whether such motion is

required or not (Vinuya vs. Secretary Romulo,).

To establish the timeliness of the petition for certiorari, the date of

receipt of the assailed judgment, final order or resolution or the denial of

the motion for reconsideration or new trial must be stated in the petition;

otherwise, the petition for certiorari must be dismissed. The importance of

the dates cannot be understated, for such dates determine the timeliness of

the filing of the petition for certiorari (Vinuya vs. Secretary Romulo).

D. Three (3) essential dates that must be stated in a petition for

certiorari brought under Rule 65

First, the date when notice of the judgment or final order or

resolution was received; second, when a motion for new trial or

reconsideration was filed; and third, when notice of the denial thereof was

received. Failure of petitioner to comply with this requirement shall be

sufficient ground for the dismissal of the petition. Substantial compliance

will not suffice in a matter involving strict observance with the Rules

(Tambong vs. R. Jorge Development Corporation).

The requirement of setting forth the three (3) dates in a petition for

certiorari under Rule 65 is for the purpose of determining its timeliness.

Such a petition is required to be filed not later than sixty (60) days from

notice of the judgment, order or Resolution sought to be assailed.

Therefore, that the petition for certiorari was filed forty-one (41) days

from receipt of the denial of the motion for reconsideration is hardly

relevant. The Court of Appeals was not in any position to determine when

this period commenced to run and whether the motion for reconsideration

itself was filed on time since the material dates were not stated. It should

not be assumed that in no event would the motion be filed later than

fifteen (15) days. Technical rules of procedure are not designed to frustrate

the ends of justice. These are provided to effect the proper and orderly

disposition of cases and thus effectively prevent the clogging of court

dockets. Utter disregard of the Rules cannot justly be rationalized by

harking on the policy of liberal construction (Santos vs. CA,).

E. Motion for reconsideration is required before filing a petition

for certiorari under Rule 65 in the Higher Court; exception

Petition for certiorari before a Higher Court will not prosper unless

the inferior court has been given, through a motion for reconsideration, a

chance to correct the errors imputed to it. EXCEPTIONS: (1) when the

issue raised is purely of law; (2) when public interest is involved, or (3) in

case of urgency (Philippine International Trading Corporation vs.

COA,).

F. Order denying the demurrer to evidence reviewable by

Certiorari under Section 1, Rule 65 of the Rules of Court

Generally, the order denying the motion for leave of court to file

demurrer to evidence or the demurrer itself shall not be reviewable by

appeal or by certiorari before judgment,25 because action on a demurrer or

on a motion to dismiss rests on the sound exercise of judicial discretion.26

It is an interlocutory order, not appealable neither can it be the subject of a

petition for certiorari.27

However, this admits an exception: when the denial of a demurrer

to evidence is attended by grave abuse of discretion, patently erroneous or

25Section 23, Rule 119, 2000 Revised Rules of Criminal Procedure. 26Tan vs. Court of Appeals, 347 Phil. 320, 329 (1997); Bernardo vs. Court of Appeals,

344 Phil. 335, 346 (1997) cited in Nicolas vs. Sandiganbayan and the companion case,

G.R. Nos. 175930-31, February 11, 2008. Also in People vs. Singh, G.R. No. 129782,

June 29, 2001, 360 SCRA 404; People vs. Mercado, No. L-33492, March 30, 1988, 159

SCRA 453 cited in People vs. Almendras, G.R. No. 145915, April 24, 2003. 27David vs. Rivera, 464 Phil. 1006; Tadeo vs. People, 360 Phil. 914, 919 (1998); Cruz vs.

People, 363 Phil. 156; Katigbak vs. Sandiganbayan, 453 Phil. 515 cited in Nicolas vs.

Sandiganbayan and the companion case, G.R. Nos. 175930-31, February 11, 2008.

issued with grave abuse of discretion.28 Though interlocutory in character,

an order denying a demurrer to evidence may be the subject of a certiorari

proceeding, provided the petitioner can show that it was issued with grave

abuse of discretion; and that appeal in due course is not plain, adequate or

speedy under the circumstances. When the plaintiff’s evidence is utterly

and patently insufficient to prove the complaint, it would be capricious for

a trial judge to deny the demurrer and to require the defendant to present

evidence to controvert a non- existing case. The denial of the demurrer to

evidence will constitute an unwelcome imposition on the court’s docket

and an assault on the defendant’s resources and peace of mind, and if

denied, it effectively denies justice.29

“Interlocutory” refers to intervening developments between the

commencement of a suit and its complete termination; hence, it is a

development that does not end the whole controversy. An “interlocutory

order” merely rules on an incidental issue and does not terminate or

finally dispose of the case; it leaves something to be done before the case

is finally decided on the merits (Marmot vs. Anacay)

.

G. A one liner sentence denying a motion for reconsideration to

the final order or judgment for lack of merit is not

certiorariable (Rule 65)

A one liner sentence denying a motion for reconsideration to the

final order or judgment for lack of merit is not certiorariable.

The filing of a motion to amend order denying the reconsideration

on ground of null and void as it does not conform to the requirements of

Section 14, Article VIII of the Constitution and Section 1, Rule 36 of the

1997 Revised Rules of Civil Procedure will not and does not toll the

running of the period towards finality. The aggrieved party may avail any

of the following remedies:

1. He may file a Motion for New Trial under Rule 37; or

28

People vs. Ong, G.R. No. 140904, October 9, 2000 citing Cruz vs. People (303 SCRA

533 [1999]). 29Choa vs. Choa, G.R. No. 143376, November 26, 2002.

2. He may file a Motion for Reconsideration, also under

Rule 37; or

3. He may file an Appeal under Rule 41; or

4. He may file a petition for Relief from Judgment under

Rule 38, or;

5. He may file a petition for Annulment of Judgment under

Rule 47.

A decision that has acquired finality becomes immutable and

unalterable and may no longer be modified in any respect even if the

modification is intended to correct erroneous conclusions of fact or law

and whether it will be made by the court that rendered it or by the highest

court of the land.30

0-0-0

APPEAL (RULE 41) VS. CERTIORARI (RULE 65)

“An appeal may be taken from a judgment or final

order that completely disposes of the case or of a particular

matter therein when declared by these Rules to be

appealable” (Section 1, Rule 41, 1997 Revised Rules of

Civil Procedure).

“When any tribunal, board or officer exercising

judicial or quasi-judicial functions has acted without or in

excess of jurisdiction, or with grave abuse of discretion

30Sofio vs. Valenzuela, G.R. No. 157810, February 15, 2012.

amounting to lack or excess of jurisdiction, and there is no

appeal, or any plain, speedy and adequate remedy in the

ordinary course of law, a person aggrieved thereby may file

a verified petition in the proper court, alleging the facts

with certainty and praying that judgment be rendered

annulling or modifying the proceedings of such tribunal,

board or officer, and granting such incidental reliefs as law

and justice may require” (Section 1, Rule 65, 1997 Revised

Rules of Civil Procedure).

Tible & Tible Company, Inc. vs. Royal Savings and Loan Association,

G.R. No. 155806, April 8, 2008 citing Madrigal Transport, Inc. vs.

Lapanday Holdings Corporation, G.R. No. 156067, August 11, 2004 d

discussed the differences between the remedies provided under Rule 41

and Rule 65, thus:

“Appeal and Certiorari Distinguished

Between an appeal and a petition for certiorari, there

are substantial distinctions which shall be explained below.

As to the Purpose. Certiorari is a remedy designed

for the correction of errors of jurisdiction, not errors of

judgment. In Pure Foods Corporation v. NLRC, we

explained the simple reason for the rule in this light:

“When a court exercises its jurisdiction, an

error committed while so engaged does not

deprive it of the jurisdiction being exercised

when the error is committed. If it did, every

error committed by a court would deprive it of

its jurisdiction and every erroneous judgment

would be a void judgment. This cannot be

allowed. The administration of justice would

not survive such a rule. Consequently, an error

of judgment that the court may commit in the

exercise of its jurisdiction is not correctable

through the original civil action of certiorari.”

The supervisory jurisdiction of a court over the

issuance of a writ of certiorari cannot be exercised for the

purpose of reviewing the intrinsic correctness of a

judgment of the lower court – on the basis either of the law

or the facts of the case, or of the wisdom or legal soundness

of the decision. Even if the findings of the court are

incorrect, as long as it has jurisdiction over the case, such

correction is normally beyond the province of certiorari.

Where the error is not one of jurisdiction, but of an error of

law or fact – a mistake of judgment – appeal is the remedy.

As to the Manner of Filing. Over an appeal, the CA

exercises its appellate jurisdiction and power of review.

Over a certiorari, the higher court uses its original

jurisdiction in accordance with its power of control and

supervision over the proceedings of lower courts.

An

appeal is thus a continuation of the original suit, while a

petition for certiorari is an original and independent action

that was not part of the trial that had resulted in the

rendition of the judgment or order complained of. The

parties to an appeal are the original parties to the action. In

contrast, the parties to a petition for certiorari are the

aggrieved party (who thereby becomes the petitioner)

against the lower court or quasi-judicial agency, and the

prevailing parties (the public and the private respondents,

respectively).

As to the Subject Matter. Only judgments or final

orders and those that the Rules of Court so declare are

appealable. Since the issue is jurisdiction, an original

action for certiorari may be directed against an

interlocutory order of the lower court prior to an appeal

from the judgment; or where there is no appeal or any

plain, speedy or adequate remedy.

As to the Period of Filing. Ordinary appeals should

be filed within fifteen days from the notice of judgment or

final order appealed from. Where a record on appeal is

required, the appellant must file a notice of appeal and a

record on appeal within thirty days from the said notice of

judgment or final order. A petition for review should be

filed and served within fifteen days from the notice of

denial of the decision, or of the petitioner’s timely filed

motion for new trial or motion for reconsideration. In an

appeal by certiorari, the petition should be filed also within

fifteen days from the notice of judgment or final order, or

of the denial of the petitioner’s motion for new trial or

motion for reconsideration.

On the other hand, a petition for certiorari should be

filed not later than sixty days from the notice of judgment,

order, or resolution. If a motion for new trial or motion for

reconsideration was timely filed, the period shall be

counted from the denial of the motion.

As to the Need for a Motion for Reconsideration.

A motion for reconsideration is generally required prior to

the filing of a petition for certiorari, in order to afford the

tribunal an opportunity to correct the alleged errors. Note

also that this motion is a plain and adequate remedy

expressly available under the law. Such motion is not

required before appealing a judgment or final order.”

With these distinctions, it is plainly discernible why a

party is precluded from filing a petition for certiorari when

appeal is available, or why the two remedies of appeal and

certiorari are mutually exclusive and not alternative or

successive. Where appeal is available, certiorari will not

prosper, even if the ground availed of is grave abuse of

discretion.”

---0---

2. PROHIBITION

A. The Rule

When the proceedings of any tribunal, corporation, board, officer

or person, whether exercising judicial, quasi-judicial or ministerial

functions, are without or in excess of its or his jurisdiction, or with grave

abuse of discretion amounting to lack or excess of jurisdiction, and there is

no appeal or any other plain, speedy, and adequate remedy in the ordinary

course of law, a person aggrieved thereby may file a verified petition in

the proper court, alleging the facts with certainty and praying that

judgment be rendered commanding the respondent to desist from further

proceedings in the action or matter specified therein, or otherwise granting

such incidental reliefs as law and justice may require (Section 2, Rule 65,

1997 Revised Rules of Civil Procedure).

The petition shall likewise be accompanied by a certified true copy

of the judgment, order or resolution subject thereof, copies of all pleadings

and documents relevant and pertinent thereto, and a sworn certification of

non-forum shopping as provided in the third paragraph of section 3, Rule

46 (Paragraph 2, Section 2, Rule 65, 1997 Revised Rules of Civil

Procedure).

B. Writ of prohibition an extraordinary writ

A writ of prohibition is an extraordinary writ. It may be issued

only in the absence of a plain, speedy and adequate remedy in the ordinary

course of law Purok Bagong Silang Association, Inc. vs. Judge Yuipco,).

C. General rule, a writ of prohibition cannot be issued against an

inferior court, exception

A writ of prohibition will not be issued against an inferior court

unless the attention of the court whose proceedings are sought to be stayed

has been called to the alleged lack or excess of jurisdiction. The

foundation of this rule is the respect and consideration due to the lower

court and the expediency of preventing unnecessary litigation; it cannot be

presumed that the lower court would not properly rule on a jurisdictional

objection if it were property presented to it (Esquivel vs. Hon.

Ombudsman,).

D. Prohibition lies against judicial or ministerial functions, but

not against legislative or quasi-legislative functions

Prohibition lies against judicial or ministerial functions, but not

against legislative or quasi-legislative functions. Generally, the purpose of

a writ of prohibition is to keep a lower court within the limits of its

jurisdiction in order to maintain the administration of justice in orderly

channels. Prohibition is the proper remedy to afford relief against

usurpation of jurisdiction or power by an inferior court, or when, in the

exercise of jurisdiction in handling matters clearly within its cognizance

the inferior court transgresses the bounds prescribed to it by the law, or

where there is no adequate remedy available in the ordinary course of law

by which such relief can be obtained. Where the principal relief sought is

to invalidate an IRR, petitioners’ remedy is an ordinary action for its

nullification, an action which properly falls under the jurisdiction of the

Regional Trial Court. In any case, petitioners’ allegation that “respondents

are performing or threatening to perform functions without or in excess of

their jurisdiction” may appropriately be enjoined by the trial court through

a writ of injunction or a temporary restraining order (Holy Spirit

Homeowners’ Association vs. Defensor,).

E. Prohibition does not lie to restrain an act which is already a fait

accompli

Prohibition does not lie to restrain an act which is already a fait

accompli. The “operative fact” doctrine protecting vested rights bars the

grant of the writ of prohibition (Chavez vs. NHA,).

F. Petition for prohibition venue of action

The petition for prohibition filed by respondents is a special civil

action which may be filed in the Supreme Court, the Court of Appeals, the

Sandiganbayan or the regional trial court, as the case may be. It is also a

personal action because it does not affect the title to, or possession of real

property, or interest therein. Thus, it may be commenced and tried where

the plaintiff or any of the principal plaintiffs resides, or where the

defendant or any of the principal defendants resides, at the election of the

plaintiff. The choice of venue is sanctioned by Section 2, Rule 4 of the

Rules of Court (GSIS Board of Trustees vs. Velasco,).

---0---

1. MANDAMUS

A. The Rule

When any tribunal, corporation, board, officer or person

unlawfully neglects the performance of an act which the law specifically

enjoins as a duty resulting from an office, trust, or station, or unlawfully

excludes another from the use and enjoyment of a right or office to which

such other is entitled, and there is no other plain, speedy and adequate

remedy in the ordinary course of law, the person aggrieved thereby may

file a verified petition in the proper court, alleging the facts with certainty

and praying that judgment be rendered commanding the respondent,

immediately or at some other time to be specified by the court, to do the

act required to be done to protect the rights of the petitioner, and to pay the

damages sustained by the petitioner by reason of the wrongful acts of the

respondent (Section 3, Rule 65, 1997 Revised Rules of Civil Procedure).

The petition shall also contain a sworn certification of non-forum

shopping as provided in the third paragraph of section 3, Rule 46

(Paragraph 2, Section 3, Rule 65, 1997 Revised Rules of Civil

Procedure).

B. Concept of ministerial duty

A ministerial duty is one which is so clear and specific as to leave

no room for the exercise of discretion in its performance. It is a duty

which an officer performs in a given state of facts in a prescribed manner

in obedience to the mandate of legal authority, without regard to the

exercise of his/her own judgment upon the propriety of the act done

(Symaco v. Hon. Aquino,).

Mandamus lies to compel the performance, when refused, of a

ministerial duty, but not to compel the performance of a discretionary

duty. A purely ministerial act or duty is one which an officer or tribunal

performs in a given state of facts, in a prescribed manner, in obedience to

the mandate of a legal authority, without regard to or the exercise of his

own judgment upon the propriety or impropriety of the act done. The duty

is ministerial only when the discharge of the same requires neither the

exercise of official discretion or judgment. When an official is required

and authorized to do a prescribed act upon a prescribed contingency, his

functions are ministerial only, and mandamus may be issued to control his

action upon the happening of the contingency (Paloma vs. Mora,).

The remedy of mandamus is available only to compel the

performance of a ministerial duty. The distinction between a ministerial

and discretionary act is well delineated. A purely ministerial act or duty is

one which an officer or tribunal performs in a given state of facts, in a

prescribed manner, in obedience to the mandate of a legal authority,

without regard to or the exercise of his own judgment upon the propriety

or impropriety of the act done. If the law imposes a duty upon a public

officer and gives him the right to decide how or when the duty shall be

performed, such duty is discretionary and not ministerial. The duty is

ministerial only when the discharge of the same requires neither the

exercise of official discretion or judgment (Torregoza vs. CSC,).

For a writ of mandamus to be issued, it is essential that petitioner

should have a clear legal right to the thing demanded and it must be the

imperative duty of the respondent to perform the act required. The writ

neither confers powers nor imposes duties. It is simply a command to

exercise a power already possessed and to perform a duty already

imposed. Mandamus applies as a remedy only where petitioner’s right is

founded clearly in law and not when it is doubtful. The writ will not be

granted where its issuance would be unavailing, nugatory, or useless

(Philippine Coconut Authority vs. Primex Coco Products).

If the law imposes a duty upon a public officer and gives him the

right to decide how or when the duty shall be performed, such duty is

discretionary and not ministerial (Philippine Coconut Authority vs.

Primex Coco Products, Inc,).

Mandamus is an extraordinary writ and discretionary remedy and

should not be granted when it will achieve no beneficial result such as

when act sought to be compelled has been performed (Philippine Coconut

Authority vs. Primex Coco Products,).

Mandamus will not be issued to compel the renewal of a license

for a period which has expired. If the right sought to be enforced by writ

of mandamus is or has become a mere abstract right, enforcement of

which will be of no substantial or practical benefit to the plaintiff, the writ

will not issue though the applicant would otherwise be entitled to it. To

warrant the issuance of a writ of mandamus, it must appear that the writ

will be effectual as a remedy, it should be denied where it would be

useless by reason of events occurring subsequent to commencement

proceedings (Philippine Coconut Authority vs. Primex Coco Products,

Inc,).

Mandamus is never granted to compel the performance of an act

until there has been an actual, as distinguished from an anticipated, refusal

to act. This is true even if there is a strong presumption that the persons

whom it is sought to coerce by the writ will refuse to perform their duty

when the proper time arrives. Its function is to compel the performance of

a present existing duty as to which there is default. It is not granted to take

effect prospectively, and it contemplates the performance of an act which

is incumbent on respondent when the application for a writ is made

(Philippine Coconut Authority vs. Primex Coco Products,).

---0---

FINALITY OF JUDGMENT

AND ENTRY OF JUDGMENT

A. Doctrine of finality of judgment

The doctrine of finality of judgments is grounded on fundamental

considerations of public policy and sound practice that at the risk of

occasional errors, the judgments of courts must become final at some

definite date fixed by law (Torno vs. IAC, ).

After the lapse of the 15-day reglementary period to appeal, an

order becomes executory, and it goes beyond the jurisdiction of the court

that rendered it to further amend or revoke. The subsequent filing of a

Motion for Reconsideration cannot disturb the finality of a judgment, nor

restore jurisdiction that had already been lost. A final and executor

judgment or order cannot be modified in any respect, even if the

modification sought is to correct an erroneous conclusion by the court that

rendered it (Tupaz vs. Judge Apurillo,).

B. Date of entry of judgment, determinative of

1. for purposes of ensuring the finality of judgment for

execution purposes;

2. for purposes of determining the period to file petition for

relief from judgment; and

3. for purposes of determining the period to file action for

revival of judgment.

The 60-day period must be counted after petitioner learns of the

judgment or final order, and the 6-month period is counted from the

finality of judgment or final order. These are the so-called double period.

These double periods are both inextendible and uninterruptible; is

jurisdictional and should be strictly complied with (Madarang vs.

Morales,). The 60-day reglementary period shall reckone from the time

the party’s counsel receives notice of the decision for notice to counsel of

the decision is notice to the party for purposes of Section 3 of Rule 38

(Mercury Drugs Corporation vs. CA,).

---0---

LIBERAL INTERPRETATION OF THE RULES

A. The Rule

The 1997 Revised Rules of Civil Procedure shall be liberally

construed in order to promote their objective of securing a just, speedy and

inexpensive disposition of every action and proceeding (Section 6, Rule 1,

1997 Revised Rules of Civil Procedure).

B. Liberal interpretation on right to appeal

The right to appeal is not a natural right and is not part of due

process, but merely a statutory privilege to be exercised only in

accordance with the law. Being the party who sought to appeal, he must

comply with the requirements of the relevant rules; otherwise, he would

lose the statutory right to appeal (Magsino vs. Ocampo,).

The procedures regulating appeals as laid down in the Rules of

Court must be followed because strict compliance with them is

indispensable for the orderly and speedy disposition of justice (Polintan

vs. People,).

For appealed cases, the High Court in Galvez vs. CA, , recognized

three (3) guideposts for the Court of Appeals to consider in determining

whether or not the rules of procedures should be relaxed:

“First, not all pleadings and parts of case records are

required to be attached to the petition. Only those which are

relevant and pertinent must accompany it. The test of

relevancy is whether the document in question will support

the material allegations in the petition, whether said

document will make out a prima facie case of grave abuse

of discretion as to convince the court to give due course to

the petition.

Second, even if a document is relevant and pertinent

to the petition, it need not be appended if it is shown that

the contents thereof can also found in another document

already attached to the petition. Thus, if the material

allegations in a position paper are summarized in a

questioned judgment, it will suffice that only a certified

true copy of the judgment is attached.

Third, a petition lacking an essential pleading or part

of the case record may still be given due course or

reinstated (if earlier dismissed) upon showing that

petitioner later submitted the documents required, or that it

will serve the higher interest of justice that the case be

decided on the merits.”

C. Liberal interpretation on strict adherence to procedural

rules

The following are the justification for a court to suspend a strict

adherence to procedural rules:

(a) matters of life, liberty, honor or property in involved;

(b) the existence of special or compelling circumstances;

(c) the merits of the case;

(d) a cause not entirely attributable to the fault or

negligence of the party favored by the suspension of the

rules;

(e) a lack of any showing that the review sought is merely

frivolous and dilatory; and

(f) the fact that the other party will not be unjustly

prejudiced thereby (Sanchez vs. CA,).

---0---